You are on page 1of 259

OUTDOOR EMERGENCY CARE


QUIZ AND TEST


WORKBOOK
Chapter 1 .......................................................................................................4

Chapter 2......................................................................................................10

Chapter 3......................................................................................................16

Chapter 4 ......................................................................................................21

Chapter 5 .....................................................................................................28

Chapter 6 .....................................................................................................35

Chapter 7 .....................................................................................................54

Chapter 8 .....................................................................................................63

Chapter 9 .....................................................................................................69

Chapter 10....................................................................................................78

Chapter 11 ....................................................................................................84

Chapter 12....................................................................................................92

Chapter 13 ....................................................................................................97

Chapter 14 ..................................................................................................104

Chapter 15 ..................................................................................................110

Chapter 16 ..................................................................................................119

Chapter 17 ..................................................................................................124

Chapter 18 ..................................................................................................129

Chapter 19 ..................................................................................................137

Chapter 20..................................................................................................142

Chapter 21 ..................................................................................................157

Chapter 22 ..................................................................................................167

Chapter 23 ..................................................................................................173

Chapter 24..................................................................................................180
Chapter 25..................................................................................................185

Chapter 26..................................................................................................192

Chapter 27.................................................................................................. 197

Chapter 28 .................................................................................................203

Chapter 29 .................................................................................................208

Chapter 30 ..................................................................................................214

Chapter 31..................................................................................................224

Chapter 32 ..................................................................................................231

Chapter 33 .................................................................................................236

Chapter 34 .................................................................................................241

Chapter 35 .................................................................................................248

Chapter 36 .................................................................................................254
CHAPTER 1
“Introduction to Outdoor Emergency Care”

QUIZ

1. Good Samaritan laws will generally provide liability protection to volunteer rescuers unless:
A. The assistance or care is provided at the scene of the accident.
B. The action or omission is not one of gross negligence.
C. The assistance or care is provided without fee.
D. The rescuer exhibits reckless disregard of the injured person.

2. You may be liable under the Doctrine of Abandonment if you:


A. Leave a patient unattended to respond to another incident.
B. Protect yourself from life-threatening injury by leaving a dangerous accident scene.
C. Provide all the care that is required to be given.
D. Continue care until the patient is safely transported to the next tier in the EMS.

3. As a rescuer, you may have a duty to act unless:


A. You are in uniform but off shift.
B. The accident site is not in your area of responsibility.
C. The patient's family does not want you to help.
D. Doing so will place yourself or others at risk.

4. Which of the following is an ethical principle that an OECT should follow?


A. Promote the economic success of your local ski patrol
B. Avoid the appearance of negligent behavior
C. Protect others and never let them refuse care
D. Do not provide an intervention that could cause further harm

5. Which of the following is considered an acceptable form of patient consent?


A. Relative consent
B. Comparative consent
C. Absolute consent
D. Informed consent


6. Which of the following regarding standard of training and standard of care is most correct?
A. If the standard of care is to perform an intervention in which you are not trained, you must do it anyway
because of the Doctrine of Public Reliance.
B. If you are trained in a procedure, the standard of care is that you must perform it when expected to do so.
C. They are the same thing.
D. Standard of care is commonly influenced by local protocols, EMS restrictions, geography, and the
availability of higher levels of care.


CHAPTER 1
Answer Key: Chapter 1 - Quiz
1=D, 2=A, 3=D, 4=D, 5=D, 6=D

TEST

1. You arrive at the scene of an accident where a 22-year-old female skier has hit a tree and has a large bleeding
cut on her head. When you ask her if she needs help, she refuses care and says she just needs to sit for a while.
Noticing the amount of bleeding, you calmly express your concern for her, but she continues to say shedoesn't
want help. You hesitate to touch her because you are concerned that if you do, she could accuse you of:
A. Duty to act
B. Assault
C. Battery
D. Breach of duty


2. The 10th Mountain Division was founded:


A. by members of the 87th Mountain Infantry Regiment, many of whom were skiers.
B. through the Norwegian Army, which had a cold-weather unit during World War I.
C. through a request from the U.S. president to Minnie Dole.
D. after Minnie Dole convinced the U.S. Army that a winter warfare unit would be valuable.


3. Which of the following statements about OEC 5th Edition is False?


A. It is to be used by instructors to teach students the basic concepts of care.
B. It supersedes local medical direction and management directives.
C. It contains the most recent information about prehospital care at the time of its publication.
D. It is a resource text for the NSP OEC course.

4. The type of consent that is based on an appreciation and understanding of the facts, implications, and possible
future consequences of an action is known as:
A. Implied consent
B. Educated consent
C. Informed consent
D. Expressed consent

5. Dr. Warren Bowman is considered the father of Outdoor Emergency Care because he:
A. was the first program director for the National Ski Patrol.
B. co-authored the first ski patrol manual with the American Red Cross.
C. created the concept of "intermediate" medical care for ski patrollers.
D. founded the National Ski Patrol as a separate entity.


6. Which of the following is not considered a form of patient consent?


A. Informed consent
B. Minor consent
C. Absolute consent
D. Implied consent


CHAPTER 1
7. Which of the following descriptions is the best example of expressed consent?
A. A 52-year-old skier who comes to the aid room and asks for help
B. A 19-year-old male who appears to be intoxicated and lets you examine him
C. A 37-year-old man who is found unresponsive
D. A man who asks you to help his unresponsive 29-year-old diabetic wife

8. In order to maintain OEC certification, you must:


A. Complete a refresher course that covers the key objectives of the entire curriculum every year.
B. Demonstrate CPR and AED skills every two years.
C. Complete a refresher course that covers the key objectives of the entire curriculum every three years.
D. Complete a refresher course that covers one-third of the curriculum every year.

9. Although the National Ski Patrol was originally a subcommittee of the National Ski Association, the NSP
became an independent organization in:
A. 1980
B. 1975
C. 1962
D. 1953


10. A situation in which an OEC Technician leaves a patient before the patient's care is transferred to another
qualified individual is considered:
A. Assault
B. Breach of duty
C. Abandonment
D. Negligence

11. What type of consent would apply for an unresponsive 28-year-old male victim of a motorcycle collision?
A. Assumed consent
B. Informed consent
C. Implied consent
D. Expressed consent

12. What type of consent would apply for an unresponsive 28-year-old male victim of a motorcycle collision?
A. Implied consent
B. Expressed consent
C. Informed consent
D. Assumed consent


13. The type of consent that is based on an appreciation and understanding of the facts, implications, and possible
future consequences of an action is known as:
A. expressed consent.
B. informed consent.
C. educated consent.
D. implied consent.

14. Although regulations may differ from state to state, the general laws that may protect OEC Technicians from
liability while providing emergency care to a patient are known as:
A. Emergency rescue protection laws

CHAPTER 1
B. Good Samaritan laws
C. Volunteer rescue liability laws
D. First rescuer liability laws

15. Upon finding an unconscious patient at the scene of a bad accident, your next step should be to:
A. Delay your examination until you can locate a spouse or a friend of the patient.
B. Begin your assessment but not provide any treatment until another patroller arrives.
C. Delay your examination and ask other patrollers to respond as witnesses.
D. Assume "implied consent" and begin to examine and treat the patient.


16. Which of the following descriptions is the best example of expressed consent?
A. A 52-year-old skier who comes to the aid room and asks for help
B. A man who asks you to help his unresponsive 29-year-old diabetic wife
C. A 19-year-old male who appears to be intoxicated and lets you examine him
D. A 37-year-old man who is found unresponsive


17. The course called "Outdoor Emergency Care" was created by the National Ski Patrol to:
A. provide a standard of training for OEC Technicians and others involved in outdoor recreation.
B. respond to a request from Minnie Dole to develop training for patrollers working at ski resorts.
C. create a worldwide standard of care for anyone providing outdoor emergency care.
D. create a standard of care for ski patrollers and others involved in outdoor emergency care.

18. Although regulations may differ from state to state, the general laws that may protect OEC Technicians from
liability while providing emergency care to a patient are known as:
A. volunteer rescue liability laws.
B. emergency rescue protection laws.
C. Good Samaritan laws.
D. first rescuer liability laws.

19. Good Samaritan laws generally:


A. Apply to people who do not receive compensation for helping people.
B. Protect a ski patroller from being sued in every circumstance.
C. Are the same in every state.
D. Provide enough protection so that you need not to know whether your mountain has insurance.

20. The creed of the National Ski Patrol is:


A. "Service and Safety.”
B. "Snowsports and Safety.”
C. "Skiing and Safety.”
D. "First Aid and Service."


21. While in the aid room treating a patient with a fractured wrist, the patient tells you that he was skiing on a
slope that was too difficult for him, and that he started going too fast and got out of control. You recognize that
this information must be documented on the accident report because it provides evidence of:
A. Assumption of risk.
B. Gross negligence.
C. Willful negligence.
D. Contributory negligence.

CHAPTER 1
22. A situation in which an OEC Technician leaves a patient before the patient's care is transferred to another
qualified individual is considered:
A. Abandonment.
B. breach of duty.
C. Assault.
D. negligence.

23. While you are alone and caring for an injured skier in the aid room, a fellow patroller calls and tells you that
your own young child was just injured in the terrain park. Concerned for your child, you leave the patient's care
unfinished and run to the terrain park. Your action could be viewed as:
A. Abandonment
B. Your duty to act
C. A breach of ethics
D. Denial of care


24. The failure to exercise the care that is not up to the technician's standard of training is called:
A. Assault
B. Gross negligence
C. Standard of care breach
D. Negligence

25. The person who first organized a volunteer "ski patrol" and then was asked to organize a national patrol was:
A. Roger Langely
B. Warren Bowman
C. Minnie Dole
D. Roland Palmedo

26. You have just finished your shift, and while driving home still wearing your patrol jacket you come upon a car
collision in which people appear to be injured. You recognize that if you approach the collision wearing your
patrol jacket, members of the public may have a reasonable expectation that you will provide care. This
expectation is known as:
A. Professional ethics
B. Moral obligation
C. The absence of negligence
D. The doctrine of public reliance


27. Which of the following statements concerning "standard of training" is


true?
A. "Standard of training" includes your state's laws and procedures.
B. "Standard of training" includes the mountain area's protocols.
C. Acting within the "standard of training" is fundamental to reducing an OEC Technician's legal risk.
D. "Standard of training" is the same thing as "standard of care."

28. The first emergency responder textbook created exclusively for ski patrollers was titled:
A. Winter Emergency Care
B. Safety on the Snow
C. Outdoor Emergency Care

CHAPTER 1
D. Ski Safety and First Aid


29. You are skiing down your favorite slope and notice a small group gathered around a middle-aged man sitting
on the ground. You note that the boarder is holding his wrist and appears in pain. He agrees to have you examine
him but then refuses any more help. You explain the possible consequences of not splinting the wrist and of the
boarder trying to get down the mountain without help. This explanation of the risk of refusing care is provided in
order to:
A. diminish your risk of being accused of abandonment.
B. reduce your risk of an accusation of battery.
C. demonstrate that you know how to treat a wrist injury.
D. help the patient understand why you are about to put the splint on anyway.

30. The 5th Edition of Outdoor Emergency


Care:
A. Can only be taught in the winter so that students can practice the skills taught in the course.
B. Must be learned in its entirety before taking the OEC Technician evaluation.
C. Exceeds the knowledge and skills identified by the U.S. Department of Transportation for Emergency
Medical Responder.
D. Contains the baseline knowledge and skills identified by the U.S. Department of Transportation curriculum
for EMTs in all settings.

31. In order to become an "on the hill" Alpine patroller, you need to do all of the following
except:
A. Complete the OEC course.
B. Successfully complete a three-month candidacy program at your mountain.
C. Successfully complete a written exam and a practical exam.
D. Demonstrate a level of skiing/boarding and toboggan handling proficiency.

32. You are skiing down your favorite slope and notice a small group gathered around a middle-aged man sitting
on the ground. You note that the boarder is holding his wrist and appears in pain. He agrees to have you examine
him but then refuses any more help. You explain the possible consequences of not splinting the wrist and of the
boarder trying to get down the mountain without help. This explanation of the risk of refusing care is provided in
order to:
A. Diminish your risk of being accused of abandonment.
B. Help the patient understand why you are about to put the splint on anyway.
C. Reduce your risk of an accusation of battery.
D. Demonstrate that you know how to treat a wrist injury.


Answer Key: Chapter 1 - Test


1=C, 2=D, 3=B, 4=C, 5=C, 6=C, 7=A, 8=D, 9=D, 10=C, 11=C, 12=A, 13=B, 14=B, 15=D, 16=A, 17=A,
18=C, 19=A, 20=A, 21=D, 22=A, 23=A, 24=D, 25=C, 26=D, 27=C, 28=A, 29=A, 30=C, 31=B, 32=A

CHAPTER 1
CHAPTER 2
“Introduction to Emergency Care Systems"

QUIZ

1. An OECT participates in a quality-improvement process by all of the following EXCEPT:


A. Assuming responsibility for the actions of one's partner.
B. Writing accurate and complete patient care reports.
C. Maintenance of equipment used in patient care.
D. Critiquing your own performance after each call.


2. Which of the following is NOT a characteristic of an effective emergency care system?


A. Public education
B. Medical direction
C. Public access
D. Public acceptance


3. Protocols are best described as:


A. Ongoing reviews of treatment outcomes.
B. A summary of training methods.
C. Guidelines developed at the local level by emergency personnel familiar with area's medical needs,
available resources, system capabilities, and local standard of care.
D. The basis of patient care documentation.


4. Transmissions between low-power handheld radios can be adversely affected by all of the following EXCEPT
the:
A. Terrain.
B. Weather.
C. Time of day.
D. Buildings.


5. Commonly used radio communication terms include all of the following EXCEPT:
A. Indeterminate.
B. Negative.
C. Affirmative.
D. Stand by.


6. Continuity of care is best described as:


A. A process used to oversee the provision of advanced care procedures or medications.
B. An ongoing process that is designed to ensure that products and services meet or exceed expectations.
C. The seamless delivery of high-quality emergency medical care as the patient transitions from his or her
initial contact with a First Responder through definitive treatment.

CHAPTER 2
D. Identifying examples of exceptional care or showing ways to improve care the next time.


Answer Key: Chapter 2 - Quiz


1=A, 2=D, 3=C, 4=C, 5=A, 6=C

TEST

1. Most emergency care systems deploy personnel to emergencies using a stratified approach based on the
patient's condition or anticipated needs. Which of the following lists presents the normal order of deployment of
responders?
A. EMT, AEMT, EMR, Paramedic
B. EMR, EMT, AEMT, Paramedic
C. AEMT, Paramedic, EMT, EMR
D. EMR, AEMT, Paramedic, EMT


2. Which of the following is the umbrella under which every facet of health care is sheltered?
A. The emergency care system
B. Community wellness programs
C. Research
D. Public health


3. An OEC Technician meets or exceeds the NHTSA national training requirements of:
A. A Paramedic
B. An advanced Emergency Medical Technician
C. An Emergency Medical Technician
D. An Emergency Medical Responder


4. Which of the following goals best describes a key goal of an emergency care system?
A. To have trained personnel respond and provide emergency care on the scene
B. To identify and modify illness and injury risks
C. To have trained personnel understand the limitations of their training and to "do no harm”
D. To have trained personnel knowledgeable in all aspects of emergency medical care

5. Which of the following choices is not an example of direct medical oversight?


A. Telephone communication
B. Physical presence of a physician
C. Written protocols
D. Video conferencing

6. Which of the following characteristics is not a characteristic of indirect medical control?


A. Communication by radio
B. Use for expediting patient care
C. Common use for "pre-authorizing" routine and life-saving procedures
D. Provision through the use of protocols


7. Which of the following is not an attribute of an emergency care system?

CHAPTER 2
A. Public access
B. Human resources
C. Equipment
D. Research

8. The seamless delivery of high-quality emergency medical care as patient transitions from initial contact with an
EMT through definitive treatment best describes:
A. Medical direction
B. Online medical control
C. Integration of health services
D. Continuity of care


9. Which of the following is most effective for reducing the incidence of injury or illness in skiing or other
outdoor activities?
A. Clinical care
B. Emergency personnel
C. Prevention
D. Public access

10. What time is 6:43 a.m. in military time?


A. 2343
B. 0643
C. 1843
D. 1243

11. Which of the following tips is not a part of common radio etiquette?
A. Include detailed information in a single transmission that takes no longer than 60 seconds.
B. Speak directly into the microphone.
C. Hold the transmit key in for 1–2 seconds before speaking.
D. Do not use the patient's name.

12. Which of the following tips is not a part of common radio etiquette?
A. Do not use the patient's name.
B. Hold the transmit key in for 1-2 seconds before speaking.
C. Speak directly into the microphone.
D. Include detailed information in a single transmission that takes no longer than 60 seconds.


13. Written instructions that indicate what should be done in a given situation are best described as:
A. Protocols.
B. Regulations.
C. direct medical oversight.
D. indirect medical oversight.

14. Critical care providers who bring advanced life support equipment, therapies, and interventions to the scene
are typically deployed in what tier?
A. Tier 3
B. Tier 4
C. Tier 2

CHAPTER 2
D. Tier 1

15. Which of the following levels of training emphasizes the provision of immediate life-saving treatment and
stabilization to critically ill or injured patients while waiting for additional emergency response?
A. Advanced Emergency Medical Technician
B. Emergency Medical Responder
C. Emergency Medical Technician
D. Critical care responder

16. Which of the following is not a role of OEC Technicians in the quality improvement process?
A. Participating in case reviews
B. Blaming their partner for a mistake in care
C. Participating in training
D. Being receptive to on-scene instructions given by another rescuer

17. When speaking to another person over the airwaves, the use of radio codes:
A. is discouraged because the codes are not universal to all public safety agencies.
B. is required when everyone on your patrol is trained.
C. decreases confusion in communication.
D. is preferred so that the public cannot understand your transmission.

18. Which of the following attributes is not one of the 14 attributes of an effective emergency care system?
A. System finance
B. Integration of health services
C. Public access
D. Public acceptance

19. Guidelines developed at the local level by emergency personnel familiar with an area's medical needs,
available resources, system capabilities, and local standard of care are:
A. Advanced procedures
B. Policies
C. Medical attributes
D. Protocols

20. What is 3:35 p.m. in military time?


A. 1335
B. 1535
C. 1735
D. 0335

21. What do you say using the NATO Phonetic Alphabet to ensure that the letters SMD are understood?
A. "Sierra Mike Delta”
B. "Saltine Mary Diamond”
C. "Sierra Mary Diamond”
D. "Saltine Mike Delta"

22. Many emergency care systems deploy personnel to emergencies using a tiered approach. OEC Technicians are
most likely deployed in which tier?
A. Tier 3

CHAPTER 2
B. Tier 4
C. Tier 2
D. Tier 1

23. The reason it is important that OEC Technicians participate in quality improvement programs in services in
which they work or volunteer is:
A. To ensure that individuals making false calls for help are prosecuted.
B. To guarantee that the public receives quality emergency medical care.
C. To identify problem employees and create a corrective action plan for them.
D. To identify problems in care delivery and develop a plan to prevent their recurrence

24. Which of the following is not one of the four nationally recognized prehospital emergency care provider
levels?
A. Emergency Medical Technician
B. Outdoor Emergency Care Technician
C. Advanced Emergency Medical Technician
D. Emergency Medical Responder

25. A network of specially trained personnel, equipment, facilities, and other resources that respond to medical
emergencies -- regardless of of cause, location, or the patient's ability to pay -- best
describes:
A. A community wellness program
B. A public health system
C. A health care system
D. An emergency care system


26. Written instructions that indicate what should be done in a given situation are best described as:
A. Regulations
B. Direct medical oversight
C. Indirect medical oversight
D. Protocols


27. Which of the following methods is the best method for ensuring that the treatment rendered by OEC
Technicians meets or exceeds customer expectations and national education standards for emergency medical
personnel?
A. Prevention training
B. Quality improvement
C. Public education
D. Continuity of care


Answer Key: Chapter 2 - Test


1=B, 2=D, 3=D, 4=B, 5=C, 6=A, 7=C, 8=D, 9=C, 10=B, 11=A, 12=D, 13=A, 14=B, 15=B, 16=B, 17=A,
18=D, 19=D, 20=B, 21=A, 22=D, 23=D, 24=B, 25=D, 26=D, 27=B

CHAPTER 2
CHAPTER 2
CHAPTER 3
“Rescue Basics”

QUIZ

1. Standard Precautions to prevent the spread of infectious diseases include:


A. Determining the patient's immunization status.
B. Administering the hepatitis B vaccine to all First Responders.
C. Donning gloves prior to patient contact
D. Checking for open wounds.

2. Scene safety principles include:


A. Summon law enforcement for crowd control.
B. Assess the adjacent terrain for additional hazards.
C. Immediately extricate the patient from the accident site.
D. Contact risk management prior to entering the scene.

3. Mechanisms to increase body core temperature include all of the following EXCEPT:
A. Constriction of skin blood vessels.
B. Increased sweat production.
C. Increased metabolic rate.
D. Increased epinephrine release.

4. If you suspect an accident scene is complicated by electrical hazard, you should:


A. Summon law enforcement for crowd control.
B. Immediately remove the patient from the scene.
C. Recognize that patient care may need to be delayed until the scene is secured by trained personnel.
D. Find a wooden pole to push the wire away.

5. Heat transfer from warmer to colder objects occurs by all of the following EXCEPT:
A. Evaporation.
B. Conduction.
C. Alteration.
D. Convection.


Answer Key: Chapter 3 - Quiz


1=D, 2=B, 3=B, 4=C, 5=C

TEST

1. Metabolism:
A. Is the same as the digestive process.

CHAPTER 3
B. Involves only glucose and enzymes.
C. Is defined as the chemical processes occurring within a living cell or organism
D. Always involves insulin from the pancreas.

2. During the "fight or flight" stress response, the release of epinephrine results in:
A. Blood vessels dilating leading to decreased blood pressure
B. Decreased blood flow to the muscles of the arms and legs
C. The respiratory rate and depth decreasing
D. The heart rate increasing

3. After scene safety is established in a trauma situation, what is the next important thing to determine?
A. MOI
B. SAMPLE
C. ABCDs
D. NOI

4. The practice of protecting yourself from exposure to body fluids based on the assumption that all patients are
potentially infectious is referred to as:
A. Standard precautions.
B. Body fluid precautions.
C. Personal protective equipment.
D. Exposure control.

5. When the body's core temperature drops, the body:


A. Constricts blood vessels to increase heat loss
B. Sends a signal to the heart to speed up.
C. Starts shivering to increase heat production
D. Increases breathing to raise the oxygen level.

6. Some vector-borne illnesses are transmitted to humans via:


A. Deer ticks
B. Contact with human feces
C. Sneezing and coughing
D. Eating partially cooked foods

7. Some hazardous products can cause health problems if ingested or inhaled. Long-term inhalation of asbestos
can cause:
A. kidney tumors.
B. colon cancer.
C. lung cancer.
D. pancreatic cancer.

8. Which of the following SPF ratings would provide the greatest level of protection?
A. 30
B. 50
C. 15
D. 6

CHAPTER 3
9. Which one of the following techniques is the single best technique that OEC Technicians can use to prevent the
spread of infection?
A. Wearing gloves
B. Getting immunizations
C. Disinfecting their equipment
D. Washing their hands

10. The best strategy to maintain warmth in the cold winter months is to wear:
A. Clothing in one layer; multi; le-purpose outerwear.
B. Clothing in three layers; a base to retain heat, a middle insulating layer and a water and wind-repelling
outer layer.
C. Clothing primarily made of cotton
D. Clothing in two layers; a base to retain heat and a water and wind-repelling outer layer.

11. Who is in charge of a crime scene at your ski area?


A. The responding law enforcement officer
B. The area's operation manager
C. The area's patrol director
D. The area's risk manager

12. Which of the following actions is not a method for ensuring scene safety?
A. Having a bystander stand uphill of the incident site
B. Calling for risk management personnel
C. Placing skis in an "X" uphill of the scene
D. Conducting a scene size-up

13. The best natural material used in winter clothing is:


A. Polyester.
B. Silk.
C. Wool.
D. Cotton.

14. The structure that regulates body temperature is the:


A. Brain stem
B. Hypothalamus
C. Cerebellum
D. Medulla

15. Cross-contamination occurs when:


A. You touch multiple patients without changing your gloves.
B. Patients do not wash their hands after using a restroom.
C. Patients inject themselves with a needle they have used before.
D. You touch a second patient after changing your gloves.

16. Wearing clean gloves, an impervious gown, and goggles while cleaning a long-board contaminated with blood
helps protect the OEC technician from which form of potential infectious disease transmission?
A. Ingestion
B. Direct contact with a human with infectious pathogens
C. Airborne

CHAPTER 3
D. Indirect contact with an object contaminated with pathogens

17. The body gets energy from all of following except:


A. Carbohydrates
B. Fats
C. Sodium
D. Proteins

18. After scene safety is established in a trauma situation, what is the next important thing to determine?
A. NOI
B. ABCDs
C. SAMPLE
D. MOI

19. A communicable disease is:


A. A disease that is caused and spread by coughing only.
B. A disease that can be spread from one person to another.
C. Any of a number of diseases that is deadly once contracted.
D. A disease for which no immunization or treatment exists.

20. To prevent self-contamination, you must remove your gloves properly and dispose of them with contaminated
waste.  Properly removing your gloves includes all of the following "dirty to dirty and clean to clean" principles
except:
A. As a last step, placing your contaminated gloves in your medical kit or jacket pocket
B. The contaminated external glove of one hand should only touch the contaminated external glove of the
other hand when removing
C. The clean, ungloved hand should only touch the clean inside surface of the contaminated gloved hand
when removing
D. When finished, both gloves should be turned inside out, with the second glove serving as the container for
the first glove


21. A disinfection involves the use of:


A. 1 quart of a solution with a pH of 7.
B. an EPA-registered sterilizing chemical or a 1:100 solution of household bleach.
C. a bleach solution consisting of ¼ cup of bleach to 1 gallon of water.
D. a bleach solution consisting of 2.5 tbs. of bleach to 1 gallon of water.

22. Which of the following tasks is the first component of a scene size-up?
A. Determine the MOI.
B. Scene safety
C. Identify the number of patients involved.
D. Determine whether additional resources are needed.

23. The direction of heat transfer is always:


A. From a cooler object to a warmer object.
B. From a human object to a non-human object.
C. From a warmer object to a cooler object.
D. From a non-human object to a human object

CHAPTER 3
24. When caring for a patient who was stabbed with a knife in the ski area's bar (a crime scene situation), it is
important for the OEC technician to do all of the following to support "chain of custody" except:
A. Notify law enforcement immediately
B. Do not remove, move or otherwise disturb anything in the environment, except as is absolutely necessary
to provide critical care to patient
C. Let law enforcement personnel handle any materials that could be used as evidence
D. Wrap the knife found on the floor next to the patient in plastic and hand it to the EMTs so they can show
the length of the knife to the Emergency Department staff

25. Cross contamination occurs when:


A. you touch multiple patients without changing your gloves.
B. patients do not wash their hands after using a restroom.
C. you touch a second patient after changing your gloves.
D. patients inject themselves with a needle they have used before.

26. Non-disposable PPE (personal protective equipment) and reusable equipment should be disinfected per:
A. The ski area associations protocol
B. The NSP protocol.
C. The manufacturer's recommendations.
D. The medical control protocol.

27. Who is in charge of a crime scene that occurs at your ski area?
A. The area's risk manager
B. The area's patrol director
C. The responding law enforcement officer
D. The area's operation manager

28. Conduction is the transfer of heat:


A. From a warm object to a cold object through direct contact.
B. Through respirations.
C. Through a gas or liquid moving past the surface of the skin
D. Through radiation.

29. There is generally at least a two-week period before one experiences the early flu-like symptoms of rabies.
Effective medical treatment for rabies should occur within:
A. 1 year.
B. 10 days.
C. 1 month.
D. 24 hours.

Answer Key: Chapter 3 - Test


1=C, 2=D, 3=A, 4=A, 5=C, 6=A, 7=C, 8=B, 9=D, 10=B, 11=A, 12=B, 13=C, 14=B, 15=A, 16=D, 17=C,
18=D, 19=B, 20=A, 21=B, 22=B, 23=C, 24=D, 25=A, 26=C, 27=C, 28=A, 29=D

CHAPTER 3
CHAPTER 4
“Incident Command and Triage”

QUIZ

1. The Incident Command System is a method for managing which of the following?
A. Regional disasters only
B. Earthquakes that destroy regional hospitals
C. Any incident regardless of cause, size, scope, or complexity
D. Terrorist attacks involving a major city

2. Which of the following is NOT one of the major components of the National Incident Management System
(NIMS)?
A. Rescuer performance review
B. Span of control
C. Resource management
D. Preparedness

3. In NIMS, an incident is defined as:


A. A declaration of a state of emergency by the local government.
B. Any out-of-the-ordinary activity that necessitates a response.
C. An event that overwhelms local rescue agencies.
D. Any call to 9-1-1 asking for assistance.

4. Functional areas within the Incident Command System include all of the following EXCEPT:
A. Planning.
B. Logistics.
C. Conflict resolution.
D. Operations.

5. Triage is:
A. Stopped when advanced medical teams are on site.
B. Performed by the incident commander.
C. A dynamic process requiring ongoing reassessment.
D. Performed once per incident.

6. In the United States, which is the most commonly used triage system?
A. START (Simple Triage and Rapid Treatment)
B. MASS (Move, Assess, Sort, Send)
C. SALT (Sort, Assess, Life-saving Triage)
D. CIT (Continuous Integrated Triage)

Answer Key: Chapter 4 - Quiz


1=C, 2=A, 3=B, 4=C, 5=C, 6=A

CHAPTER 4
TEST

1. Which of the following tasks is not a responsibility of the Incident Commander?


A. Providing overall management of the incident
B. Determining the incident strategy
C. Triaging the injured patients
D. Establishing the Incident Command Post

2. The primary function of the Incident Commander is to:


A. Ensure that radio communications are established.
B. Assess the nature and scope of the incident.
C. Obtain supplies needed to care for all patients.
D. Provide overall leadership and direction.

3. The most widely used triage categorization system in the world is "ID-ME." This acronym represents the four
specific triage categories used by NATO forces, National Disaster Life Support, public safety agencies, and search
and rescue groups. The ID-ME triage categories are:
A. Immediate, delayed, minimal, and exceptional.
B. Immediate, delayed, minimal, and expectant.
C. Immediate, detain, minimal, and expectant.
D. Immediate, delayed, maximum, and exceptional.

4. In the acronym "START," the letter "S" stands for:


A. safe.
B. simple.
C. selective.
D. sophisticated.

5. The Incident Commander has assigned you to be the Operations Section Chief. In this role you are responsible
for all of the following except:
A. Determining incident objectives and strategy.
B. Participating in the planning process.
C. Reducing immediate hazards.
D. Restoring the scene to normalcy.

6. Which of the following tasks is not a responsibility of the Incident Commander?


A. Triaging the injured patients
B. Providing overall management of the incident
C. Establishing the Incident Command Post
D. Determining incident objectives and strategies

CHAPTER 4
7. A formal, organized method for managing an incident, regardless of its cause, size, scope, or complexity, is
called:
A. An emergency strike team.
B. A multi-agency coordination system.
C. An incident command system.
D. A federal incident management system.

8. All of the following are functional areas of the Incident Command Structure except:
A. Planning Section
B. Finance/ Administration Section
C. Transportation Section
D. Operations Section

9. A formal, organized method for managing an incident, regardless of its cause, size, scope, or complexity, is
called:
A. an emergency strike team.
B. a multi-agency coordination system.
C. a federal incident management system.
D. an incident command system.

10. Which one of the following statements about the Incident Command System is true?
A. Each EMS agency can use its own codes and terms.
B. ICS operates with standardized language to avoid confusion.
C. There is a separate commander for EMS, fire, and law enforcement personnel.
D. Each individual responder is accountable to two supervisors.

11. The person responsible for maintaining equipment and ensuring that facilities meet specified needs is the:
A. Operations Chief.
B. Planning Chief.
C. Logistics Chief.
D. Maintenance Chief.

12. In the acronym "START," the letter "S" stands for:


A. Simple.
B. Selective.
C. Safe.
D. Sophisticated.

13. The most widely used triage categorization system in the world is "ID-ME." This acronym represents the four
specific triage categories used by NATO forces, National Disaster Life Support, public safety agencies, and search
and rescue groups. The ID-ME triage categories are:
A. immediate, delayed, maximum, and exceptional.
B. immediate, detain, minimal, and expectant.
C. immediate, delayed, minimal, and expectant.
D. immediate, delayed, minimal, and exceptional.

14. Which one of the following triage tag indicates the lowest priority for care, "Expectant", for a patient at a
multiple-casualty incident?
A. A green tag

CHAPTER 4
B. A yellow tag
C. A red tag
D. A black tag

15. When handling a mass-casualty incident, you use triage to:


A. do the most good for the most people.
B. minimize the cost of responding to the incident.
C. do the most good for a limited number of people.
D. work on all injured parties.

16. When you are placed in charge of the treatment unit at the scene of a multiple-casualty incident, which of the
following four patients will you treat first?
A. A 21-year-old male who goes into cardiac arrest
B. A 13-year-old boy who walked to the treatment area complaining of a severe headache
C. An elderly female patient with a femur fracture and a yellow tag on her wrist
D. A confused 69-year-old male with a respiratory rate of 40 and a rapid and weak radial pulse

17. All of the following are functional areas of the Incident Command Structure except:
A. Planning
B. Operations
C. Transportation
D. Finance

18. While in charge of triage at a scene involving 30 patients exposed to carbon monoxide, you find a young
female who is not breathing. After you open her airway and find that breathing does not return, you should:
A. Apply oxygen and check her mental status.
B. Provide her 15 seconds of CPR and recheck for a radial pulse.
C. Provide her 15 seconds of assisted ventilation.
D. Place a black tag on her and move on to the next patient.

19. In the Incident Command System, the section responsible for maintaining injury, death, and damage
documentation as well as maintaining reimbursement records is the:
A. Operations section
B. Finance/administration section
C. Planning section
D. Logistics section

20. In the Incident Command System, the operations section is responsible for:
A. collecting, assessing, and distributing incident-related data.
B. providing financial management.
C. providing support for all functional areas.
D. executing the strategy of the Incident Action Plan.

21. Of the five functional areas within the Incident Command Structure (ICS) structure, the first to be established
should be:
A. Planning section.
B. Incident Command.
C. Logistics section.
D. Operations Section.

22. There is a fight involving approximately 20 bar patrons. Weapons were used, and there are varying degrees
and types of injuries. The first person on scene has started assessing a person lying on the floor with blood

CHAPTER 4
covering his shirt. Quick assessment reveals him to be breathing at 24 times per minute. Which of the following
actions should be done next?
A. Attend to the next patient.
B. Place a yellow tag on his wrist.
C. Assist ventilation with a BVM.
D. Check for a radial pulse.

23. In the Incident Command System, the operations section is responsible for:
A. Providing support for all functional areas.
B. Providing financial management.
C. Executing the strategy of the Incident Action Plan.
D. Collecting, assessing, and distributing incident-related data.

24. You are at an outside skiing exhibition when a spectator bleacher collapses. Several injured people are walking
around with various injuries. Which of the following instructions is most appropriate for these ambulatory
patients?
A. "Sit down here so I can do a quick assessment on you.”
B. "Put this yellow tag on your wrist and go to the ticket booth at the bottom of the slope.”
C. "Go and wait by the ticket booth at the bottom of the slope.”
D. "Leave the ski area and go to the hospital. Others are hurt more seriously than you.”

25. In the Incident Command System, the section responsible for maintaining injury, death, and damage
documentation as well as maintaining reimbursement records is the:
A. planning section.
B. operations section.
C. finance/administration section.
D. logistics section.

26. While responding to an MCI drill involving an explosion, you recognize that the dozen ambulances sitting in a
parking lot three blocks away from the incident constitute the:
A. Treatment unit
B. Triage area
C. Transport unit
D. Staging unit

27. You are triaging using the Simple Triage and Rapid Transport (START) system and you find a patient who is
breathing 18 times per minute and has a radial pulse. Which one of the following actions should you take next?
A. Check to see if the patient can follow simple directions.
B. Check the patient's blood pressure.
C. Place a yellow tag on the patient.
D. Move the patient to the treatment area.

28. In the Incident Command System, which of the following sections is responsible for maintaining all incident
reports except for financials?
A. Planning section
B. Logistics section
C. Records section
D. Operations section

29. When properly trained in the START system, an OEC Technician should be able to complete a patient
assessment in less than:
A. 45 seconds.

CHAPTER 4
B. 90 seconds.
C. 30 seconds.
D. 60 seconds.

30. Which of the following is not a key characteristic of facilities utilized in an Incident Command System?
A. They must be able to grow in size as needed.
B. They must be located in a secure, safe area.
C. They should be situated in public view.
D. They must be accessible to incident personnel.

31. At a multiple-casualty incident, you are brought a patient with a red tag tied to his wrist. You should
recognize which one of the following?
A. Immediate care and transport of the patient are necessary.
B. The patient is deceased.
C. The patient has a minor wrist injury.
D. Delayed care and transport of the patient are permissible.

32. You are transporting a patient at a multiple-casualty incident. The patient has a yellow triage tag. This tag
means:
A. The patient has a head injury with altered mental status.
B. The patient has no injuries.
C. Treatment of the patient could be delayed for up to 4 hours.
D. The patient should be with the "walking wounded.”

33. In using the Simple Triage and Rapid Transport (START) system of triage, which one of the following patients
should have a yellow tag applied before being moved to the treatment area?
A. A male with a respiratory rate of 40 and a palpable radial pulse who squeezes your fingers when instructed
to do so
B. A female with gasping respirations, no radial pulse, and unresponsive mental status
C. A male with a respiratory rate of 8, no radial pulse, and unresponsive mental status.
D. A female with a respiratory rate of 22 and a palpable radial pulse who squeezes your fingers when
instructed to do so.

34. At the scene of a multiple-casualty incident, you are presented with a patient who is not breathing. According
to the Simple Triage and Rapid Transport (START) system, which of the following actions should you take next?
A. Start assisting ventilation with a BVM.
B. Place a red tag on the patient.
C. Place a black tag on the patient.
D. Open the airway.

35. When properly trained in the START system, an OEC Technician should be able to complete a patient
assessment in no more than:
A. 90 seconds.
B. 60 seconds.
C. 45 seconds.
D. 30 seconds.

36. A teen involved in a multiple-casualty incident is found lying on the ground with an obvious deformity to his
left thigh. Using the START system for triage, you note that he is not breathing. Which one of the following
should you do next?
A. Provide the patient 30 seconds of assisted ventilation.
B. Open the patient's airway, next check for a radial pulse.
C. Give the patient a red tag; next move him to the treatment area.

CHAPTER 4
D. Tag the patient as black, then move on to the next patient.

Answer Key: Chapter 4 - Test


1=C, 2=D, 3=B, 4=B, 5=A, 6=A, 7=C, 8=C, 9=D, 10=B, 11=C, 12=A, 13=C, 14=D, 15=A, 16=D, 17=C,
18=D, 19=B, 20=D, 21=B, 22=D, 23=C, 24=C, 25=C, 26=D, 27=A, 28=A, 29=C, 30=C, 31=A, 32=C,
33=D, 34=D, 35=D, 36=B

CHAPTER 4
CHAPTER 5
“Lifts, Loads and Transportation”

QUIZ

1. An 8-months-pregnant patient has a twisted ankle and is lying on her back, complaining of being "light-
headed." She should be positioned:
A. On her left side.
B. On her right side.
C. Head down.
D. Head up.

2. Exceptions to the "transport with the injured part uphill" principle include all of the following EXCEPT:
A. Significant head injury.
B. Shoulder dislocation.
C. Respiratory distress.
D. Shock.

3. The risk of injuring your back during a lift or load can be reduced by all of the following EXCEPT:
A. Maintaining a healthy weight.
B. Immobilizing a patient to a long spine board.
C. Exercising regularly.
D. Using good body mechanics.

4. Urgent moves are made by:


A. Sliding the patient sideways.
B. Pulling on the most secure part of the patient's clothing.
C. Pulling on the extremity closest to the rescuer.
D. Preserving the long axis of the spine.

5. Ideally, the decision to use a helicopter to evacuate a critically injured patient would include all of the following
EXCEPT the:
A. Exposure of a ground evacuation team to hazardous terrain.
B. Cost.
C. Altitude at the accident site.
D. Time of day.

Answer Key: Chapter 5 - Quiz


1=A, 2=B, 3=B, 4=D, 5=B

CHAPTER 5
TEST

1. To use an urgent move called the shoulder drag, you should:


A. stand behind the patient and grab the patient's belt while their shoulders are against your chest.
B. grab the patient's clothing at the shoulders, support the head, and then bend your knees and drag the
patient.
C. stand upright behind the patient, bend over, and then grab the patient under the armpits and drag.
D. grab one arm at the shoulder and drag the patient to a safe position.

2. The uphill positioning principle has exceptions. For patients with breathing difficulty the patient should be
placed either seated or uphill depending on other injuries. The uphill concept allows easier breathing because:
A. It calms patients by enabling them to see where they are going during transport.
B. The ribs have greater flexibility of movement in the uphill position.
C. Air flows more easily through a patient's nose compared to a head-downhill position.
D. Of less pressure from the abdominal contents pushing upwards on the thorax.

3. Which of the following instructions for approaching a running rescue helicopter is most dangerous?
A. Remove any loose clothing.
B. Remain in sight of the pilot at all times.
C. Remain low.
D. Remain in an upright position and move slowly.

4. For a patient with a suspected spinal injury, the urgent move most likely to cause further injury to the patient
is the:
A. Feet drag
B. Shoulder drag
C. Underarm-wrist drag
D. Blanket drag

5. The best non-urgent move for OEC Technicians to use when alone is the:
A. Fore and aft carry
B. Back carry
C. Human crutch
D. Chair carry

6. Which of the following pieces of equipment do many OEC Technicians use most often to move, lift, and carry a
patient?
A. Orthopedic stretcher
B. Portable stretcher
C. Basket stretcher
D. Long spine board (LSB)

7. The high-Fowler position, in which the patient is seated with the head at 90 degrees, is typically used for
patients with:
A. Eye injuries
B. A fractured collar bone
C. Severe respiratory problems

CHAPTER 5
D. Abdominal injuries

8. A woman who is six months pregnant and experiencing back pain should be placed on a backboard and then
placed in a toboggan in which of the following positions?
A. With her head downhill
B. With her head uphill also correct, change to Sitting up
C. Positioned on her left side
D. In a supine position

9. After a landing helicopter is safely on the ground, one may approach the aircraft only when signaled by the:
A. Senior police officer on the scene.
B. Pilot or flight crew chief.
C. Fire Department chief.
D. Landing coordinator.

10. When a person exhibits good body mechanics and lifts properly, the spine is:
A. Straight at the thoracic region and bent in the lumbar region.
B. Straight and in an anatomical position
C. Slightly bent in the upper thoracic region.
D. Bent at no more than 15 degrees.

11. An urgent move is required when:


A. A crowd of people has formed around you and the patient.
B. The rescuer and the patient must move to a safer location.
C. The patient complains of being cold on a 10-degree day.
D. A patient is in severe pain.

12. The greatest risk posed by an urgent move is:


A. delaying interventions for life-threatening conditions.
B. changing the position of the patient and causing a risk-management investigation.
C. not getting the patient's name and address.
D. incurring a lawsuit by the patient.

13. Commercially produced products are favored for draw sheet lifts because they generally have:
A. Waterproof qualities
B. Insulating qualities
C. Stitched edges with better material
D. Handles

14. To lessen one's chances of injury while lifting and moving a patient, one should:
A. Combine good lifting and moving techniques.
B. Avoid manual moves and instead use mechanical devices.
C. Use a long-axis drag to avoid lifting.
D. Roll the patient.

15. Shock patients who are not having difficulty breathing are placed head-first downhill because doing so:
A. Increases perfusion of blood to vital organs.
B. Decreases blood pressure.

CHAPTER 5
C. Increases the heart rate.
D. Increases blood pressure.

16. Of the four lift techniques OEC Technicians use, the draw sheet method:
A. Requires the fewest rescuers to use.
B. Has the lowest rate of dropped patient incidents.
C. Is the safest for the patient.
D. Has the lowest risk of back injury to a rescuer.

17. During helicopter landing or takeoff, most protocols only allow the _______ to be in the LZ.
A. OEC Technician and the patient
B. Patrol director
C. Landing coordinator
D. Risk manager

18. The first step in placing a patient in the toboggan for transport is:
A. Deciding the fastest route of transport.
B. Deciding who will be in the handles of the toboggan during transport.
C. Deciding whether or not to put the patient's injury uphill.
D. Deciding how to get the patient and equipment off the hill.

19. An appropriate landing zone (LZ) for safely landing a helicopter is an open area that is approximately:
A. 100 feet X 200 feet
B. 75 feet X 100 feet
C. 100 feet X 100 feet
D. 200 feet X 200 feet

20. In most instances, an urgent move is performed:


A. after the primary and secondary assessments have been completed.
B. only when Medical Control orders it.
C. only after additional help has arrived.
D. before any assessment has been performed.

21. When used correctly, the power grip maximizes the force of the hands, which are placed:
A. Palms down and 18 inches apart.
B. Palms up and 10 inches apart.
C. Palms up and 5 inches apart.
D. Palms down and 8 inches apart.

22. A device best used to transport a patient in a tight space is known as:
A. A short board.
B. A wheeled stretcher.
C. An evacuation chair.
D. A Kendrick extrication device.

23. When using the power grip, the hands are:


A. Placed on the side of the lifting device and held firmly.
B. Placed underneath the lifting device and held firmly.

CHAPTER 5
C. Held on the side of the device at a 45-degree angle.
D. Placed underneath the lifting device and held loosely.

24. When approaching most helicopters, whether running or not, the safest way is to do so from the:
A. Left side
B. Back
C. Right side
D. Front

25. In many instances, an urgent move is performed:


A. Only after additional help has arrived.
B. After the primary and secondary assessments have been completed.
C. Before any assessment has been performed.
D. Only when Medical Control orders it.

26. The greatest risk posed by an urgent move is:


A. Incurring a lawsuit by the patient.
B. Changing the position of the patient and causing a risk-management investigation.
C. Not getting the patient's name and address.
D. Delaying interventions for life-threatening conditions.

27. Like a direct ground lift, a ________ lift is used to raise patients who are lying on their back.
A. LEAN
B. SEAN
C. BEAN
D. MEAN

28. Transporting a patient using a basket litter:


A. Requires one patroller at the front and one patroller on the tail rope.
B. Requires four to six rescuers.
C. Involves towing it behind a motorized vehicle.
D. Can be achieved using a chairlift.

29. During transport, the patient must be properly positioned for comfort and to do no further harm. The Semi-
Fowler position places the patient in a:
A. Sitting position with the head raised 45 degrees.
B. Supine position with the legs raised 15 degrees.
C. Sitting position with the head raised 90 degrees.
D. Supine position with the legs raised 45 degrees.

30. A basket stretcher is a good method for transporting a patient:


A. Who is extremely heavy?
B. With a fractured femur.
C. On an LSB in any terrain.
D. Out of the back country.

31. For a patient with a suspected spinal injury, the least desirable urgent move for OEC Technicians to use is the:

CHAPTER 5
A. blanket drag.
B. underarm-wrist drag.
C. feet drag.
D. shoulder drag.

32. Which of the following instructions is most important for combining optimal anatomic position with good
body mechanics in a power lift?
A. Straighten your legs to lift.
B. Squat down and keep your knees outward at a 45-degree angle.
C. Maintain a wide stance (at least 24 inches).
D. Keep your back muscles loose and your head bent forward.

33. The best non-urgent move for OEC Technicians to use when alone is the:
A. fore and aft carry.
B. human crutch.
C. back carry.
D. chair carry.

34. To use an urgent move called the shoulder drag, you should:
A. Grab one arm at the shoulder and drag the patient to a safe position.
B. Stand upright behind the patient, bend over, and then grab the patient under the armpits and drag.
C. Grab the patient's clothing at the shoulders, support the head, and then bend your knees and drag the
patient.
D. Stand behind the patient and grab the patient's belt while their shoulders are against your chest.

35. Extremity lifts may not be tolerated by:


A. Elderly patients and patients with respiratory disorders.
B. Obese and young patients.
C. Patients with visual impairments.
D. Patients with ankle injuries.

36. To maintain proper alignment when moving or lifting a heavy object, the weight of the object must be:
A. Offset using shoulder and chest strength.
B. Transferred to the forearms and biceps.
C. Evenly transferred to the legs.
D. Distributed to the lower back.

37. With a BEAM lift, the patient is:


A. Assisted into a chair.
B. Rolled onto a long board.
C. Lifted and carried a short distance.
D. Assisted to a standing position.

Answer Key: Chapter 5 - Test


1=B???, 2=D, 3=D, 4=A, 5=C, 6=D, 7=C, 8=C, 9=B, 10=B, 11=B, 12=A, 13=D, 14=A, 15=A, 16=D, 17=C,
18=C, 19=C, 20=D, 21=B, 22=C, 23=B, 24=D, 25=C, 26=D, 27=C, 28=B, 29=A, 30=D, 31=C, 32=A,
33=B, 34=B???, 35=A, 36=C, 37=C, ??? Online Quiz wrong, confirm with book

CHAPTER 5
CHAPTER 5
CHAPTER 6
“Human Anatomy and Physiology”

A N ATO M Y L A B E L I N G 01
Answer Key:

A. Mandible
B. Neck
C. Shoulder
D. Arm
E. Elbow
F. Forearm
G. Wrist
H. Hand
I. Cranium
J. Face
K. Thigh
L. Knee
M. Leg
N. Ankle
O. Foot

CHAPTER 6
ANATOMY L ABELING 02

Answer Key:

A. Midline
B. Proximal
C. Distal
D. Mid-clavicular
E. Medial
F. Lateral
G. Dorsal
H. Posterior (dorsal)
I. Superior
J. Mid-axillary
K. Inferior
L. Anterior (central)

CHAPTER 6
ANATOMY L ABELING 03

Answer Key:

A. Thyroid Cartilage
B. Cricoid cartilage
C. Trachea
D. Right main bronchus
E. Lungs
F. Nasopharynx
G. Oropharynx
H. Epiglottis
I. Larynx
J. Left main bronchus
K. Diaphragm
L. Bronchiole
M. Alveolus

CHAPTER 6

ANATOMY L ABELING 0 4

Answer Key:

A. Superior vena cava


B. Pulmonary artery
C. Pulmonary veins
D. Right atrium
E. Valves
F. Right ventricle
G. Inferior vena cava
H. Aorta
I. Pulmonary veins
J. Left atrium
K. Valves
L. Left ventricle

CHAPTER 6

ANATOMY L ABELING 05

Answer Key:

A. Carotid Artery
B. Pulmonary artery
C. Aorta
D. Brachial
E. Radial
F. Femoral
G. Posterior tibial
H. Dorsal pedis
I. Pulmonary vein
J. Superior vena cava
K. Inferior vena cava

CHAPTER 6
ANATOMY L ABELING 06

Answer Key:

A. Frontal lobe
B. Pituitary gland
C. Sphenoidal sinus
D. Pons
E. Meninges
F. Occipital lobe
G. Cerebellum
H. Medulla oblongata
I. Durameter
J. Spinal cord

ANATOMY L ABELING 07



Answer Key:

A. Suprasternal (jugular) notch


B. Clavicle
C. Sternum
D. Diaphragm
E. Umbilicus
F. Sternoclavicular joints
G. Pectoral Region
H. Xiphoid process
I. Costal arch (margin)
J. Pubis
K. Scapular region
L. Lumbar region
M. Iliac crest

CHAPTER 6
ANATOMY L ABELING 08

Answer Key:

A. Frontal Region
B. Orbit
C. Maxilla
D. Mandible
E. Larynx
F. Trachea
G. Suprasternal (jugular) notch
H. Parietal region
I. Temporal region
J. Occipital region
K. Mastoid process
L. Temporo-mandibular joint
M. Zygomatic region

ANATOMY L ABELING 09



Answer Key:

A. Lateral femoral condyle


B. Medial femoral condyle
C. Lateral malleolus
D. Medial malleolus
E. Calcaneus
F. Femoral head
G. Femoral neck
H. Shaft of femur
I. Patella
J. Fibula
K. Tibia

CHAPTER 6
ANATOMY L ABELIN G 10

Answer Key:

A. Humeral head
B. Radial and ulnar styloids
C. Shaft of humerus
D. Elbow
E. Acromioclavicular joint
F. Ulner shaft
G. Glenohumeral joint
H. Lateral humeral condyle
I. Medial Humeral condyle

A N A T O M Y L A B E L I N G 11 

Answer Key:

A. Cervical
B. Thoracic
C. Lumbar
D. Sacral
E. Coccyx

CHAPTER 6
A N AT O M Y L A B E L I N G 12 

Answer Key:

A. Skull
B. Cervical spine (Neck)
C. Acromion Process
D. Manubrium
E. Sternum (breast bone)
F. Xiphoid process
G. Thoracic spine
H. Costal cartilage
I. Lumber spine
J. Ilium
K. Pelvis
L. Femur head
M. Acetabulum
N. Pubis
O. Sacral Spine

A N AT O M Y L A B E L I N G 13 

Answer Key:

A. Clavicle (Collarbone)
B. Scapula (shoulder blade)
C. Ribs
D. Humorus
E. Elbow
F. Ulna
G. Radius

CHAPTER 6
A N AT O M Y L A B E L I N G 14 

Answer Key:

A. Coccyx (tail bone)


B. Carpals (wrist)
C. Metacarpals (hand)
D. Phalanges (fingers)
E. Femur (thigh bone)
F. Patella (knee cap)
G. Tibia
H. Fibula
I. Tarsals (ankle)
J. Metatatsals (foot)
K. Phalanges (toes)
L. Calcaneus (heel)
M. Medial malleolus
N. Lateral malleolus

A N AT O M Y L A B E L I N G 15 

Answer Key:

A. Orbicularis Oculi
B. Masseter
C. Orbicularis Oris
D. Trapezius
E. Deltoid
F. Pectoralis
G. Triceps
H. Serratus
I. Anterior Biceps
J. Lattisimus Dorsi
K. Rectus Abdominis
L. Rectus Femoris
M. Vastus Lateralis
N. Vastus Medialis
O. Gluteus Maximus

CHAPTER 6
A N AT O M Y L A B E L I N G 16 

Answer Key:

A. Tongue
B. Teeth
C. Sublingual Gland
D. Trachea
E. Parotid Gland
F. Pharynx
G. Submaxillary Gland
H. Larynx
I. Esophagus

A N A T O M Y L A B E L I N G 17 

Answer Key:

A. Liver
B. Gallbladder
C. Cystic Duct
D. Common Bile Duct
E. Duodenum
F. Hepatic Flexure
G. Ascending Colon
H. Ileocecal Valve
I. Cecum
J. Appendix
K. Rectum

CHAPTER 6
A N AT O M Y L A B E L I N G 18 

Answer Key:

A. Cardiac Spincter
B. Spleen
C. Stomach
D. Bile Ducts
E. Splenic Flexure
F. Pyloric Sphincter
G. Pancreas
H. Transverse Colon
I. Descending Colon
J. Ileum
K. Sigmoid Colon
L. Anus

QUIZ

1. The leaf-shaped flap that helps prevent food from entering the trachea is called the:
A. Pharynx.
B. Trachea.
C. Epiglottis.
D. Bronchiole.

2. What is one of the functions of the respiratory system?


A. To provide nutrients to the cells
B. To move air into and out of the lungs
C. To transport oxygen throughout the body
D. To release oxygen from the body

3. A patient complains of pain in the upper jaw, which is also called the:
A. Mastoid.
B. Zygoma.
C. Mandible.
D. Maxilla.

4. The right ventricle:


A. Pumps blood to the lungs.
B. Receives blood from the pulmonary veins.

CHAPTER 6
C. Receives blood from the veins of the body.
D. Pumps blood to the body.

5. The spinal column is composed of what sections as described from inferior to superior?
A. Cervical, lumbar, thoracic, sacral, coccyx
B. Coccyx, sacral, lumbar, thoracic, cervical
C. Cervical, thoracic, lumbar, sacral, coccyx
D. Sacral, thoracic, lumbar, cervical, coccyx

Answer Key: Chapter 6 - Quiz


1=C, 2=B, 3=D, 4=A, 5=B

TEST

1. Which chamber of the heart is responsible for pumping blood to the lungs?
A. The left atrium
B. The left ventricle
C. The right ventricle
D. The right atrium

2. Anatomy Labelling 01

3. Anatomy Labelling 11

4. Anatomy Labelling 09

5. When the diaphragm and intercostal muscles relax, which one of the following occurs?
A. Inspiration
B. Diastolic pressure increase
C. Systolic pressure increases
D. Exhalation

6. A patient experiencing thoracic spine pain has pain in his:


A. Tailbone
B. Upper back
C. Neck
D. Lower back

7. Which one of the following lists represents a correct sequence for the passage of air into the lungs?
A. Mouth, pharynx, trachea, bronchi, alveoli
B. Nose, bronchi, larynx, trachea, pharynx
C. Larynx, esophagus, trachea, bronchi, alveoli
D. Epiglottis, trachea, bronchi, alveoli, nasopharynx

8. Which of the following is a function of the skeletal system?


A. Producing blood cells

CHAPTER 6
B. Providing structure to blood vessels
C. Preventing the body from infection
D. Securing the abdominal organs in place

9. You arrive at the scene where a patient has a possible fracture of the left hip. You note that she is lying on her
back and her left leg is turned outward. You would document this position as:
A. Supine with external rotation of the left lower extremity.
B. Lateral recumbent with dorsal rotation of the left leg.
C. Supine with lateral rotation of the left lower extremity.
D. Supine with pronation of the left leg.

10. Under normal circumstances, carbon dioxide is excreted from the body by what system?
A. The urinary tract
B. The respiratory system
C. The intestinal tract
D. The integumentary system

11. A patellar fracture affects which part of the body?


A. The knee
B. The elbow
C. The tibia
D. The hip

12. Which of the following statements best describes the function of the respiratory system?
A. Providing the body oxygen
B. Releasing oxygen from the body
C. Providing nutrients to cells
D. Transporting oxygen throughout the body

13. Following chemotherapy, a patient has a low white cell count increases his risk for which of the following?
A. Seizures
B. Bleeding
C. High blood pressure
D. Infection

14. Anatomy Labelling 05

15. Which of the following groups of bones could be involved in a patient with a "broken leg"?
A. Radius, ulna, humerus
B. Femur, tibia, fibula
C. Acetabulum, calcaneous, carpals
D. Orbit, maxillae, mandible

16. The structure containing the vocal cords is the:


A. Larynx
B. Trachea
C. Sternum
D. Pharynx

CHAPTER 6
17. The bones of the upper extremities include the:
A. Phalanges and tibia.
B. Radius, ulna, and tarsals.
C. Humerus and calcaneous.
D. Humerus and radius.

18. Anatomy Labelling 02

19. A female patient suffers from a muscular disease and cannot walk. Based on this fact, you should recognize
which of the following muscle types is affected?
A. Involuntary muscle
B. Smooth muscle
C. Skeletal muscle
D. Cardiac muscle

20. You apply and use an AED on a patient lying on his back. His pulse returns and he is breathing adequately.
What position is he in?
A. Prone position
B. Supine
C. Left lateral recumbency
D. Right lateral recumbency

21. A patient has suffered damage to his brainstem. As a result of this injury which of the following signs might
you expect to see?
A. Sweating
B. Abnormal respirations
C. Pinpoint pupils
D. Muscle spasms

22. The nervous system is subdivided into which two main parts?
A. The central and peripheral nervous system
B. The brain and spinal cord
C. The brain and extremity nerves
D. The spinal cord and peripheral nerves

23. Anatomy Labelling 07

24. Anatomy Labelling 12

25. You are called to the lodge for a patient who has fallen. The report indicates the patient has a hematoma in
the occipital area of her head. Based on this description, you should expect to find the injury in which area of the
head?
A. The back
B. The side
C. The front
D. The top

CHAPTER 6
26. A new patroller tells you that he injured a tendon above his patella three years ago. You recognize that this
injury involves a structure that:
A. Connects the cartilage in the knee.
B. Attaches a ligament to the knee.
C. Attaches the patella to the femur.
D. Connects muscles to the patella.

27. The leaf-shaped flap that helps prevent food from entering the lower respiratory system is called:
A. The trachea
B. The epiglottis
C. A bronchiole
D. The pharynx

28. Anatomy Labelling 06

29. The thyroid glands, adrenal glands, pituitary glands, and gonads are part of the:
A. Respiratory system.
B. Hormonal system.
C. Endocrine system.
D. Nervous system.

30. The terms occipital, frontal, and parietal refer to what part of the body?
A. The pelvis
B. The chest
C. The spine
D. The cranium

31. Blockage of which of the following respiratory prevent air flow into and out of the lungs?
A. The trachea
B. The nasopharynx
C. The esophagus
D. The right bronchus could be correct use clavicle

32. The liquid that blood cells and nutrients are suspended in is called:
A. Platelet fluid
B. Hemoglobin
C. Plasma
D. Lymph

33. The lower jaw is also called the:


A. Mandible.
B. Maxilla.
C. Zygoma.
D. Mastoid.

34. What body system provides support and structure to the body?
A. The endocrine system
B. The nervous system

CHAPTER 6
C. The skeletal system
D. The exocrine system

35. The left atrium:


A. Pumps blood to the lungs.
B. Pumps blood to the body.
C. Receives blood from the veins of the body.
D. Receives blood from the pulmonary veins.

36. Anatomy Labelling 03

37. The pressure exerted on the inside walls of arteries when the left ventricle contracts is called:
A. Diastolic pressure
B. ventricle pressure
C. contraction pressure
D. Systolic pressure

38. As a patient gives her medical history to you, which of the following would you relate to the endocrine
system?
A. Removal of the gallbladder
B. Heart failure
C. Removal of the thyroid
D. Failure of the kidneys

39. The primary function of the heart is:


A. Transferring oxygen to the cells.
B. Pumping blood throughout the body.
C. Oxygenating blood in the lungs.
D. Maintaining the volume of blood.

40. Anatomy Labelling 13

41. Anatomy Labelling 17

42. Which of the following lists identifies the regions of the spinal column from superior to inferior?
A. Cervical, thoracic, lumbar, sacral, and coccyx
B. Cervical, lumbar, thoracic, sacral, and coccyx
C. Thoracic, lumbar, cervical, coccyx, and sacral
D. Coccyx, lumbar, thoracic, cervical, and sacral

43. Which of the following statements regarding the integumentary system is true?
A. The skin protects the body against bacteria and other pathogens.
B. The dermal layer is the outermost layer of the skin and contains sensory nerves.
C. The epidermis is the thickest and most important layer of the skin.
D. The skin contains four layers: the epidermal, dermal, subcutaneous, and nerve layers.

44. Anatomy Labelling 04

CHAPTER 6
45. A patient has suffered a burn to the skin. Based on the functions of the skin, to which of the following
conditions is the patient most susceptible?
A. Heart problems
B. Endocrine disorders
C. Fluid overload
D. Infection

46. Anatomy Labelling 15

47. Which one of the following descriptions best describes the normal anatomical position?
A. Standing with the arms down at the sides and the palms facing forward
B. Upright and facing away from you, with the arms raised and the legs straight
C. Prone with the arms upward and the palms facing upward
D. Supine with the arms at the sides and the palms facing downward

48. Anatomy Labelling 10

49. Anatomy Labelling 16

50. Which one of the following anatomical statements is true?


A. The shoulder is distal to the hand.
B. The knee is proximal to the ankle.
C. The hip is distal to the knee.
D. The hand is proximal to the elbow.

51. Anatomy Labelling 18

52. Involuntary or smooth muscles are found in which of the following structures?
A. The central nervous system
B. The heart and blood vessels
C. Blood vessels and intestines
D. Muscles of the arms and legs

53. Anatomy Labelling 08

54. Which of the following structures brings oxygen-depleted blood to the right atrium?
A. The vena cavae
B. The aorta
C. The pulmonary artery
D. The right ventricle

55. Which one of the following events causes an individual to take a breath?
A. The chest cavity decreases in size.
B. The intercostal muscles relax.
C. Pressure in the chest increases.
D. The diaphragm contracts.

56. Anatomy Labelling 14

CHAPTER 6
Answer Key: Chapter 6 - Test
1=C, 2=Matching, 3=Matching, 4=Matching, 5=D, 6=B, 7=A, 8=A, 9=A, 10=B, 11=A, 12=A, 13=D,
14=Matching, 15=B, 16=A, 17=D, 18=Matching, 19=C, 20=B, 21=B, 22=A, 23=Matching, 24=Matching,
25=A, 26=D, 27=B, 28=Matching, 29=C, 30=D, 31=A, 32=C, 33=A, 34=C, 35=D, 36=Matching, 37=D,
38=C, 39=B, 40=Matching, 41=Matching, 42=A, 43=A, 44=Matching, 45=D, 46=Matching, 47=A,
48=Matching, 49=Matching, 50=B, 51=Matching, 52=C, 53=Matching, 54=A, 55=D, 56=Matching

CHAPTER 6
CHAPTER 7
“Patient Assessment”

QUIZ

1. You are caring for a 25-year-old skier who has impacted a tree at high speed. He is bleeding from an obviously
fractured nose. Which of the following additional injuries or conditions would be discovered upon conducting the
secondary assessment?
A. A decreased level of consciousness
B. Bruising of the left side of the chest and left upper quadrant abdominal tenderness
C. Labored respirations
D. A rapid carotid pulse and sweaty skin

2. During the primary assessment, how should you best determine the adequacy of breathing?
A. By determining the patient's mental status
B. By determining the number of breaths per minute
C. By observing of the rise and fall of the chest
D. By comparing the respiratory rate to the heart rate

3. You cannot palpate either radial pulse on a 34-year-old male who is unresponsive, has an odor of alcohol on his
breath, and is breathing adequately. Which of the following should you do immediately?
A. Start positive pressure ventilation
B. Attach the automated external defibrillator (AED)
C. Start cardiopulmonary resuscitation
D. Check for the carotid pulse

4. You are approaching a patient who is complaining of shortness of breath and has a history of heart failure.
When assessing the airway, which of the following immediate observations best indicates a patent airway?
A. She is lethargic and has a gray overall appearance
B. She comfortably tells you that she is not having chest pain
C. She is sitting in an upright position because she cannot lie flat
D. She cannot speak in entire sentences without taking a breath

5. Which of the following best describes the purpose of serial documentation of the Glasgow Coma Scale (GCS)
score in a patient with a head injury?
A. It helps determine whether a patient's mental status is improving or deteriorating.
B. It allows the OECT to predict the patient's likelihood of recovery.
C. The score is important in guiding the OECT's treatment of the patient.
D. The score gives information about the type of injury to the brain.

6. Which one of the following patients should be described as unresponsive?


A. A 70-year-old female who has suffered a stroke

CHAPTER 7
B. A 38-year-old male who does not respond when his sternum is rubbed
C. A 52-year-old who cannot answer your questions
D. A 46-year-old-female who is confused

Answer Key: Chapter 7 - Quiz


1=B, 2=C, 3=C, 4=C, 5=A, 6=B

TEST

1. An OEC Technician identifies the medications a patient takes during which portion of a patient assessment?
A. The OPQRST exam
B. The primary assessment
C. The medical history
D. The chief complaint

2. Which of the following lists presents the order in which an OEC Technician should assess a patient in a field
setting?
A. Scene size-up, history, secondary assessment, primary assessment
B. Scene size-up, primary assessment, secondary assessment
C. Primary assessment, reassessment, secondary assessment
D. Scene size-up, secondary assessment, primary assessment

3. An OEC Technician's assessment of a trauma patient's circulation status during a primary assessment should
include:
A. Carotid palpation, capillary refill, blood pressure, and skin color.
B. Pulse rate, blood pressure, capillary refill, and skin condition.
C. Radial pulse, examination for the presence of external bleeding, and LOR.
D. Skin color, pulse rate, blood pressure, and skin temperature.

4. Which of the following patients should be classified as unresponsive?


A. A 39-year-old female who is confused
B. A 70-year-old female who is talking and has suffered a stroke
C. A 52-year-old who cannot answer your questions
D. A 46-year-old male who does not respond to a shoulder pinch

5. Which one of the following signs should an OEC Technician detect during a secondary assessment of a
critically injured patient?
A. Decreased level of responsiveness
B. Weak carotid pulse and clammy skin
C. Inadequate respiratory effort
D. Bruising and tenderness to the abdomen

6. While taking vital signs, you cannot locate a radial pulse in a possible broken arm of a 42-year-old male who is
alert and breathing adequately. Which one of the following should you do?

CHAPTER 7
A. Start cardiopulmonary resuscitation.
B. Skip taking pulse and move on to splinting
C. Attach the automated external defibrillator (AED).
D. Assess the carotid artery gently for a pulse.

7. Which of the following responses is an appropriate response for the letter P in the OPQRST mnemonic for a
patient with abdominal pain?
A. "I have a past history of asthma.”
B. "I ate pizza about two hours ago.”
C. "It hurts worse when I take a deep breath.”
D. "My pulse feels as though it is racing.”

8. A primary assessment should take about how long?


A. 4 minutes
B. 2 minutes
C. 3 minutes
D. 30-60 seconds

9. You have been dispatched for an elderly male complaining of shortness of breath. When should you start the
process of forming a general impression about this patient?
A. After completing a primary assessment
B. As soon as you obtain his chief complaint
C. While you and your partner are approaching him
D. After you have taken his vital signs

10. A complete set of vitals is taken at the:


A. Beginning of the secondary assessment.
B. End of the primary assessment.
C. End of the secondary assessment.
D. Beginning and end of the secondary assessment.

11. What letter in a SAMPLE interview indicates that a patient had pancakes for breakfast two hours ago?
A. L
B. P
C. E
D. M

12. A secondary assessment consists of which of the following three steps?


A. Medical history, trauma exam, and vital signs
B. The on-going exam, detailed physical exam, and vital signs
C. SAMPLE, physical exam, and vital signs
D. Applying an AICD, the on-going exam, and vital signs

13. As you assess a patient for a medical emergency, the patient states the pain is in their chest up to the shoulder
and down the right arm. To what letter of OPQRST would this apply?
A. R
B. Q
C. O

CHAPTER 7
D. P

14. While you are performing a secondary assessment, your trauma patient complains of the sudden onset of
difficulty breathing. Which of the following is your most appropriate response?
A. Stop the assessment and provide appropriate care.
B. Make a mental note and continue with the assessment.
C. Reevaluate the mechanism of injury.
D. Reassess the patient, beginning with the head.

15. You are by the side of a patient who is unresponsive and has a history of heart failure. When assessing the
airway, which one of the following observations best indicates an open airway?
A. She has a pulse of 80 per minute.
B. You can see that her tongue has fallen back where it appears to be blocking the airway.
C. You can hear normal respirations and see the chest rise and fall.
D. Her mouth is open and you can hear gurgling sounds.

16. Which of the following things does not indicate a possible safety issue during a scene size-up?
A. Ice or mud
B. Downed wires
C. Wild animals
D. ABCDs

17. Which of the following statements indicates that an OEC Technician correctly understands capillary refill?
A. "For a capillary refill test to be normal, it must be assessed at room temperature and be less than two
seconds.”
B. "Capillary refill is a reliable sign of perfusion in adults, but not as reliable in infants and children.”
C. "A capillary refill of four or more seconds in a patient of any age indicates that the patient is well
oxygenated.”
D. "Flushed skin in an adult is a normal finding when assessing capillary refill."

18. Which of the following statements would indicate that an OEC Technician is ready to begin the first phase of
a patient assessment?
A. "The scene is safe and appears to be free of hazards.”
B. "Can you tell me why you called for help?”
C. "I have placed an oral airway in the patient.”
D. "Blood pressure is 124/80 mmHg.”

19. Which of the following findings is a symptom?


A. Ringing in the ears
B. Hives resulting from an allergic reaction
C. High blood pressure
D. A deep laceration

20. You are assessing an eight-month-old whose mother states has been vomiting for two days and not eating or
drinking. When assessing the pulse in an infant, which site should you check first?
A. The brachial artery
B. The radial artery
C. The femoral artery

CHAPTER 7
D. The temporal artery

21. During assessment of a responsive medical patient, you gather a medical history using the memory aid
SAMPLE. To obtain information related to M, which one of the following questions should you ask?
A. "Do you have any past medical problems?”
B. "Are you currently taking any medications?”
C. "Can you point to where it hurts the most?”
D. "Are you allergic to any medications?"

22. You are assessing a four-year-old patient whose panicked mother states that she cannot wake him. The child
is breathing but his eyes are closed. To best determine the child's level of responsiveness, you should first:
A. Apply pressure to the child's nail beds.
B. Consider the child to be unresponsive.
C. Ask the mother how long the child has been asleep.
D. Say loudly, "Open your eyes!"

23. How should an OEC Technician best determine the adequacy of a patient's breathing during a primary
assessment?
A. Compare the patient's blood pressure and pulse rates.
B. Look for the rise and fall of the patient's chest.
C. Determine the patient's mental status.
D. Assess the patient's ability to raise their shoulders.

24. You have been dispatched to an 89-year-old female with an unspecified complaint. When assessing this
patient, which one of the following will the OEC Technician do as quickly as possible to develop a better
understanding of the emergency?
A. Gather a medical history, including information on allergies.
B. Determine if the chief complaint is medical (NOI) or trauma related (MOI).
C. Contact medical direction for advice.
D. Obtain the patient's vital signs and current medications.

25. The Glasgow Coma Scale score for verbal reaction that is assigned to a patient who answers questions
inappropriately is:
A. 2
B. 3
C. 4
D. 5

26. You arrive at the scene of a fall, where a 42-year-old woman fell backward off a stepladder while cleaning
windows. She is lying on the ground complaining of pain to her ankle. She tells you, "If I had just been more
careful and moved the ladder instead of reaching, this never would have happened!" Based on this information,
which of the following can you conclude?
A. She has an open airway, is breathing, and has circulating blood to her brain.
B. She does not require rapid transport.
C. Her pulse rate is within normal limits.
D. She does not have any other injuries.

27. An OEC Technician should obtain the medical history of an alert and oriented trauma patient during the:

CHAPTER 7
A. Secondary assessment
B. Reassessment
C. Primary assessment
D. Scene size-up

28. What letter of the OPQRST mnemonic reflects a medical emergency patient's report that difficulty in
breathing became worse about two hours ago?
A. Q
B. T
C. S
D. R

29. Forming a general impression is done before which phase of patient assessment?
A. Secondary assessment
B. Primary assessment
C. Reassessment
D. Communication and documentation

30. The finding that a patient's skin is warm, pink, and dry during a primary assessment suggests:
A. Normal circulation
B. Possible fever
C. Possible shock
D. Poor oxygenation

31. DCAP-BTLS is mnemonic to assist you in remembering:


A. What to assess while inspecting and palpating a patient.
B. The order in which to assess a patient only during a primary physical exam.
C. A trauma patient's vital signs.
D. Medications, allergies, and the patient's medical history.

32. A 36-year-old patient on the AVPU scale who has overdosed on an unknown drug is breathing and will not
open his eyes, even when his name is loudly called. Which one of the following should the OEC Technician do
next?
A. Identify what drug the patient took.
B. Check the patient's vital signs.
C. Assess the patient for a radial pulse.
D. Perform a shoulder pinch.

33. Which of the following tasks is part of a secondary assessment?


A. Checking for life-threatening bleeding
B. Inserting an OPA
C. Taking a blood pressure reading
D. Opening the airway

34. A primary assessment should take about how long?


A. 30-60 seconds
B. 3 minutes
C. 4 minutes

CHAPTER 7
D. 2 minutes

35.
Which of the following actions should you take during a secondary assessment of a 45-year-old female with a
severe headache?
A. Assess the head, reevaluate vital signs, and continue emergency care
B. Assess from head to toe, obtain a SAMPLE history, and obtain vital signs
C. Continue emergency care, obtain the chief complaint, and gather a SAMPLE history
D. Reassess vital signs, open the airway, and obtain OPQRST information

36. A secondary assessment should take about how long?


A. 30 seconds
B. 6-8 minutes
C. 2–5 minutes
D. 1 minute

37. Signs and symptoms can be identified using the:


A. OPQRST mnemonic.
B. AVPU mnemonic.
C. SAMPLE acronym.
D. DCAP-BTLS mnemonic.

38. Your patient is 5 years old and has fallen, resulting in a scraped knee. What letter indicates the scraped knee
in the DCAP-BTLS mnemonic?
A. D
B. S
C. A
D. L

39. You are performing a secondary assessment on the unrestrained adult passenger of a motor vehicle that rolled
several times at a high rate of speed. The patient has a skull laceration and is responsive to painful stimuli and in
a state of hypoperfusion. When assessing the head, which one of the following is appropriate?
A. Performing a blind finger sweep to clear the airway
B. Cleaning a scalp laceration
C. Checking the pupils with a pen light
D. Applying pressure to a skull laceration

40. As you assess a patient using the DCAP-BTLS mnemonic, the letter that denotes edema of the ankle is:
A. C
B. S
C. P
D. T

41. What letter of the OPQRST mnemonic reflects a medical emergency patient's report that difficulty in
breathing is worse while attempting to walk?
A. O
B. R
C. P

CHAPTER 7
D. Q

42. Which of the following findings is a sign, not a symptom?


A. An open leg fracture
B. Chest pain
C. An upset stomach
D. A sharp headache

43. A chief complaint is defined as the:


A. Primary reason the person is seeking medical care.
B. Findings from a secondary survey.
C. Findings from a primary survey.
D. Findings from a SAMPLE interview.

44. What letter in a SAMPLE interview indicates that a patient has a history of seizures?
A. M
B. A
C. S
D. P

45. Which one of the following statements best describes the purpose of obtaining a single Glasgow Coma Scale
score (GCS) in a patient with a head injury?
A. The score provides an objective measure of the patient's overall neurologic condition.
B. The score helps determine whether a patient's mental status is improving or deteriorating.
C. The score gives information about the type of injury to the brain.
D. The score guides an OEC Technician's treatment of the patient.

46. The D in DCAP-BTLS stands for:


A. Dyspnea
B. Damage
C. Deformity
D. Dislocation

47. A 44-year-old male was on a ladder cutting limbs from a tree when he fell. He is found to be unresponsive
with normal respirations. His breathing is adequate at a rate of 12 per minute and his carotid pulse is strong at 88
per minute. Following the primary assessment and initial management, which one of the following actions will
the OEC Technician complete while performing the secondary assessment?
A. Get a medical history from a family member.
B. Start positive pressure ventilation with a bag-valve mask.
C. Perform a jaw-thrust maneuver to open the airway.
D. Place an oral airway and put a cervical collar on the patient.

48. A Glasgow Coma Scale score less than ____ is associated with a high risk for major neurological injury.
A. 10
B. 8
C. 12
D. 13

CHAPTER 7
49. A secondary assessment should take about how long?
A. 30 seconds
B. 2-5 minutes
C. 6- 8 minutes
D. 1 minute

50. You are assisting at an Outdoor Emergency Care course. One of the students asks you how the head tilt-chin
lift airway maneuver works to open the airway. Your best response is:
A. "It opens the airway by lifting the tongue from the back of the throat.”
B. "It stimulates the patient to begin taking deeper breaths, thereby moving air into the lungs.”
C. "It creates an open airway by separating the lips, thereby allowing air to enter.”
D. "It opens the airway by relaxing and expanding the size of the throat.”

51. An easy way for an OEC Technician to remember what to look for in assessing injuries in a trauma patient is
the:
A. Acronym SAMPLE.
B. Mnemonic OPQRST.
C. Mnemonic APVU.
D. Mnemonic DCAP-BTLS.

52. You are interviewing a prospective candidate for the position of OEC Technician with your service. During the
interview, you ask the applicant to describe the purpose of the primary assessment. Which of the following best
describes that purpose?
A. To perform a comprehensive exam focused on the patient's chief complaint
B. To determine if the patient's vital signs are stable
C. To establish a chief complaint and gather an extensive medical history
D. To identify and treat life-threatening conditions

53. You have been called for an adult patient who suffered a seizure. He is sitting on the floor, and he looks at you
as you enter the room. When questioned, he responds to his name, but cannot remember the date or where he is.
This patient's mental status is best described as:
A. Alert and not confused.
B. Lethargic and agitated.
C. Visually disoriented.
D. Having an altered response to verbal stimuli.

Answer Key: Chapter 7 - Test


1=C, 2=B, 3=C, 4=D, 5=D, 6=D, 7=C, 8=D, 9=C, 10=D, 11=A, 12=C, 13=A, 14=A, 15=C, 16=D, 17=A,
18=A, 19=A, 20=A, 21=B, 22=D, 23=B, 24=B, 25=C, 26=A, 27=A, 28=B, 29=A, 30=A, 31=A, 32=D,
33=C, 34=A, 35=B, 36=C, 37=C, 38=C, 39=C, 40=B, 41=C, 42=A, 43=A, 44=D, 45=A, 46=C, 47=A,
48=D, 49=B, 50=A, 51=D, 52=D, 53=D

CHAPTER 7
CHAPTER 8
“Medical Communications”

QUIZ

1. Which statement about medical communication is NOT true?


A. Medical communication data flow has little or no effect on the safety of outdoor recreational activities.
B. Written and verbal communication skills are an essential component of outdoor emergency care.
C. Miscommunication with other agencies can adversely affect a patient's condition, outcome, or mortality.
D. The improper recording of data could result in a lawsuit being filed against an OECT.

2. An 18-year-old patient quietly tells you that she fears she may be pregnant, and you document this on the
patient's SOAP form. When the patient's mother arrives, she demands to see the form, stating that she has a right
to see it because she pays for her daughter's health insurance. Your best response to the mother would be to:
A. Hand the form to the daughter and tell her to give it to her mother because you cannot.
B. Tell her you cannot show her the form, but you are willing to answer any questions she may have about
her daughter's condition.
C. Allow her to see the form, because she is the payor on her daughter's health insurance.
D. Explain to her that HEPA privacy laws forbid your sharing this information with her.

3. Miscommunication can occur due to all but which of the following reasons?
A. Stress or data overload
B. The recipient does not interpret the message correctly
C. Active listening with interactive feedback
D. The listening/speaking rate gap

4. For which of the following could you be sued due to a HEPA violation?
A. Providing an incoming EMS unit with the patient's name; address; date of birth; and other private, personal
information
B. Advising base of an incident's status by way of the SAILER mnemonic
C. Sharing with an incoming medic unit that you believe the patient may have been using cocaine, although
the patient denies it
D. Responding to a phone call from a patient's employer that a patient was indeed sent to a local hospital in
the aftermath of an accident

5. Your handwriting and spelling are terrible. Which of the following would be the best method for you to
complete an incident form or SOAP report?
A. Ask a colleague to scribe the information for you, then both of you date and sign the form
B. Do your best, sign and date the form, and submit it as is
C. Do not complete the form until you can do so electronically with the help of 'spell check’
D. Transmit the information by radio or verbal report to incoming medic units

CHAPTER 8
6. You find that you have made an error on a patient report form. How do you best handle this?
A. Ignore it; once written it cannot be changed. Simply write the correct information below it and label it "use
this information," and then initial and date it
B. Blacken out the incorrect information so that it is not visible to others reading the form
C. Draw a single line through the error, and then initial and date it
D. Throw out the form and start a new one

Answer Key: Chapter 8 - Quiz


1=A, 2=D, 3=C, 4=D, 5=A, 6=C

TEST

1. Rather than rewriting the whole report when an error is made, which of the following is the best method for
correcting written errors on the report?
A. Draw a single line through the error, then write the correct information, initial, date and time the change.
B. Use white out to cover the mistake then correct the information and initial, date and time the change.
C. Erase the error then write the correct information then initial, date, and time the change.
D. Scratch the error out completely then write the correct information then initial, date, and time the change.

2. Written medical communications are generally used for all of the following except:
A. when giving press releases about mountain accidents.
B. for describing ways to improve the safety of outdoor activities.
C. to transmit information to the provider who is taking over care of a patient.
D. for quality-improvement activities.

3. In the acronym SAILER, the letter S stands for:


A. subjective.
B. splint.
C. symptoms.
D. sex.

4. The acronym FACTUAL-OEC helps OEC Technicians remember that good report writing should include which
of the following groups of characteristics?
A. Factual information, local patrol terminology, unbiased information
B. Accurate information, complete details, written in black or blue ink
C. Complete details, your determination of cause of accident, written legibly
D. Abnormal findings written in red, complete and accurate details, accepted medical terms

5. You are caring for a patient that you realize will require ALS transport to the hospital. In the acronym SAILER,
the letter that will assist you to remember to transmit this information during your radio call is:
A. S.
B. E.
C. R.
D. I.

6. In the CHEATED acronym, the letter T stands for:

CHAPTER 8
A. time.
B. testament.
C. technique.
D. treatment.

7. The three types of written medical communication OEC Technicians may encounter are:
A. accident reports, witness reports, and field notes.
B. incident reports, rescue reports, and patient care notes.
C. field care notes, patient care reports, and incident report forms.
D. patient care notes, accident reports, and rescue reports.

8. Medical responders need to be proficient in which two types of medical communication?


A. Written documentation and radio transmissions
B. Medical terminology and radio transmissions
C. Verbal communication and medical terminology
D. Written documentation and oral communication

9. When patients refuse care, the most important thing to document is:
A. whether they were accompanied by anyone when you left them.
B. the names of witnesses to what you said to them.
C. that the patients understood and accepted the risks of refusal.
D. their condition when you last saw them.

10. What is the best possible way to add or amend information to a report that has already been submitted?
A. Submit the additional information as an addendum to the report then initial, date, and time the addendum
B. Once the report is submitted you can no longer add information and should only add the information to
your notes.
C. Ask for the report back so you can add to the report then initial the changes
D. Write the additional information on a copy of the form you kept for your records and resubmit the form
with your initials and date.

11. As an OEC technician you are more than likely considered a "mandated reporter." Below is a list of the injuries
or crimes the law requires you to report except.
A. A raccoon bite
B. Assault
C. Verbal aggressiveness
D. Abusive care that causes injury to an 81 year old

12. The characteristics of good report writing can easily be remembered by using mnemonic FACTUAL-OEC.
Some of these acronym letters stand for:
A. actions taken, clinical picture, and timeline.
B. clinical picture, timeline, and assessment.
C. facts, terms, unbiased, and legible.
D. focused, age, concurrent, and unusual events.

13. An incident report form (IRF) is provided by the area's insurance carrier. What is the purpose of the IRF?
A. To collect data surrounding the patient and incident, gather patient care data and is used to communicate
medical information to other parties.

CHAPTER 8
B. It is designed to communicate only medical information to other parties
C. To collect data regarding your opinion of the incident
D. To collect data concerning only the patient and the circumstances surrounding an incident

14. Which of the following documents may be used as a medical-legal record for patient care?
A. NSP awards, Fund raising awards, Snow report
B. Supply Report, Annual Budget, Insurance carrier
C. Patrol schedule, automobile insurance certificate, List of surrounding resorts
D. Field Notes of the Patroller, Patient Care Report (PCR), Incident Report Forms

15. An individual who is required to report cases of child abuse to the appropriate state agency is called a(n):
A. mandated reporter.
B. required abuse reporter.
C. emergency protector.
D. child guardian.

16. In the acronym FACTUAL-OEC, the letters OEC stand for:


A. objective, effective, and care given.
B. outdoor emergency care.
C. organized, error free, and checked.
D. organized, effective, and complete.

17. The goal of effective communication is to:


A. use medical terminology as much as possible.
B. deliver information in a manner that is understood by the recipient.
C. speak clearly and more slowly than normal so that you are easier to understand.
D. give as much information as possible in less than three minutes.

18. The PCR (patient care record) is a legal medical document completed by the OEC technician. Therefore, good
written documentation is important. In the list below choose the answers that represent the components of a
good written case report.
A. Using only slang spoken on the street and by the skier
B. Errors are not a problem.
C. Appropriate medical abbreviations and terms
D. Biased with your opinion

19. When a patient arrives at the first-aid station a status of the patient is given. The name of the brief oral report
is called what?
A. Pass-off report
B. Hand-off report
C. Tag-out report
D. Trade-off report

20. You are the lead responder at a serious accident. After the accident you can provide patient care information
to all of the following except:
A. the EMT to whom you transfer patient care.
B. a reporter who is writing a news story.
C. your patrol director, who is reviewing the PCR with you.

CHAPTER 8
D. your medical director, who is providing you stress debriefing.

21. The Patient Care Report (PCR) is best organized and completed by using which of the following methods.
A. CHEATED and DISTRACTED
B. LIES and CHEATED
C. SOAP and CHEATED
D. SOAP and LATHER

22. The OEC technician can verbally report to others in a timely method about an incident using the acronym
"SAILOR." The acronym is best represented by which one of the following?
A. The patient's gender and complaints, your location, oxygen, and blankets
B. The patient's name, gender, and complaints, your location, a toboggan, and airway
C. The patroller, the patient's gender and complaints, location of the patrol room and other patrollers.
D. Who you are, gender and age of the patient, the chief complaint of the patient, your location, the
equipment you need and the other resources you need

23. Among the injuries or crimes that must be reported to an appropriate state agency are:
A. animal bites, physical assaults, and child abuse.
B. abuse, gunshot wounds, and alcohol abuse.
C. incidents involving alcohol, domestic violence, and stab wounds.
D. stab wounds, accidents involving snow guns, and communicable diseases.

24. Which of the following pairs are the best examples of the two main types of communication at which OEC
Technicians must be proficient?
A. Face-to-face communication and radio communication
B. Radio communication and telephone communication
C. Written communication and nonverbal communication
D. Radio communication and written communication

25. The three forms of communication are:


A. written, verbal, and listening.
B. written, listening, and hand gestures.
C. speaking, hand gestures, and facial expressions.
D. oral, nonverbal, and written.

26. A patient may elect not to receive treatment. Which two criteria must be included on a Refusal of Care (or
A.M.A.) form?
A. The patient name and their religious preference
B. The patient understood the risks of refusal and accepts the risk or refused to listen to the risks
C. The family's signature and the lawyers signature
D. The patient refusal to leave the hill and injury

27. On which form would you be most likely to use the SOAP or CHEATED acronyms
A. Hand-off reports
B. NSP application
C. Patient Care Report
D. Annual Report

CHAPTER 8
28. Identify the two types of medical communications that are most important to inform other medical providers
of a patient's condition and progress.
A. Verbal communication and sign language
B. Oral communication and written documentation
C. Verbal and nonverbal communication
D. Oral and transmitted communication

29. After your initial evaluation of the patient you would communicate by radio with patrol base. Following the
acronym SAILER, what information would you include?
A. Surname, age, address, chief complaint
B. Age, next of kin, treatment initiated, request for EMS (if needed)
C. Sex, age, chief complaint, splints needed
D. Surname, location of accident, age, insurance company

30. Please select the most effective components that the OEC technician uses in medical documentation and that
best represent the primary parts of the assessment and management process.
A. Patient name, history, exam of incident, asking for advice, temperament, involvement and distractors
B. Chief complaint, history, examination, assessment, treatment, evaluation and disposition
C. Chief counsel, highest injury, achievement, trail skied, evolution and deposition
D. Chances taken, history, written exam, attributes, toboggan used, evacuation and disposition

31. You are completing your patient care report and are using the acronym CHEATED to guide you. In this
acronym, the letter A indicates:
A. actions taken.
B. age.
C. assessment.
D. address.

32. Communication is defined as the process by which:


A. a message is encoded from the sender to the receiver.
B. verbal and nonverbal actions deliver information to another person.
C. information is documented on a patient accident report.
D. a message is transmitted from a sender to a receiver.

Answer Key: Chapter 8 - Test


1=A, 2=A, 3=D, 4=B, 5=C, 6=D, 7=C, 8=D, 9=C, 10=A, 11=C, 12=C, 13=A, 14=D, 15=A, 16=C, 17=B,
18=C, 19=A, 20=B, 21=C, 22=D, 23=A, 24=D, 25=D, 26=B, 27=C, 28=B, 29=C, 30=B, 31=C, 32=D


CHAPTER 8
CHAPTER 9
“Airway Management"

QUIZ

1. Correctly applied, what is the device most likely to deliver the maximum amount of oxygen to a spontaneously
breathing patient?
A. Nonrebreather face mask
B. Simple face mask
C. Venture mask
D. Nasal cannula

2. A jaw-thrust maneuver would be best applied in which of the following circumstances?


A. Stroke with gurgling respirations
B. Cardiac arrest on the ski slop
C. An unresponsive patient with snoring sounds after falling from a ski lift
D. Snoring respirations following a seizure

3. A spontaneously breathing patient has just had his airway opened and cleared. While waiting for additional
equipment to arrive, an easy way to help maintain this airway is to:
A. Use the crossed-finger maneuver to open the mouth.
B. Maintain the head-tilt/chin-lift position.
C. Insert an oropharyngeal airway to keep the mouth open.
D. Place the patient in the lateral recovery position.

4. Which of the following patients would benefit from the head-tilt/chin lift manual airway maneuver?
A. An 85-year-old female who hit her head on a tree limb and is now responsive to painful stimuli
B. A 27-year-old male with a suspected intravenous drug overdose and snoring respirations
C. A 45-year-old female who is alert and vomiting blood
D. A 67-year-old female who fell down a flight of stairs and is unresponsive

5. Which one of the following best describes the path that carbon dioxide must travel to get from the lungs to the
outside air?
A. Alveoli, bronchioles, bronchi, trachea, larynx, oropharynx
B. Nasopharynx, oropharynx, trachea, alveoli
C. Alveoli, bronchi, esophagus, pharynx, nose
D. Bronchioles, alveoli, trachea, pharynx, larynx

Answer Key: Chapter 9 - Quiz


1=A, 2=C, 3=D, 4=B, 5=A

CHAPTER 9
TEST

1. You hear a candidate OEC Technician explaining the use of a barrier shield to another candidate. Which of the
following statements would indicate that he does not fully understand its purpose or how to use it?
A. "A barrier device is as good as any of the other airway adjuncts for delivering oxygen.”
B. "A barrier device cannot be used as personal protective equipment.”
C. "A barrier device may have either a mesh opening or a breathing tube in the center.”
D. "You need to open the patient's airway before using a barrier device."

2. How is the jaw-thrust maneuver indicated for a patient with a possible spinal injury?
A. It is less painful for the patient.
B. It minimizes movement of the head and cervical spine.
C. It is the rescuer's preference as to whether to use the jaw-thrust maneuver.
D. It is a permanent intervention that does not require insertion of an airway if performed correctly.

3. How is the jaw-thrust maneuver indicated for a patient with a possible spinal injury?
A. It is less painful for the patient.
B. It minimizes movement of the head and cervical spine.
C. It is the rescuer's preference as to whether to use the jaw-thrust maneuver.
D. It is a permanent intervention that does not require insertion of an airway if performed correctly.

4. You have been asked to put a patient on a nonrebreather mask at 15 LPM. After you have finished attaching the
regulator to the oxygen cylinder, your next step is to:
A. Attach the nonrebreather mask to the regulator and fill the reservoir.
B. Check the pressure gauge indicator to determine how much pressure is in the cylinder.
C. Turn the oxygen control knob to 15 LPM.
D. Turn the valve stem on the top of the cylinder using a special oxygen wrench or key.

5. You have been asked to put a patient on a nonrebreather mask at 15 LPM. After you have finished attaching the
regulator to the oxygen cylinder, your next step is to:
A. Attach the nonrebreather mask to the regulator and fill the reservoir.
B. Check the pressure gauge indicator to determine how much pressure is in the cylinder.
C. Turn the oxygen control knob to 15 LPM.
D. Turn the valve stem on the top of the cylinder using a special oxygen wrench or key.

6. Which of the following occurs during the proper application and use of a nasal cannula is true?
A. Patients often prefer a nasal cannula over a non-rebreather because it is less restrictive
B. A nasal cannula provides high flow oxygen
C. The oxygen flow rate can be adjusted anywhere from 10 LPM to 15 LPM, depending on how much oxygen
the patient needs.
D. It is best to see how a patient tolerates a nasal cannula before attempting to place a nonrebreather mask on
the patient.

7. Which of the following statements indicates that the speaker knows how to properly size an oropharyngeal
airway before its placement?

CHAPTER 9
A. "The length of the airway should approximate the distance from the tip of the nose to the angle of the jaw.”
B. "An appropriately sized oral airway can be selected by looking in the patient’s mouth and estimating the
length of the tongue.”
C. "The length of the airway should approximate the distance from the corner of the mouth to the angle of
the jaw.”
D. "To select an appropriately sized airway, you must first estimate the patient’s height and weight."

8. The lower airway consists of which of the following structures?


A. Larynx, bronchi, and alveoli
B. Trachea, bronchi, and alveoli
C. Pharynx, bronchi, and alveoli
D. Epiglottis, trachea, and bronchi

9. You have been asked to teach a new patroller how to put a patient in the recovery position. Which of the
following statements about the recovery position would be correct?
A. "After turning the patient on his side, rest the patient's head on your aid pack.”
B. "Elevate the patient's lower body so that his oral secretions will drain out easily.”
C. "Roll the patient onto his back and then hold him there until help arrives.”
D. "Turn the patient on his side and flex his upper leg so that it anchors him on his side."

10. When monitoring a patient receiving oxygen through a nasal cannula, which of the following observations
warrants immediate intervention?
A. Prongs in the nostrils that curve posteriorly into the nose
B. Tubing that is positioned over the ears and under the chin
C. Normal patient breathing while the nasal cannula is in place
D. An oxygen flow rate of 15 LPM

11. The lower airway consists of which of the following structures?


A. Epiglottis, trachea, and bronchi
B. Pharynx, bronchi, and alveoli
C. Larynx, bronchi, and alveoli
D. Trachea, bronchi, and alveoli

12. Which of the following statements shows an understanding by the speaker of the use of a pocket mask?
A. "A pocket mask must be securely placed over the patient's mouth and nose.”
B. "A pocket mask should be used only when oxygen is connected to it.”
C. "Although a pocket mask is very effective at ventilating a patient, you are directly exposed to the patient's
secretions.”
D. "When using a pocket mask to ventilate a patient, you must make sure that the reservoir bag is in place."

13. You appropriately size a nasopharyngeal airway by measuring the:


A. Distance from the patient's mouth to the angle of the jaw.
B. Distance from the patient's nose to the earlobe.
C. Diameter of the patient's larger nostril.
D. Diameter of the patient's thumb.

14. By placing an oropharyngeal airway in a patient, you have:


A. Kept the tongue from occluding the airway.

CHAPTER 9
B. Obtained a patent airway by keeping the mouth and nares from closing.
C. Protected the airway from vomit or other secretions.
D. Minimized the risk of vomiting by closing off the esophagus.

15. What does the letter S in the acronym SLIC stand for?
A. Size
B. Septum
C. Slippery
D. Slowly

16. Your patient has just had a seizure and is presenting with snoring respirations. You will open her airway with
the head tilt-chin lift method. Which option correctly describes this method?
A. Place one hand on the forehead and one hand under the back of the neck
B. Place on hand on the forehead and one hand on the boney part of the lower jaw
C. Place one hand on the forehead and one hand in the mouth
D. Place your fingers under the angles of the jaw

17. The most commonly used method for opening a patient's mouth to suction the oropharynx, to perform a
finger sweep, or to insert an oral airway is:
A. a Yankauer catheter.
B. Sellick's maneuver.
C. the finger sweep.
D. the crossed finger technique.

18. You have been ventilating the patient with an oropharyngeal airway and bag-valve mask. Suddenly the patient
regains consciousness and starts to gag. Your immediate action should be to:
A. Remove the airway.
B. Leave the airway in but stop ventilations.
C. Suction around the airway
D. Reassure and calm the patient.

19. You are caring for an unconscious patient who has vomited and appears to have a blocked airway. Using the
crossed finger technique you open the patient's mouth and can see a large piece of undigested food near the back
of the throat. You would:
A. Insert an airway
B. Perform a finger sweep
C. Administer abdominal thrusts
D. Encourage the patient to cough

20. To clear an airway of fluid and debris:


A. Insert the tip of the suction catheter as deeply as possible
B. Apply suction prior to placing the tip into the mouth
C. Suction the airway deep by using an up and down motion
D. Insert the tip of the suction catheter only as far as you can see

21. Which of the following statements describes the correct positioning of a BVM on an adult patient's face?
A. If two rescuers are using the BVM, the mask need only be placed over the mouth while the nose is pinched
closed.

CHAPTER 9
B. The mask is properly positioned when the top portion lies over the bridge of the nose, and the lower
portion is below the chin.
C. The narrow part of the mask is over the bridge of the nose, and the bottom part is in the. Groove between
the lower lip and chin.
D. The wide portion of the mask is at the top of the nose, and the narrow part is below the lower lip.

22. You are checking equipment at the beginning of your shift. The D size oxygen cylinder measures 1000 psi.
You calculate that if you needed to use this tank at 15 LPM it would last:
A. 5 minutes.
B. 15 minutes.
C. 8.5 minutes.
D. 12 minutes.

23. What is the most common oxygen delivery device used by OEC Technicians?
A. A venture mask
B. A simple face mask
C. A nonrebreather mask
D. A nasal cannula

24. Which of the following statements about airway management in trauma patients is correct?
A. Trauma patients should always have their airway opened with the jaw-thrust maneuver, whereas medical
patients should receive the head tilt-chin lift maneuver.
B. Performing the head tilt-chin lift maneuver on a patient with a possible spinal injury could further injure
the patient.
C. Research has shown that the jaw-thrust maneuver is easiest to perform and is therefore indicated only for
critically injured trauma patients.
D. The jaw-thrust maneuver is useful in trauma patients but only if they are complaining of head or neck
pain.

25. You have been asked to put a patient on a nonrebreather mask at 15 LPM. After you have finished attaching
the regulator to the oxygen cylinder, your next step is to:
A. attach the nonrebreather mask to the regulator and fill the reservoir.
B. check the pressure gauge indicator to determine how much pressure is in the cylinder.
C. turn the valve stem on the top of the cylinder using a special oxygen wrench or key.
D. turn the oxygen control knob to 15 LPM.

26. You want to open your patient's mouth to check for obstruction with the cross-finger technique. Which of the
following describes the best way to accomplish the cross-finger technique?
A. Place your fingers on the lower teeth and pull downwards to open the mouth
B. Place your index finger and thumb between the patient's teeth and into the oropharynx. Then push the
teeth apart
C. Cross your index finger and thumb on the boney part of the lower jaw. Then push up on the jaw to open
the mouth
D. Place your index finger on the upper teeth and thumb on the lower teeth. Then spread your finger and
thumb apart opening the mouth.

CHAPTER 9
27. Despite coaching and explaining the benefits of a nonrebreather face mask, a female patient with chest pain
panics and states that she cannot tolerate the mask over her face. The more that she panics, the worse the chest
pain becomes. Your best course of action is to:
A. Decrease the oxygen flow rate entering the nonrebreather mask.
B. Discontinue oxygen therapy and continually monitor breath sounds.
C. Disconnect the nonrebreather and replace it with a nasal cannula.
D. Remove the nonrebreather mask and apply a simple face mask.

28. You are observing a rescuer insert an oropharyngeal airway into the airway of a 36-year-old male who has
overdosed. Which one of the following observations indicates correct technique?
A. The rescuer inserts the airway with the tip pointed towards the tongue until the flange of the airway is 1
cm above the patient's lips.
B. The rescuer inserts the airway into the patient's mouth with the tip pointed towards the roof of the
mouth and then turns it 180 degrees once it is halfway in the mouth.
C. The rescuer inserts the airway sideways into the mouth and then rotates it 180 degrees once it has reached
the base of the tongue.
D. The rescuer inserts the airway into the patient's mouth with the tip pointed towards the roof of the mouth
and then turns it 90 degrees.

29. Which of the following statements indicates that the speaker understands how and when to use an
oropharyngeal airway?
A. "I must watch the patient for vomiting or gagging even with the proper insertion.”
B. "It can be used for patients who respond to painful stimuli”
C. "It protects the patient from aspirating vomit or other secretions.”
D. "It should be sized from the tip of the nose to the angle of the jaw"

30. Your patrol director has asked you to check the oxygen tanks and let her know how long they will last at an
accident. To calculate this information you would use which of the following formulas?
A. (Gauge pressure divided by flow rate) minus safe residual pressure
B. (Gauge pressure in psi minus safe residual pressure) times cylinder size constant divided by flow rate in
LPM
C. (Flow rate in LPM divided by cylinder size) times gauge pressure
D. (Cylinder size minus safe residual pressure in psi) times flow rate in LPM

31. A time-honored technique that may be used on conscious or unconscious patients to quickly remove fluid and
solids from the airway is:
A. the crossed finger technique.
B. gravity.
C. a jaw thrust.
D. a finger sweep.

32. The most commonly used method for opening a patient's mouth to suction the oropharynx, to perform a
finger sweep, or to insert an oral airway is:
A. Sellick's maneuver
B. The crossed finger technique
C. A Yank Auer catheter
D. The finger sweep

CHAPTER 9
33. Which of the following statements shows that the OEC technician understands how to suction patients
properly in a field setting?
A. "It is helpful to pour sterile water into a patient's mouth to liquefy vomit and make it easier to suction
out.”
B. "When there is active bleeding in the mouth, I may need to use gravity and suction concurrently.”
C. "A rigid suction catheter is affected more by temperature than a flexible catheter and therefore flexible
catheters should be chilled”
D. "To thoroughly suction a patient, I insert the rigid tip of the catheter past the base of the tongue and into
the pharynx."

34. You are correctly performing the jaw-thrust maneuver when you:
A. Maintain the patient's head in a neutral position and tilt the head slightly backward.
B. Open the patient's airway by slightly pushing down on the forehead and thrusting the jaw upward.
C. Place one hand on the patient's forehead and lift the jaw upward with the other hand by inserting the
thumb in the mouth.
D. Use the thumbs or fingers as a lever to lift the patient's mandible forward.

35. You have opened and cleared your unresponsive patient's airway and there is no evidence of head or spinal
injury. While waiting for other equipment or personnel, the easiest way to maintain this state and ensure
adequate breathing is to put this patient into the recovery position. Which of the following best describe this
position?
A. Place the patient into a semi seated position
B. Place the patient onto his back
C. Place the patient onto his left side
D. Place the patient with his head down and feet up

36. Which of the following lists the anatomical structures of the upper airway?
A. Nose, pharynx, bronchi, alveoli
B. Nasopharynx, oropharynx, trachea, alveoli
C. Nasopharynx, Oropharynx, larynx, epiglottis
D. Pharynx, larynx, alveoli, bronchioles

37. You have been asked to teach a new patroller how to put a patient in the recovery position. Which of the
following statements about the recovery position would be correct?
A. "Turn the patient on his side and flex his upper leg so that it anchors him on his side.”
B. "Roll the patient onto his side and then hold him there until help arrives.”
C. "After turning the patient on his side, rest the patient's head on your aid pack.”
D. "Elevate the patient's lower body so that his oral secretions will drain out easily."

38. Which of the following patients has an actual or potential critical occlusion of the upper airway?
A. A 78-year-old with a mucus plug in a right bronchioles
B. A 61-year-old with a severe cough
C. A 3-year-old with a fever and swelling of the larynx
D. A 45-year-old with spasm of the bronchioles

39. If while checking a size D oxygen tank you notice that the reading on the pressure regulator is 1000 psi, you
should recognize that the tank is:
A. Leaking

CHAPTER 9
B. Almost empty
C. Overfilled
D. Half full

40. Which of the following statements shows that the speaker understands how to properly store an oxygen tank
after use?
A. "I'll tighten the valve stem as much as I can so it won't come loose.”
B. "I'll stand the oxygen tank on the floor beside the stretcher so everyone will know where it is.”
C. "I'm going to leave the regulator set to 15 LPM so it will be ready when we need it.”
D. "I'll place the oxygen tank in its protective case until we need to use it again."

41. Which of the following statements about a nasal cannula is true?


A. The oxygen flow rate can be adjusted anywhere from 1 LPM to 10 LPM, depending on how much oxygen
the patient needs.
B. It is best to see how a patient tolerates a nasal cannula before attempting to place a nonrebreather mask on
the patient.
C. When the nasal cannula is set to the highest appropriate oxygen flow rate, it delivers 24-44 percent
oxygen.
D. A nasal cannula is preferred over a nonrebreather mask because the cannula administers the oxygen
through the nose instead of through the mouth.

42. Which of the following occurs during the proper application and use of a nonrebreather mask?
A. The flow rate should be set to 1 to 6 p.m.
B. The mask will cover the mouth only so that air can be breathed into the nose.
C. The oxygen reservoir should completely collapse each time the patient inhales.
D. The oxygen reservoir should be inflated prior to application of the mask

43. For which of the following patients is placement of an oropharyngeal airway indicated?
A. A patient who has vomited and responds to painful stimuli by moaning
B. An unresponsive patient who has neither a gag reflex nor a cough reflex
C. A responsive but confused patient with stridor us respirations
D. A patient with snoring respirations who coughs as the oral airway is placed into his mouth

44. Which of the following statements shows an understanding by the speaker of the use of a pocket mask?
A. "Although a pocket mask is very effective at ventilating a patient, you are directly exposed to the patient's
secretions.”
B. "A pocket mask should be used only when oxygen is connected to it.”
C. "When using a pocket mask to ventilate a patient, you must make sure that the reservoir bag is in place.”
D. "A pocket mask must be securely placed over the patient's mouth and nose."

45. You recognize that an oropharyngeal airway has been appropriately inserted when:
A. It cannot be dislodged by the rescuer.
B. Air moves freely in and out of the airway.
C. The flange sits about 1/4 inch from the patient's lips.
D. Vomiting is no longer occurring.

46. Which of the following statements shows that the speaker understands how to properly store an oxygen tank
after use?

CHAPTER 9
A. "I'm going to leave the regulator set to 15 LPM so it will be ready when we need it.”
B. "I'll stand the oxygen tank beside the stretcher so everyone will know where it is.”
C. "I'll place the oxygen tank in its protective case until we need to use it again.”
D. "I'll tighten the valve stem as much as I can so it won't come loose.”

47. Your patrol director has asked you to check the oxygen tanks and let her know how long they will last at an
accident. To calculate this information you would use which of the following formulas?
A. (Flow rate in LPM divided by cylinder size) times gauge pressure
B. (Gauge pressure in psi minus safe residual pressure) times cylinder size constant divided by flow rate in
LPM
C. (Cylinder size minus safe residual pressure in psi) times flow rate in LPM
D. (Gauge pressure divided by flow rate) minus safe residual pressure

48. You hear a candidate OEC Technician explaining the use of a barrier device to another candidate. Which of the
following statements would indicate that he fully understands its purpose or how to use it?
A. "It is important to prefill the oxygen reservoir prior to using the barrier device”
B. "A barrier device cannot be used as personal protective equipment.”
C. "A barrier device will keep the tongue from blocking the airway”
D. "When ventilating a patient with a barrier device, watch for the rise and fall of the patient's chest."

49. To clear an airway of fluid and debris, suctioning should be applied:


A. as deeply as needed to remove as much fluid as possible.
B. for as long as needed.
C. for no more than 30 seconds at a time.
D. for no more than 10-15 seconds at a time.

50. If a patient gags while you are inserting an oropharyngeal airway, you should remove the airway and:
A. Use a tongue depressor to better place the airway
B. Reattempt to insert it
C. Have suction ready
D. Reattempt to insert it using a smaller airway

51. You have opened and cleared your patient's airway. While waiting for other equipment or personnel, the
easiest way to maintain this state and ensure adequate breathing is to:
A. use the crossed finger technique.
B. place a suction catheter in the corner of the patient's mouth to continually remove secretions.
C. place the patient in the left lateral recumbent (recovery) position if the patient is unresponsive and spinal
injury is not suspected.
D. maintain the head tilt-chin lift position.

Answer Key: Chapter 9 - Test


1=A, 2=B, 3=C, 4=D, 5=C, 6=A, 7=C, 8=B, 9=D, 10=D, 11=D, 12=A, 13=B, 14=A, 15=A, 16=B, 17=D,
18=A, 19=B, 20=D, 21=C, 22=C, 23=C, 24=B, 25=C, 26=D, 27=C, 28=B, 29=A, 30=B, 31=B, 32=B, 33=B,
34=D, 35=C, 36=C, 37=A, 38=C, 39=D, 40=D, 41=C, 42=D, 43=B, 44=D, 45=B, 46=C, 47=B, 48=D,
49=D, 50=C, 51=C


CHAPTER 9
C HAPTER 10
“Shock Management”

QUIZ

1. Shock can be described as a physiologic state in which:


A. There is adequate oxygen in the lungs but not enough nutrients are in the bloodstream for cell survival.
B. The amount of oxygen to tissues is adequate but carbon dioxide is not being eliminated.
C. The cells of critical organs are getting glucose and other nutrients but not oxygen.
D. The cells of all organs are not getting enough oxygen and waste products are accumulating.

2. You are reassessing a patient who has sustained an open femoral shaft fracture, which has been appropriately
treated with bandaging and a traction splint. It has been 10 minutes since the initial evaluation. Which of the
following reassessment findings best indicates that the patient is deteriorating and is in the decompensating
phase of shock?
A. Blood pressure has dropped from 116/63 to 88/50
B. The patient has become anxious and restless
C. Respiratory rate has increased from 16 to 25
D. There is continued bleeding from the thigh wound despite bandaging and splinting

3. In your evaluation of a patient with chest pain and hyptotension, you come to the conclusion that he is likely
experiencing cardiogenic shock. He has a palpable carotid pulse. Your most appropriate immediate action would
be to:
A. Ask the patient if he has nitroglycerin pills.
B. Administer high-flow oxygen.
C. Immediately apply an AED.
D. Transport to a hospital with the patient supine.

4. Which of the following is most correct when evaluating the physiologic effects of significant bleeding?
A. The amount of blood loss is often overestimated by the amount visible on the snow.
B. Children will show signs of shock much quicker than an adult.
C. Advanced age and poor general health may significantly contribute to intolerance of blood loss.
D. Signs of shock do not generally appear until a large amount of blood is lost.

5. A snowboarder has missed the correct landing area on a terrain feature and has impacted at high speed the flat
runout zone from an estimated height of 25 feet. You cannot feel radial pulses, the carotid pulse is weakly
palpable at 125 beats per minute, he is anxious, and the skin is pale and sweaty. His abdomen is exquisitely
tender to palpation. What best describes this situation?
A. Obstructive shock from blood clots in the major blood vessels
B. Cardiogenic shock from damage to the heart during impact
C. Hypovolemic shock from internal bleeding
D. Distributive shock from shunting blood from the extremities to the core

C HAPTER 10
Answer Key: Chapter 10 - Quiz
1=D, 2=A, 3=B, 4=C, 5=C

TEST

1. Which of the following lists of assessment findings indicates that a trauma patient may be in compensated
shock?
A. Slightly confused, tachycardia, hypotensive, warm and flushed skin
B. Anxious, tachycardia, normal BP, pale and cool skin
C. Confused, very bradycardic, hypotensive, cool and cyanotic skin
D. Confused and anxious, very tachycardic, very hypotensive, cool and mottled skin

2. A 56-year-old female has struck a tree. Your assessment reveals gurgling respirations, rapid breathing, and
cool, diaphoretic skin. You also observe bruising to the chest and abdomen. Which one of the following should be
your first concern?
A. Elevate her feet
B. Open and maintain her airway
C. Determine her pulse rate
D. Evaluate her for shock

3. You are treating a patient with severe chest pain and believe they are in cardiogenic shock. One of your first
treatments would be:
A. Application of an AED
B. Rapid transport in a prone position
C. High-flow oxygen
D. Having the patient take their nitroglycerin

4. A Factor that does not affect the body's ability to compensate for shock is:
A. The lack of injury or illness
B. Mental health and mind altering substances
C. Preexisting medical conditions
D. Body Temperature

5. The underlying cause of distributive shock is:


A. Loss of blood volume
B. Dilation of blood vessels
C. Poor fluid intake
D. A damaged heart that has poor contractility

6. Which of the following conditions is the most probable cause of cardiogenic shock?
A. Severe vomiting and diarrhea
B. Systemic infection
C. Gastrointestinal bleeding
D. Myocardial infarction

C HAPTER 10
7. The degree to which hemoglobin has bound to oxygen is called:
A. Hematocrit
B. Oxygen saturation
C. Oxygen capacity
D. Deoxyhemoglobin saturation

8. The smaller branches of the arterial tree that contain a muscular layer that enables changes in vessel diameter
are called _______; these vessels ________.
A. Metabolic vessels; trigger the release of glucose into the circulatory system
B. Vernicles; exchange O2 and CO2 and rid the body of waste products
C. Resistance vessels; enable the body to control how much blood is directed toward an area based on that
area's metabolic needs
D. Distributive vessels; sense oxygen levels and increase heart rate when the brainstem detects dropping O2
levels or rising CO2 levels

9. You are treating a 42-year-old female exhibiting signs of shock. You do not suspect a spinal injury. To try to
improve blood flow to the heart, you should position the patient:
A. supine with the bed tilted so that her head is lower than her feet.
B. supine with her feet elevated 8-12 inches above the level of the heart.
C. flat on her left side.
D. with her head slightly elevated and her arms raised over her head.

10. You get a call for a "serious" injury on the expert trail. As you are skiing up to the patient your first priority is:
A. Performing a primary survey
B. Trying to determine how many people may be injured
C. Assessing the scene for rescuer safety
D. Looking to see how much blood is on the snow

11. Which of the following statements made by a patient's family member would lead you to suspect that the
patient is suffering from hypovolemic shock?
A. "He got up this morning and was having hard time breathing.”
B. "He has had a rash for the past three days.”
C. "He cannot stop throwing up.”
D. "He has been taking an antibiotic for a chest cold."

12. You are caring for a 55-year-old man with significant trauma. His medical history reveals high blood pressure
for which he takes a beta blocker. You realize that this group of drugs may:
A. Cause him to have higher-than-normal blood pressure.
B. Inhibit his heart's ability to beat faster.
C. Cause his heart rate to increase.
D. Diminish his reactions to pain.

13. Which of the following is not a characteristic of the heart?


A. It pumps blood through arteries to the tissues of the body.
B. It has four chambers and two sides.
C. It pumps oxygen-depleted blood to the lungs through the pulmonary veins.
D. It is about the size of a clenched fist.

C HAPTER 10
14. A patient who sustained blunt trauma to the abdominal and pelvic areas in a very serious collision is probably
in which type of shock?
A. Obstructive
B. Distributive
C. Cardiogenic
D. Hypovolemic

15. Shock is defined as:


A. Adequate blood pressure and multiple organ failure.
B. A decrease in the amount of circulating blood for any reason.
C. A state of inadequate tissue perfusion.
D. Adequate blood pressure but no pulse.

16. Shock can best be described as:


A. The amount of oxygen reaching the cells is adequate, but carbon dioxide is not being removed.
B. The blood contains an adequate amount of oxygen but not enough nutrients for cells to survive
C. The cells are getting glucose and other nutrients but not oxygen.
D. Inadequate tissue perfusion in which cells do not receive sufficient amounts of oxygen and nutrients.

17. Which of the following statements about irreversible shock is correct?


A. If the pupils are dilated, the patient is in irreversible shock.
B. To survive, the OEC technician should give the patient fluids and get the patient to the hospital quickly.
C. In irreversible shock, the radial pulse is weak but the carotid pulse stays strong.
D. Even with treatment, death will result.

18. When performing a primary assessment, one of the first signs that the body may be in shock is:
A. Tachycardia
B. Hypertension
C. A slowed respiratory rate
D. Skin that is warm and diaphoretic

19. Which of the following statements best indicates that the speaker understands the role of caring for a patient
in shock in a prehospital setting?
A. "If shock is in the compensatory or early stage, it is not yet life threatening, so you can take your time to
carefully assess and treat the patient.”
B. "Because shock is a life-threatening condition, it is important to identify its exact cause so that the OEC
technician can administer the proper care to correct it.”
C. "Because shock is best treated in the hospital, one should provide care to correct problems affecting
ABCDs and then rapidly transport the patient.”
D. "The job of the prehospital provider is not to treat the patient for shock and get the patient to the hospital,
where treatment for shock can be started."

20. The four major types of shock are:


A. Hemorrhagic, distributive, hypoxic, and obstructive.
B. Hypovolemic, cardiogenic, obstructive, and distributive.
C. Hypoglycemic, obstructive, distributive and hypovolemic.
D. Burn, hypovolemic, distributive, and hypoxic.

C HAPTER 10

21. Which of the following factors must be kept in mind when considering the severity of external bleeding?
A. Signs of shock do not appear until a large amount of blood is lost.
B. A person in poor health cannot tolerate the same amount of blood loss as a healthier person.
C. The amount of blood loss is easily estimated by the amount of blood visible on the snow.
D. Children show signs of shock much more quickly than do adults.

22. Which of the following statements correctly defines cardiac output?


A. It is the volume of blood that is pumped in one minute.
B. Reduced stroke volume always means reduced cardiac output.
C. It is the amount of blood pumped in each heartbeat.
D. It is the number of heartbeats per minute.

23. You are treating a 42-year-old female exhibiting signs of shock. You do not suspect a spinal injury. To try to
improve blood flow to the heart, you should position the patient:
A. with her head slightly elevated and her arms raised over her head.
B. flat on her left side.
C. Supine with her feet elevated 8–12 inches above the level of the heart.
D. supine with the bed tilted so that her head is higher than her feet.

24. The key components of blood are:


A. Red blood cells, platelets, hemoglobin, and glycerin.
B. Red blood cells, white blood cells, platelets, and plasma
C. Hemoglobin, neuroses, platelets, and plasma.
D. White blood cells, plasma, proteins, and fat

25. A patient in early onset of shock informs you that he has had severe diarrhea and vomiting over the past four
days. Given this history, you would recognize the pathophysiology of the shock is probably related to:
A. Stoppage of the capillaries
B. Decreased formed elements in the blood
C. The loss of plasma volume
D. The loss of red blood cells

26. In decompensated shock, the failure of body systems despite the body's attempt to oxygenate vital organs
becomes apparent as:
A. Augmented capillary refill and increased respiratory rate.
B. Increased blood pressure and reduced pulse rate.
C. Increased respiratory rate and reduced level of consciousness.
D. Increased pulse rate and decreased blood pressure.

27. A patient with severe gastrointestinal bleeding is in what type of shock?


A. Hypovolemic
B. Hemorrhagic
C. Cardiogenic
D. Distributive

28. The cardiovascular system is composed of the:


A. Heart, blood vessels, and blood

C HAPTER 10
B. Heart, aorta, alveoli and venae calvae
C. Blood vessels, plasma, and the lungs
D. Heart, lungs, and blood

29. You approach a patient and notice a large amount of blood on the ground. The patient appears confused, pale,
and diaphoretic. Which one of the following should you do first?
A. Treat the patient for shock
B. Apply oxygen
C. Ensure that the patient's airway is open and clear
D. Look for the source of the blood

30. A patient in shock with abdominal pain indicates he noticed lots of blood in the toilet after having a bowel
movement this morning. You would recognize the possibility of what type of shock?
A. Obstructive
B. Hypovolemic
C. Hypoxic
D. Septic

31. Septic shock is caused by:


A. Blood loss
B. A collapsed lung
C. An infection
D. A failing heart

32. Which of the following conditions could be responsible for causing obstructive shock?
A. Arrhythmia
B. Hemorrhage
C. Pulmonary Embolism
D. Sepsis

33. You are reassessing a patient who has sustained blunt trauma to the chest. Which one of the following
reassessment findings best indicates that the patient is deteriorating and is in the decompensating phase of
shock?
A. Strong pulse of 96 beats per minute
B. Skin that is cool and dry
C. Blood pressure of 88/50
D. Blood oozing from an abdominal laceration

Answer Key: Chapter 10 - Test


1=B, 2=B, 3=C, 4=B, 5=B, 6=D, 7=B, 8=C, 9=B, 10=C, 11=C, 12=B, 13=C, 14=D, 15=C, 16=D, 17=D,
18=A, 19=C, 20=B, 21=B, 22=A, 23=C, 24=B, 25=C, 26=D, 27=B, 28=A, 29=C, 30=B, 31=C, 32=C, 33=C


C HAPTER 10
C H A P T E R 11
“Altered Mental Status”

QUIZ

1. The family of a 75-year-old man has called you to evaluate the patient's slurred speech and right arm weakness.
When you arrive 10 minutes later, you find that the symptoms have completely resolved. What should you do
next?
A. Obtain a detailed medical history including medications
B. Strongly advise the patient to seek urgent medical attention in a hospital setting
C. Reassure the patient and family because the symptoms are now gone
D. Accurately determine the time of onset of the symptoms

2. You have arrived on scene to care for an elderly woman who has fallen on the ice. She is now unresponsive. Her
pupils are unequal, there is a palpable radial pulse at 74, and respirations are 14 with some grunting sounds with
inspiration. What is your first action?
A. See if she is wearing a medical alert tag
B. Ask bystanders if she fell over and hit her head
C. Apply in-line cervical traction to stabilize the spine
D. Perform a jaw-thrust maneuver

3. A commonly used position in which to transport a postictal patient following a seizure is:
A. Prone position.
B. Trendelenburg position.
C. Rothberg's position.
D. Lateral recumbent position.

4. Multiple family members are frantically and simultaneously telling you everything they think might be helpful
as you evaluate your patient for unresponsiveness. Of the following, which nugget of history carries the most
import?
A. He was acting oddly and was confused last night at 11 P.M. and then went to bed
B. He missed his antihypertensive medication this morning
C. He did not take his blood thinner last night
D. He is allergic to aspirin

5. What is a commonly used technique to evaluate a suspected stroke patient for abnormal speech?
A. Ask family members if the patient's speech pattern is normal for him or her
B. Write down words on a card and ask the patient to read them back
C. Ask the patient to read names from the phone book
D. Ask the patient to repeat a common phrase

C H A P T E R 11
Answer Key: Chapter 11 - Quiz
1=B, 2=D, 3=D, 4=A, 5=D

TEST

1. Which of the following actions may result in hypoglycemia in a diabetes patient?


A. Missing a meal
B. Overeating
C. Failure to take insulin or an oral diabetes medication
D. Lack of exercise

2. You have been called for a seizure emergency. You find an adult female actively seizing with a bystander
attempting to place a spoon between her teeth. The bystander tells you that he is trying to keep the person from
biting her tongue. Which one of the following would be an appropriate response?
A. Instruct the bystander to restrain the patient while you put the spoon in place.
B. Tell the bystander that a padded tongue blade works better.
C. Tell the bystander to continue trying to get the spoon in place.
D. Ask the bystander to discontinue his efforts and then turn the patient onto her side to facilitate the
E. drainage of oral secretions.

3. Hyperglycemia:
A. Has a more rapid onset than hypoglycemia.
B. Has a more gradual onset than hypoglycemia.
C. Is preceded by an aura, such as hallucinations or detecting unusual odors.
D. Is more easily treated in prehospital environments than is hypoglycemia.

4. Insulin:
A. Increases the level of glucose circulating in the blood.
B. Enables transfer of glucose from blood into cells
C. Increases the transfer of sugar from the stomach and small intestine to the bloodstream.
D. Makes glucosamine work better as an energy source

5. As a general rule, a seizing patient should be transported in what position?


A. Supine
B. On their side
C. Sitting upright
D. Supine with legs elevated

6. In the mnemonic AEIOU-TIPS, the T stands for:


A. Trauma and tumors
B. Thrombophlebitis and tinnitus
C. Thyroiditis and thrombophlebitis
D. Trauma and thrombophlebitis

C H A P T E R 11
7. The part of the brain that controls balance and coordination is the:
A. thalamus.
B. cerebellum.
C. cerebrum.
D. brain stem.

8. In the mnemonic AEIOU-TIPS, the S stands for:


A. Sjogren's syndrome, seizure, and sinusitis
B. Seizure, stroke, and syncope
C. Sickle cell anemia, Sjogren's syndrome, and steroids
D. Steroids, seizure, and sinusitis

9. Glucose:
A. allows the body to use insulin.
B. is an essential building block for body tissues such as muscle and bone.
C. is an energy source for brain cells and other cells in the body.
D. assists the pancreas in the manufacture of insulin.

10. Which of the following statements indicates that the speaker has an accurate understanding of altered mental
status (AMS)?
A. "AMS occurs only when a patient is not responsive to verbal or painful stimuli."
B. "AMS describes any abnormal change in a person's level of awareness."
C. "AMS should be suspected only when a patient is on medications that can cause confusion."
D. "AMS is any mental state in a person with diabetes."

11. When assessing a patient with a possible stroke, the priority of care is:
A. identifying risk factors for stroke.
B. recognizing the signs of a stroke.
C. determining the type of stroke.
D. determining if there is a family history of stroke.

12. our assessment reveals a drooling 48-year-old male to have gurgling speech and left arm paralysis. Which of
the following would be your priority?
A. Monitor the patient's blood pressure
B. Suction the patient's airway
C. Protect the patient's left arm
D. Determine the patient's risk factors for stroke

13. Your patient is unresponsive and there is no evidence of trauma. Which piece of information given to you by
the patient's family is most important in relation to the treatment of a possible stroke?
A. "We think he had too much to drink last night.”
B. "He took some ibuprofen last night for a fever and cough.”
C. "We noticed him acting oddly about 7 p.m. last night.”
D. "He forgot to take his blood thinner last night."

14. Which of the following questions is critical for establishing a window of treatment for a patient who is weak
and dizzy and may have suffered a stroke?
A. "What medications do you take?"

C H A P T E R 11
B. "Does your father have a history of heart problems or stroke?"
C. "Do you have any pain in your arms or legs?"
D. "What time did the weakness and dizziness start?"

15. Which of the following statements indicates that the speaker understands the danger posed by status
epilepticus?

A. "Patients who do not have a history of seizures are at greater risk for status epilepticus.”
B. "Status epilepticus is an extremely dangerous condition because the patient can go into shock from blood
loss.”
C. "Status epilepticus indicates that the patient's medications have reached toxic levels in the body.”
D. "The longer the seizure continues, the greater the likelihood of permanent brain damage."

16. The signs and symptoms of a transient ischemic attack (TIA):


A. Are severe and permanent.
B. Are mild but permanent.
C. Are temporary and resolve within 24 hours.
D. Can persist for a few days before they resolve.

17. In the mnemonic AEIOU-TIPS, the U stands for:


A. Urticarial
B. Usher syndrome
C. Ulcers
D. Uremia (kidney failure)

18. Which of the following statements indicates that the speaker understands hemorrhagic strokes?
A. "They occur when a blood vessel in the brain ruptures.”
B. "Their signs and symptoms generally resolve within 24 hours.”
C. "They can sometimes be treated by clot-destroying medications.”
D. "They occur when clots form in blood vessels in the brain."

19. When performing a secondary assessment on a confused patient, which of the following signs is most
suggestive of a seizure?
A. Pinpoint pupils
B. Right sided facial droop
C. A slow heart rate
D. A bitten tongue

20. Glucose:
A. Assists the pancreas in the manufacture of insulin.
B. Allows the body to use insulin.
C. Is an essential building block for body tissues such as muscle and bone?
D. Is an energy source for brain cells and other cells in the body?

21. In the mnemonic AEIOU-TIPS, the O stands for:


A. Overdose and otitis
B. Oxygen and overdose
C. Osteosarcoma and oxygen

C H A P T E R 11
D. Overdose and oliguria

22. Which of the following instructions to a possible stroke patient describes the proper procedure for assessing
an arm drift?
A. "Hold your arms straight out in front of you with your palms facing up with your eyes closed.”
B. "Grab my fingers with both of your hands and squeeze as hard as you can.”
C. "Raise your hands above your head and close your eyes.”
D. "Hold your arms up over your head for 20 seconds."

23. After a delayed response, you arrive in the lodge and find a 62-year-old female is still seizing. Which one of
the following should you do first?
A. Ascertain the duration of the seizure.
B. Assess her airway and breathing.
C. Determine if she has a seizure history.
D. Move her to the aid room for further assessment and then transport her to the hospital.

24. When assessing a patient for a possible stroke patient, you should NOT:
A. see if one side of the patient's mouth is drooping.
B. ask the patient to repeat a simple phrase.
C. ask the patient stick his tongue straight out.
D. write questions on note cards for the patient to read.

25. You have been called to the lodge for a behavioral emergency. When you arrive, you find a male in his forties
sitting up against a wall. He is confused and incontinent. Bystanders state that he suddenly fell to the ground and
was grunting and shaking. From this description what should you suspect first that the man has had?
A. A syncopal episode
B. An absence seizure
C. A generalized seizure
D. A diabetic reaction

26. You have assessed a patient and found a patent airway, adequate breathing, and a strong radial pulse. The
patient is having no difficulty speaking and no facial droop, but exhibits a slight left arm drift. You should
interpret these findings as:
A. Suggestive that a stroke may be occurring.
B. Suggestive of a stroke only if the patient's pupils are not equal.
C. Suggestive of a stroke only if the patient's blood pressure is also elevated.
D. Not suggestive of a stroke.

27. tatus epilepticus is best differentiated from a generalized seizure by:


A. The presence of an aura prior to seizing.
B. The length of the postictal period.
C. A pre-existing history of seizures.
D. The duration of the seizure.

28. Hyperglycemia:
A. has a more rapid onset than hypoglycemia.
B. has a more gradual onset than hypoglycemia.
C. is more easily treated in prehospital environments than is hypoglycemia.

C H A P T E R 11
D. is preceded by an aura, such as hallucinations or detecting unusual odors.

29. Which of the following events best describes an ischemic stroke?


A. Blood has collected throughout the brain tissue.
B. A heart attack has occurred at the same time as a stroke.
C. A blood clot has obstructed a blood vessel in the brain.
D. An artery in the brain has ruptured.

30. The benefit of the Cincinnati Prehospital Stroke Scale is that it:
A. Enables you to identify the type of stroke a patient is having.
B. Enables you to screen patients at risk for a future stroke.
C. Helps you identify the probability that your patient is having a stroke.
D. Helps you predict the likelihood a patient will die from a stroke.

31. The primary problem in Type II diabetes is that:


A. sugars cannot be easily digested in the stomach and small intestine.
B. cells exhibit resistance to insulin.
C. too much insulin is produced.
D. insufficient insulin is produced.

32. The primary problem in Type II diabetes is that:


A. Too much insulin is produced.
B. Sugars cannot be easily digested in the stomach and small intestine.
C. cells exhibit resistance to insulin preventing glucose from entering the cells.
D. Insufficient insulin is produced.

33. Which of the following statements concerning the causation of seizures is correct?
A. A seizure condition is a muscle problem that causes the arms and legs to jerk.
B. A seizure occurs when there is an electrical disturbance in the brain.
C. All seizures are caused by epilepsy, a term that describes a problem somewhere in the body.
D. A seizure occurs when the heart beats irregularly, causing a decreased amount of oxygen-rich blood to
reach the brain.

34. You are called to the lodge to assess a 3-year-old boy. The parents state that their son was playing with his
brother and "blanked out" and "stared off into space" for several seconds. They deny any convulsing-like
movement. Based on this description, you would suspect what type of seizure?
A. An absence (petit mal) seizure
B. A simple partial seizure
C. A grand mal seizure
D. A febrile seizure

35. Insulin:
A. increases the transfer of sugar from the stomach and small intestine to the bloodstream.
B. increases the level of glucose circulating in the blood.
C. makes glucosamine work better as an energy source.
D. enables transfer of glucose from blood into cells.

C H A P T E R 11
36. You have been called for a 63-year-old woman with slurred speech and right arm weakness. On arrival, the
patient informs you that the slurred speech and weakness have resolved. Since your assessment reveals no
deficits, you would tell the patient:
A. "Sometimes seizures present this way; call your doctor in the morning to schedule an appointment.”
B. "You really need to be evaluated in the hospital. You could be having a stroke.”
C. "It would be best to start taking one baby aspirin every day; this will decrease the chance of this happening
again.”
D. "If this happens again, wait 10 minutes to see if you are okay. If the symptoms don't go away, call 911."

37. A young boy tells you that his twin brother suffers from seizures and asks what he can do if he sees his
brother convulsing. You should give him which one of the following instructions?
A. "Move any moveable objects and furniture away from him.”
B. "Call EMS only if the seizure lasts more than 10 minutes or he loses control of his bladder." “When he quit
seizing, give him sugar, so his blood sugar is OK.”
C. "Insert a spoon into his mouth to keep him from swallowing his tongue.”
D. "Hold him firmly against the floor until he stops seizing or until the EMTs arrive."

38. In the mnemonic AEIOU-TIPS, the first and second I stands for:
A. Insulin and infection
B. Insomnia and intestine
C. Interstitial cystitis and iodine
D. Impetigo and injury

39. In the mnemonic AEIOU-TIPS, the A stands for:


A. Acidosis and anemia
B. Alcohol and acidosis
C. Abscess and acidosis
D. ADHD and alcohol

40. Which of the following pieces of information is assessed by the Cincinnati Prehospital Stroke Scale?
A. Presence or absence of facial droop
B. Duration of symptoms
C. Blood glucose level
D. Age of the patient

41. The part of the brain that controls basic functions necessary for life is the:
A. thalamus.
B. cerebrum.
C. cerebellum.
D. brain stem.

42. Which of the following actions may result in hypoglycemia in a diabetes patient?
A. Lack of exercise
B. Overeating
C. Failure to take insulin or an oral diabetes medication
D. Missing a meal when the patient has taken their insulin

43. In the mnemonic AEIOU-TIPS, the E stands for:

C H A P T E R 11
A. Electrolyte imbalance, environment, and echovirus
B. Epilepsy, echovirus, and environment
C. Epilepsy, environment, and electrolyte imbalance
D. Edema, electrolyte imbalance, and endocarditis

44. Which of the following statements shows that the speaker has an accurate understanding of assessment
findings related to stroke?
A. "A stroke should be suspected only when one arm is paralyzed, not just weak."
B. "For a stroke to be suspected, both arms must be equally weak."
C. "If one arm is weak, stroke should be suspected only if the patient's blood pressure is also elevated."
D. "If one arm is weak, a stroke should be suspected."

45. Some patients experience a premonition indicating a seizure is about to happen known as a (n):
A. Syncopal stage
B. Aura
C. Colonic phase
D. Postictal state

46. In the mnemonic AEIOU-TIPS, the P stands for:


A. Poisoning and psychiatric conditions.
B. Parathyroid disorders and psychiatric conditions.
C. Parkinson's disease and peripheral nerve disorder.
D. Parathyroid disorders and Parkinson's disease.

Answer Key: Chapter 11 - Test


1=A, 2=D, 3=B, 4=B, 5=B, 6=A, 7=B, 8=B, 9=C, 10=B, 11=B, 12=B, 13=C, 14=D, 15=D, 16=C, 17=D,
18=A, 19=D, 20=D, 21=B, 22=A, 23=B, 24=D, 25=C, 26=A, 27=D, 28=B, 29=C, 30=C, 31=B, 32=C,
33=B, 34=A, 35=D, 36=B, 37=A, 38=A, 39=B, 40=A, 41=D, 42=D, 43=C, 44=D, 45=B, 46=A


C H A P T E R 11
CHAPTER 12
“Substance Abuse and Poisoning”

QUIZ

1. The most important principle in the management of a suspected poisoning scene is:
A. To notify law enforcement prior to entering the scene.
B. To determine which substance is responsible for the patient's signs and symptoms.
C. To collect all medication and substance containers for subsequent toxicology tests.
D. Personal and team safety.

2. A Tylenol overdose primarily affects the:


A. Brain.
B. Kidneys.
C. Liver.
D. Pancreas.

53. What is the most expedient way to obtain assistance from the nearest poison control center?
A. Contact medical control on the radio
B. Go to CHEMTREC on the web
C. Dial 1-800-222-1222 or 9-1-1
D. Refer to your pocket guide

4. Administration of activated charcoal is useful in the management of ingested (swallowed) poisons because:
A. The time since the ingestion of the poison does not matter.
B. There is no risk to the respiratory system.
C. It binds many toxins to the charcoal, which is then eliminated in the stool.
D. It induces vomiting to expel the poison.

5. Signs of opiate overdose can include all of the following EXCEPT:


A. Slow and shallow respirations.
B. Hypertension.
C. Pupil constriction.
D. A decreased level of responsiveness.

Answer Key: Chapter 12 - Quiz


1=D, 2=C, 3=C, 4=C, 5=B

TEST

C H A P T E R 12
1. You are visiting a friend's home and suddenly hear a child crying out in the kitchen. As you assess the scene
you note kitchen cabinets are open with bottles tossed around. The child is holding her hand to her mouth. Your
friend tells you that the child must have tasted something that she didn't like. Based on your scene assessment,
you would:
A. Ask your friend if she has Ipecac so you can make the child vomit.
B. Call 911 and try to identify which bottle the child may have been drinking from.
C. Agree with your friend and suggest that a cookie would take the bad taste away.
D. Offer the child a glass of milk to dilute any chemical he may have ingested.

2. It is late Friday evening and you are joining your friends at a cabin for a winter weekend trip. When you open
the door you notice how nice and warm it is inside from the wood stove. Your friends though are not very
welcoming. They tell you they are not feeling well and complain of being lightheaded and nauseated. Based on
their symptoms, you are concerned they may be suffering from:
A. Alcohol toxicity
B. Carbon monoxide poisoning
C. Excessive heat exposure
D. Carbon dioxide poisoning

3. A poisonous substance that comes from a living organism is known as a(n):


A. Chemical
B. Toxin
C. Antipsychotic
D. Legume

4. Distribution of a poisonous substance in the body occurs primarily through the:


A. Gastrointestinal system
B. Endocrine system
C. Circulatory system
D. Lymphatic system

5. You are driving on the highway and see an overturned tanker. A fluid is spilling from the tanker rapidly
covering the ground around the driver's door. The driver appears to be unresponsive. The only marking on the
tanker seems to be the name of the tanker company and hazard symbol. Your first step should be:
A. Secure the scene, you must ensure your safety...
B. Cover your mouth and nose so that you don't breathe any fumes while you rescue the driver.
C. Have someone call 911 and assess the driver's ABCDs.
D. Try not to step in the liquid while pulling the driver away from the truck.

6. For children, the three deadliest poisons are:


A. adult prescriptions, alcohol, and daily vitamins.
B. carbon monoxide, toxic plants, and aspirin.
C. household cleaners, aspirin, and adult prescriptions.
D. aspirin, acetaminophen, and iron.

7. Which of the following terms does not indicate a route of absorption in the body?
A. Transdermal
B. Inhalation
C. Distribution

C H A P T E R 12
D. Ingestion

8. The speed at which a toxin is distributed in the body is affected by:


A. The respiratory rate
B. The heart rate
C. Urination
D. Body mass

9. Which of the following parts of the body is least often affected by substances?
A. The eyes
B. The blood
C. The lymphatic vessels
D. The heart

10. The process by which the body breaks down a substance is known as:
A. Detoxification
B. Elimination
C. Metabolism
D. Distribution

11. In the United States, poison-related information is available 24 hours a day by contacting the:
A. Substance Abuse and Mental Health Hotline at1-800-666-6666.
B. National Poison Center at 1-800-222-1222.
C. Chemical Emergency Center at 1-800-262-8200.
D. National Institute on Drug Abuse at 1-800-222-HELP.

12. You are helping a friend change the battery in his truck when acid from the old battery splashes in his eye. He
is complaining of pain in his right eye and is having a hard time keeping it open. He has no problems with airway,
breathing, or circulation. What should you do?
A. Pat the eye with a dry towel to absorb the acid.
B. Place an ice pack on his eye and immediately transport him to the nearest Emergency Department.
C. Immediately irrigate the eye with copious amounts of clean water.
D. Cover both his eyes and immediately drive him to the nearest Emergency Department.

13. As a whole, the least toxic of all accidental ingestion-related poisonings involve:
A. alcoholic beverages.
B. acetaminophen.
C. aspirin.
D. toxic plants.

14. The mnemonic DUMBELS and the acronym SLUDGE can help identify specific signs and symptoms
associated with:
A. ingestion of LSD.
B. overdoses of antipsychotic drugs.
C. ingestion of ethylene glycol.
D. exposure to nerve agents.

C H A P T E R 12
15. The mnemonic DUMBELS and the acronym SLUDGE can help identify specific signs and symptoms
associated with:
A. Exposure to nerve agents
B. Ingestion of LSD
C. Ingestion of ethylene glycol
D. Overdoses of antipsychotic drugs

16. In the United States, poison-related information is available 24 hours a day by contacting the:
A. Chemical Emergency Center
B. National Institute on Drug Abuse
C. National Poison Center
D. Substance Abuse and Mental Health Hotline

17. Substance abuse is defined as:


A. The use of a substance or poison at least once a day.
B. An intentional or unintentional use of a substance or poison that results in a medical emergency.
C. The intentional misuse of a substance that results in significant impairment.
D. An accidental ingestion of a toxin that interferes with normal functioning.

18. Which of the following statements indicates that the speaker understands inhalation poisonings?
A. "Most inhalation poisonings are easily identifiable by a residue around the mouth."
B. "They can occur from inhaling common household products."
C. "They affect the lungs only and do not circulate systemically."
D. "An inhalation emergency cannot be occurring if you don't smell anything."

19. A patient in the third day of withdrawal from alcohol could develop all of the following conditions except:
A. Rash
B. Muscle tremors
C. Seizure activity
D. Hallucinations

20. The most commonly abused drug in the world is:


A. cocaine.
B. amphetamine.
C. marijuana.
D. alcohol.

21. The most commonly abused drug in the world is:


A. Marijuana
B. Alcohol
C. Amphetamine
D. Cocaine

22. A 15-year-old admits to taking "a bunch" of extra strength Tylenol tablets. Her mother states it's nothing to
worry about because it was just Tylenol. Based on your training you recognize that this ingestion could have
serious consequences. It is because toxic amounts acetaminophen can cause irreversible:
A. Liver damage
B. Hearing damage

C H A P T E R 12
C. Brain damage
D. Stomach damage

23. Agitation, elevated heart rate, and elevated blood pressure are signs of intoxication with which of the
following types of substances?
A. Inhalants and sleeping pills
B. Antipsychotics and carbon monoxide
C. Cocaine and hallucinogens
D. Alcohol and ethylene glycol

24. A 44-year-old patient was found at home unresponsive. Based on his clinical presentation and the fact that he
was heating with a kerosene heater in an enclosed room, you suspect that he is suffering from carbon monoxide
poisoning. Based on this information, which route of exposure would you recognize as responsible for the
poisoning?
A. Inhalation
B. Transdermal route
C. Absorption
D. Ingestion

25. You are teaching a class about drug-related emergencies, specifically opiate overdose. Which of the following
statements made by students indicates an understanding of possible signs of opiate overdose?
A. "You should always ask patients if they have pain because narcotics suppress pain."
B. "Only illegal narcotics can cause hallucinations, which may make the patient violent."
C. "When assessing a patient, you must be alert for respiratory depression."
D. "You should always check the patient's pupils because they will be dilated in a narcotics overdose."

26. Which of the following statements regarding poisonings is true?


A. The number-one cause of poisoning in the United States is from taking too much.
B. Majorities of poisonings are intentional and result in homicide or suicide.
C. A toxin is poison produced by minerals.
D. A poison is any substance that causes harmful effects when introduced into the body.

Answer Key: Chapter 12 - Test


1=B, 2=B, 3=B, 4=C, 5=A, 6=D, 7=C, 8=B, 9=C, 10=C, 11=B, 12=C, 13=D, 14=D, 15=A, 16=C, 17=C,
18=B, 19=A, 20=D, 21=B, 22=A, 23=C, 24=A, 25=C, 26=D

C H A P T E R 12
C H A P T E R 13
“Respiratory Emergencies”

QUIZ

1. The term dyspnea refers to:


A. Rapid breathing.
B. Respiratory failure.
C. Inadequate oxygen in the bloodstream.
D. Shortness of breath.

2. Respiratory rate and depth of respiration are controlled by:


A. The amount of oxygen in the bloodstream.
B. Conscious effort.
C. The degree of tissue hypoxia.
D. The amount of carbon dioxide in the bloodstream.

3. A 34-year-old patient in the lodge is sitting bolt upright, eyes wide, with labored breathing at a rate of 28 per
minute. Your initial action is to:
A. Immediately place the patient in the lateral recumbent position.
B. Provide supplemental oxygen at 15 lpm via an NRB.
C. Check for medical alert tags.
D. Attempt to get the patient in a supine position.

4. The term tachypnea refers to:


A. Irregular breathing.
B. Rapid breathing.
C. Periodic breathing.
D. Shallow breathing.

5. Accessory muscles of respiration include those of the chest, shoulders, and abdomen. They assist ventilations
by:
A. Pushing in on the chest wall to assist forced exhalation of carbon dioxide.
B. Allowing the diaphragm to rest.
C. Raising intrathoracic pressure in order to force air into the lungs.
D. Helping to further expand the chest wall in order to pull more air into the lungs.

Answer Key: Chapter 13 - Quiz


1=D, 2=D, 3=B, 4=B, 5=D

C H A P T E R 13
TEST

1. Hyperventilation syndrome is a common psychological condition that:


A. Results in abnormally low blood carbon dioxide levels.
B. Is characterized by shallow, irregular, rapid breaths.
C. Is often precipitated by exercise.
D. Is always benign and resolves with no treatment.

2. Which of the following actions causes an individual to inhale?


A. Pressure within the chest increases
B. The intercostal muscles relax
C. The chest cavity decreases in size
D. The diaphragm contracts and flattens

3. You and your OEC candidate are evaluating a 67-year-old patient with respiratory distress. You tell the
candidate to document that the patient complains of dyspnea. The candidate asks you what "dyspnea" means. You
explain that dyspnea is:
A. A term that describes a patient who is in respiratory distress.
B. a subjective term that means difficulty breathing
C. A term that describes your objective assessment of a patient's difficulty in breathing.
D. Another term for tachypnea.

4. The most important muscle of respiration is the:


A. Diaphragm
B. Intercostal muscle
C. Pectoralis major
D. Sternocleidomastoid

5. Which of the following conditions is responsible for a finding of wheezing accompanying shortness of breath?
A. Swelling in the throat
B. Significant hypoxia
C. Bronchiole constriction
D. Mucus in the lungs

6. You receive a call that there is an infant in the lodge who seems to be having difficulty breathing. As you are
approaching the scene with a new candidate patroller, you ask him what the normal respiratory rate is for an
infant. The candidate would be correct if he told you the rate should be:
A. 15-20 breaths per minute
B. 15-40 breaths per minute
C. 20-30 breaths per minute
D. 25-50 breaths per minute

7. During a scene size-up, which of the following observations most strongly suggests that your adult patient is
suffering from an acute respiratory emergency?
A. The patient's hands are trembling.

C H A P T E R 13
B. The patient is in the tripod position.
C. The patient's respiratory rate is 20.
D. The patient is holding a metered-dose inhaler.

8. A tachypneic patient is breathing:


A. More slowly than normal
B. Irregularly
C. Normally
D. More rapidly than normal

9. Which of the following lists best represents the correct sequence for the passage of air into the lungs once it
passes the pharynx?
A. Epiglottis, esophagus, trachea, alveoli
B. Larynx, trachea, bronchi, alveoli
C. Bronchi, larynx, trachea, cricoid
D. Trachea, uvula, bronchi, alveoli

10. You are documenting your assessment of the patient who just left in an ambulance. The patient told you that
he felt short of breath. You would document this as:
A. Anoxia
B. Arrhythmia
C. Dyspnea
D. Respiratory failure

11. You are assessing a 24-year-old skier who is anxious and complaining of shortness of breath. She tells you
that this sometimes happens when she is exercising. Her respirations are rapid and shallow. Based on this
information, you suspect the patient is suffering from:
A. crepitus.
B. pneumothorax.
C. hyperventilation syndrome.
D. asthma.

12. Which of the following statements about respiratory accessory muscles is NOT correct?
A. They are recruited whenever the body's oxygen demand exceeds oxygen availability.
B. They bring more oxygen into the body by helping the chest wall expand more fully.
C. They include chest, shoulder, and abdominal muscles.
D. They decrease negative internal chest pressure to draw more air into the lungs.
E.
13. In an adult, the most common cause of airway obstruction is:
A. A food bolus
B. An inflamed pharynx
C. The tongue
D. Mucous secretions

14. Which of the following respiratory rates may signify significant respiratory issues in an adult?
A. More than 20 respirations per minute
B. Fewer than 8 respirations per minute
C. More than respirations 24 per minute

C H A P T E R 13
D. Fewer than 12 respirations per minute

15. You are called to aid a 65-year-old man who is having difficulty breathing. When you arrive, you find that he is
sitting upright in a chair and is in obvious respiratory distress. His airway is open and his respirations are 28 per
minute. At this time you would:
A. Try to obtain a medical history from him
B. Assess his breath sounds
C. Assist him to a supine position
D. Provide him oxygen with a nonrebreather mask at 15 liters per minute

16. You are assessing a 24-year-old skier who is anxious, wheezing, and complaining of shortness of breath. She
tells you that this sometimes happens when she is exercising. Her respirations are rapid and shallow. Based on
this information, you suspect the patient is suffering from:
A. Asthma
B. Crepitus
C. Hyperventilation syndrome
D. Pneumothorax

17. Several abnormal lung sounds can be helpful in understanding what condition may be affecting a patient. One
such sound is wheezing. Which of the following statements about wheezing is false?
A. It may be heard on inhalation, exhalation, or both.
B. It is typically caused by asthma.
C. It indicates constriction of the lower airway passages.
D. It can be heard only with a stethoscope.

18. Which of the following signs would you recognize as an early sign of respiratory distress in a 7-year-old boy
who complains of difficulty breathing?
A. Ecchymosis on his chest
B. Nasal flaring
C. Tremors of his hands
D. Barrel chest

19. The normal range for respiratory rate in children is:


A. 25-40 breaths per minute
B. 40-45 breaths per minute
C. 5-10 breaths per minute
D. 15-20 breaths per minute

20. A 74-year-old male with a history of chronic emphysema is complaining of sudden onset of shortness of
breath. He appears to be in acute respiratory distress. Physical exam of the anterior chest wall reveals crepitus.
Based on the patient's history and physical exam, you would suspect which of the following conditions?
A. Acute bronchitis
B. Congestive heart failure
C. Carbon monoxide poisoning
D. Spontaneous pneumothorax

C H A P T E R 13
21. You are called to assist a 16-year-old female who is reportedly having trouble breathing. Your assessment
reveals an anxious female with rapid respirations who is complaining of pins and needles around her lips. Based
on your training you suspect that:
A. these findings will disappear if she slows her rate of breathing.
B. having her lie down with her feet elevated 8-12 inches will relieve the pins and nee
C. she is having an allergic reaction and should be treated with an epi-
D. she is close to respiratory failure.

22. You are in the cafeteria and suddenly hear someone calling for help because her friend is choking. Which of
the following signs would tell you that you need to intervene immediately?
A. The patient appears frightened
B. The patient cannot speak
C. The patient says she has a bone stuck in her throat
D. The patient was eating steak

23. Involuntary breathing is controlled by:


A. Tissue hypoxia.
B. A conscious effort of inspiration.
C. The amount of carbon dioxide dissolved in the blood.
D. The amount of oxygen dissolved in the blood.

24. 71-year-old woman is complaining of the sudden onset of shortness of breath and also some chest pain during
deep breaths. She is cyanotic and appears somewhat panicked. Her respirations are rapid and deep. Her husband
tells you that she had knee surgery about one week ago. Based on your assessment and the patient's history, you
suspect that the patient is suffering from:
A. An anxiety attack
B. COPD
C. A pulmonary embolus
D. An asthma attack

25. You are having a hard time getting the medical history of a patient who is short of breath. Which one of the
following pieces of information would lead you to believe that the patient has a history of a chronic lung disease
such as chronic emphysema?
A. She has a frequent cough
B. Her respirations are tachypneic
C. She takes one aspirin every day
D. Her chest is barrel shaped

26. A 30-year-old male arrives in the aid room complaining of shortness of breath. Which of the following actions
would you perform first?
A. Ask him if he has an inhaler
B. Determine the cause of his shortness of breath
C. Assess the adequacy of his breathing
D. Check his pulse oximetry

27. Which of the following respiratory rates may signify significant respiratory issues in an adult?
A. More than respirations 24 per minute
B. Fewer than 12 respirations per minute

C H A P T E R 13
C. Fewer than 8 respirations per minute
D. More than 20 respirations per minute

28. When the diaphragm and intercostal muscles relax, which of the following events occurs?
A. Inhalation
B. Inspiration
C. Release
D. Exhalation

29. A passive process whereby compounds move from an area of higher concentration to an area of lower
concentration is called:
A. Inhalation
B. Diffusion
C. Osmosis
D. Metabolism

30. You are assessing a 34-year-old man who is complaining of "not feeling well." He is alert and pale. His
respirations are regular at 20 per minute. Based on your training, you recognize that his respirations are:
A. Rapid but OK because they are regular
B. To be bradypnea
C. Normal for an adult
D. Rapid for an adult

31. You are called to assist a 16-year-old female who is reportedly having trouble breathing. Your assessment
reveals an anxious female with rapid respirations who is complaining of pins and needles around her lips. Based
on your training you suspect that:
A. She is having an allergic reaction and should be treated with an epi-pen.
B. These findings will disappear if she slows her rate of breathing because she is most likely hyperventilating.
C. She is close to respiratory failure.
D. Having her lie down with her feet elevated 8-12 inches will relieve the pins and needles.

32. A patient whose lungs alveoli are filled with fluid and pus secondary to pneumonia is at risk of:
A. Decreased exchange of oxygen and carbon dioxide in the lungs.
B. Decreased movement of air into the lungs.
C. Collapse of the trachea and bronchi.
D. Decreased movement of the diaphragm.

33. You arrive on scene where an unresponsive patient is being treated. The OEC Technician on scene tells you
that she thinks the patient may have had a stroke. You suggest to the patroller to first:
A. Determine if a pulse is present.
B. Place a nasal airway and suction the patient.
C. Apply a nonrebreather mask and provide oxygen.
D. Assist ventilations with a BVM.

Answer Key: Chapter 13 - Test

C H A P T E R 13
1=A, 2=D, 3=B, 4=A, 5=C, 6=D, 7=B, 8=D, 9=B, 10=C, 11=D, 12=D, 13=C, 14=B, 15=D, 16=A, 17=D,
18=B, 19=D, 20=D, 21=A, 22=B, 23=C, 24=C, 25=D, 26=C, 27=C, 28=D, 29=B, 30=C, 31=B, 32=A, 33=C


C H A P T E R 13
C H A P T E R 14
“Allergies and Anaphylaxis”

QUIZ

1. Components of the immune system include all of the following EXCEPT the:
A. The thymus.
B. The lymph nodes.
C. The pancreas.
D. The spleen.

2. Commonly encountered allergens include all of the following EXCEPT:


A. Mold.
B. Opiates.
C. Latex.
D. Peanuts.

3. An antibody can be described as:


A. A component of the bone marrow.
B. A T-cell variant found in the thymus.
C. A protein that is used by the body to destroy an antigen.
D. A cell produced by the thymus that engulfs an antigen.

4. Allergic reactions typically exhibit all of the following features EXCEPT:


A. Rapid changes in airway status.
B. Changes in heart rate and blood pressure.
C. They evolve over time.
D. Seizure activity.

5. The best early management steps of allergy and anaphylaxis management include all of the following EXCEPT:
A. Be prepared to administer an EpiPen if qualified to do so.
B. Administer supplemental oxygen with an NRB at 15 lpm.
C. Place the patient in the lateral recumbent position and administer oxygen via nasal cannula at 6 lpm.
D. Remove the patient from the allergen source and correct ABCD deficits.

Answer Key: Chapter 14 - Quiz


1=C, 2=B, 3=C, 4=D, 5=C

TEST

C H A P T E R 14
1. A patient experiencing an allergic reaction tells you that it is easier to breathe following administration of
epinephrine. Which of the following effects is an effect of epinephrine that would be responsible for this
improvement?
A. Dilation of the upper airway
B. Decreased blood pressure
C. Increased blood flow to the lungs
D. Dilation of the bronchioles

2. When the respiratory effects of an allergic reaction are obvious within a few seconds to minutes:
A. A large amount of allergen is likely involved.
B. The reaction tends to be more severe.
C. Epinephrine will not be effective.
D. The reaction tends to be localized.

3. Which of the following statements concerning allergies is true?


A. Once hypersensitized to a substance, a given individual will experience allergic reactions to that substance
randomly, not consistently.
B. Allergies affect young healthy individuals only.
C. An allergic reaction to a substance can occur minutes, or hours, after exposure to the substance.
D. Anaphylaxis is the mildest type of allergic reaction.

4. Your patient is having an allergic reaction and needs to use his epinephrine auto-injector. He is concerned that
he does not recall the correct way to do it. Which of the following things would you tell him?
A. "Place the injector against the outer upper arm, push the tip until the needle deploys, and then push the
plunger slowly to inject the medication.”
B. "Press the injector onto the outer thigh, push the tip against the thigh until the needle deploys.”
C. "Pinch the skin on the anterior thigh, press the injector tip against the skin, and wait until it activates.”
D. "Pinch the skin on the lateral thigh, press the injector against the skin, and rapidly push the plunger to
inject the medication.”

5. An acquired, abnormal immune response to a substance that does not normally cause an allergic reaction is
known as:
A. An allergy
B. An antigen
C. syncopal episode
D. An allergen

6. The overreaction of the immune system to a substance that is otherwise harmless is known as:
A. Hypersensitivity
B. Immunity
C. Hives
D. Anaphylaxis

7. During a primary assessment, which of the following findings would most lead you to suspect an allergic
reaction?
A. Hives on the face and neck
B. Heart rate of 55 beats per minute
C. A complaint of dizziness

C H A P T E R 14
D. Blood pressure of 100/60 mmHg

8. A patient who is responsive only to painful stimuli is covered with hives. He has labored respirations and a
weak and tachycardic pulse. His vital signs are pulse 128, respirations 24, and blood pressure 80/50 mmHg.
Given this presentation, you would suspect:
A. An anaphylactic reaction.
B. Pulmonary embolism
C. A head injury.
D. A stroke.

9. You are staffing a first-aid booth at a mountain bike event. A mother approaches you with her 15-year-old son
and informs you that her son is severely allergic to bees. Although he hasn't been stung, she is concerned about
the auto-injector he carries. The mother states that it was prescribed when her son was 13 years old. She shows
you the 0.15 mg auto-injector, and you note that it expired about 12 months ago. Which of the following
statements would be most appropriate for you to make?
A. "Most medications are still good for months after the expiration date. You can use this one for a little while
longer.”
B. "This medication is good for today but you should call your doctor as soon as possible to replace this.”
C. "Call your doctor's office for a new prescription because this auto-injector is out of date.”
D. "If you go to the pharmacy where you got this they will be able to give you a new one.”

10. When administering epinephrine to an infant or child, the injector should be placed at what location?
A. The anterior thigh
B. The upper arm
C. The outer buttock
D. The lateral thigh

11. A patient can have an allergic reaction caused by intramuscular administration of a medication. This method
by which an allergen enters the body is called:
A. inhalation.
B. injection.
C. ingestion.
D. topical.

12. Which of the following foods most commonly causes an allergic reaction?
A. Peanuts
B. Eggs
C. Chocolate
D. Shellfish

13. Which of the following findings is present in anaphylaxis but is not a sign of a mild allergic reaction?
A. Hives
B. Anxiousness
C. Swelling at the site of a bee sting
D. Hypotension

14. An acquired, abnormal immune response to a substance that does not normally cause a reaction is known as:
A. an allergen.

C H A P T E R 14
B. angioedema.
C. an antigen.
D. an allergy.

15. Which of the following statements from a patient suggests he may be having a mild allergic reaction?
A. "I have a rash on my face and neck.”
B. "I have had a fever and chills for the past two days.”
C. "My heart feels as though it is skipping some beats.”
D. "I feel dizzy and weak.”

16. You are staffing a first-aid booth at a mountain bike event. A mother approaches you with her 15-year-old son
and informs you that her son is severely allergic to bees. Although he hasn't been stung, she is concerned about
the auto-injector he carries. The mother states that it was prescribed when her son was 13 years old. She shows
you the 0.15 mg auto-injector, and you note that it expired about 12 months ago. Which of the following
statements would be most appropriate for you to make?
A. "If you go to the pharmacy where you got this they will be able to give you a new one.”
B. "Call your doctor's office for a new prescription because this auto-injector is out of date.”
C. "This medication is good for today but you should call your doctor as soon as possible to replace this.”
D. "Most medications are still good for months after the expiration date. You can use this one for a little while
longer.”

17. Which of the following findings would be a contraindication to the administration of epinephrine for a patient
suffering an anaphylactic reaction?
A. The patient is allergic to eggs.
B. The name on the injector is not the patient's name.
C. The patient has hypertension or bradycardia.
D. The patient has a heart rate of 120 bpm.

18. The most severe form of an allergic reaction is called:


A. Anaphylaxis
B. An antigen response
C. Rash
D. Rhinorrhea

19. You are called to the lodge to help a patient who has developed an allergic reaction after taking an antibiotic
medication. You would describe this route of exposure as:
A. Inhalation
B. Ingestion
C. Injection
D. Contact

20. Which of the following statements concerning allergies is true?


A. Allergies affect young healthy individuals only.
B. An allergic reaction to a substance can occur minutes, or hours, after exposure to the substance.
C. Once hypersensitized to a substance, a given individual will experience allergic reactions to that substance
randomly, not consistently.
D. Anaphylaxis is the mildest type of allergic reaction.

C H A P T E R 14
21. Which of the following findings is present in anaphylaxis but is not a sign of a mild allergic reaction?
A. Swelling at the site of a bee sting
B. Hypotension
C. Hives
D. Anxiousness

22. A patient can have an allergic reaction caused by intramuscular administration of a medication. This method
by which an allergen enters the body is called:
A. Ingestion
B. Inhalation
C. Injection
D. Topical

23. Which of the following statements indicates that the speaker understands allergic reactions and anaphylaxis?
A. "Anaphylaxis is a severe allergic reaction that will lead to death without emergency care."
B. "Allergic reactions are caused by pollen and food; anaphylactic reactions are caused by venoms and
medications.”
C. "An allergic reaction only occurs when the patient ingests or inhales an allergen; anaphylaxis occurs when
the allergen is injected.”
D. "Anaphylaxis describes a severe allergic reaction that occurs the first time a person is exposed to an
allergen.”

24. A foreign substance that triggers an allergic response in the body is known as:
A. histamine.
B. a mast cell.
C. an antibody.
D. an antigen.

25. As an OEC technician, Which of the following findings would be a contraindication to the administration of
epinephrine for a patient suffering an anaphylactic reaction?
A. The patient has hypertension or bradycardia.
B. The patient has a heart rate of 120 bpm.
C. The name on the injector is not the patient's name.
D. The patient is allergic to eggs.

26. The overreaction of the immune system to a substance that is otherwise harmless is known as:
A. anaphylaxis.
B. immunity.
C. hives.
D. hypersensitivity.

27. Your patient is having an allergic reaction and needs to use his epinephrine auto-injector. He is concerned that
he does not recall the correct way to do it. Which of the following things would you tell him?
A. "Pinch the skin on the anterior thigh, press the injector tip against the skin, and wait until it activates."
B. "Place the injector against the outer upper arm, push the tip until the needle deploys, and then push the
plunger slowly to inject the medication.”
C. "Press the injector onto the outer thigh, push the tip against the thigh until the needle deploys, and wait
for it to automatically inject.”

C H A P T E R 14
D. "Pinch the skin on the lateral thigh, press the injector against the skin, and rapidly push the plunger to
inject the medication.”

28. The patient has swollen lips and hives. Which of the following scene size-up observations supports your
suspicion that the patient is having an allergic reaction?
A. He states that several members of his family have the flu.
B. He has a bottle of expired nitroglycerin tablets.
C. He has a new container of antibiotics with only two pills missing.
D. He has a glucometer among his belongings.

29. The approach allergists recommend to allergies is called "Triple A," which refers to:
A. airway, assisted ventilations, and advanced life support.
B. allergy, antigen, and anaphylaxis.
C. awareness, avoidance, and action.
D. antibody, airway, and alveoli.

30. A foreign substance that triggers an allergic response in the body is known as:
A. Histamine
B. An antibody
C. An antigen
D. A mast cell

31. A patient informs you that any time she uses a particular soap; she experiences a mild allergic reaction. You
recognize that:
A. Her soap contains non-organic ingredients.
B. The soap contains antibodies that react in her body.
C. With continued use the allergic reaction will diminish.
D. The soap contains an allergen.

32. When administering epinephrine to an infant or child, the injector should be placed at what location?
A. The upper arm
B. The lateral thigh
C. The outer buttock
D. The anterior thigh

Answer Key: Chapter 14 - Test


1=D, 2=B, 3=C, 4=B, 5=A, 6=A, 7=A, 8=A, 9=C, 10=D, 11=B, 12=A, 13=D, 14=D, 15=A, 16=B, 17=B,
18=A, 19=B, 20=B, 21=B, 22=C, 23=A, 24=D, 25=C, 26=D, 27=C, 28=C, 29=C, 30=C, 31=D, 32=B

C H A P T E R 14
C H A P T E R 15
“Cardiovascular Emergencies”

QUIZ

1. Signs and symptoms of an abdominal aortic aneurysm can include all of the following EXCEPT:
A. Blood pressure that is higher in the right arm than left.
B. A palpable, pulsating abdominal mass.
C. Abdominal pain.
D. Pain that is also felt in the back.

2. You are attending a 58-year-old overweight gentleman in the lodge cafeteria who is rubbing his left shoulder
and complaining of a "squeezing" pain in his chest. He is sweating, has cool skin, and a palpable radial pulse at 82
beats per minute. Your first action should be to:
A. Administer an aspirin tablet.
B. Administer oxygen with an NRB at 15 lpm.
C. Attach an AED.
D. Administer a nitroglycerin tablet.

3. You have assisted a patient with a history of stable angina to correctly take the third of three nitroglycerin
tablets that he has been taking for chest pain over the last 15 minutes. Five minutes later his chest pain is
unchanged. What do you think may be happening?
A. He may be experiencing a myocardial infarction
B. The pain is not of cardiac origin
C. He did not take the first two tablets correctly
D. The nitroglycerin tablets have expired

4. What is an OECT's primary goal in the prehospital care of a chest pain patient?
A. To administer a baby aspirin tablet
B. To arrange for immediate EMS transport to definitive care
C. To recognize that the pain may be due to a cardiac event and provide proper care
D. To assure adequate oxygenation

5. Two OECTs observe a skier collapse in the lift line, determine cardiac arrest, and begin correctly performing
two-rescuer CPR. You arrive 3 minutes later with the AED and are informed by radio that there is an ALS unit
less than 1 minute away. You should:
A. Provide crowd control and clear an access route to the patient for the ALS team.
B. Apply and use the AED.
C. Relieve the rescuer performing chest compressions.
D. Contact the ALS unit by radio and provide details of the incident location.

C H A P T E R 15
Answer Key: Chapter 15 - Quiz
1=A, 2=D, 3=A, 4=C, 5=B

TEST

1. You are teaching a class on cardiovascular disease and need to describe what atherosclerosis is. Which of the
following statements is the most appropriate explanation?
A. "It affects veins and is caused by untreated high blood pressure."
B. "Cholesterol and lipid material accumulate within the walls of arteries."
C. "It affects the arteries of the heart and is caused by a heart attack."
D. "The walls of the coronary arteries lose elasticity, which leads to a heart attack.”

2. You are preparing to teach a class on hypertension. Which of the following statements provides incorrect
information that you would not include in your presentation?
A. The increased pressure in hypertension damages small arteries in many of the body's organs.
B. Hypertension is defined as a blood pressure with a systolic reading greater than 140 or a diastolic reading
greater than 90.
C. Hypertension results from the restriction of blood flow when the internal diameter of small arterioles is
reduced by atherosclerosis or other factors.
D. There are no known treatable risk factors for hypertension.

3. You are assisting a 67-year-old patient with chest pain. He has taken two nitroglycerin tablets but his pain
remains a 7 out of 10. He is diaphoretic; his respirations are 18 per minute and adequate; his pulse is 72; and his
blood pressure 80/62 mmHg. He is on a nonrebreather mask at 15L per minute. You would:
A. Assist him in taking his third and final nitroglycerin tablet.
B. Reassess his vital signs every 5 minutes and transfer him by EMS care.
C. Give him only half of a nitroglycerin tablet.
D. Attach him to an AED.

4. The largest artery in the body is the:


A. Carotid artery.
B. Jugular artery.
C. Femoral artery.
D. Aorta.

5. Which of the following statements about cardiovascular disease (CVD) is false?


A. CVD involves a number of diseases that affect either the heart or blood vessels.
B. Most cases of CVD are congenital and could be prevented with improved prenatal care.
C. The underlying cause of most cases of CVD is coronary artery disease or atherosclerosis of the coronary
arteries.
D. CVD is the leading cause of death worldwide.

C H A P T E R 15
6. You have been called to treat a 47-year-old man complaining of chest pain. He is alert, oriented, and
complaining of pain in his chest that came on suddenly about 15 minutes ago. His skin is sweaty and cool. Which
of the following actions should you take first?
A. Obtain a SAMPLE history.
B. Begin CPR.
C. Help him administer his nitroglycerin.
D. Apply oxygen at 15 LPM through a nonrebreather mask.

7. Which of the following statements indicates that the speaker has an understanding of chest pain and acute
myocardial infarction?
A. "If the chest pain started during activity, it is most likely cardiac in origin. If the pain started while the
patient was at rest, it is most likely not cardiac in origin.”
B. "If the patient's chest pain does not go away with nitroglycerin, you can assume that the medication has
expired and is no longer effective.”
C. "If the patient experiences chest pain that radiates into the left arm, it is most likely a heart attack. If the
pain radiates anywhere else, it is most likely not cardiac in origin.”
D. "Some patients experiencing acute myocardial infarction do not have actual chest pain; instead, they may
experience a sensation of 'severe heartburn.' “

8. Which of the following medications, if taken in the last 24 hours, would be a contraindication to administering
nitroglycerin?
A. Coumadin
B. Tylenol
C. Viagra
D. Aspirin

9. Which of the following actions is not an action you should take when using an AED?
A. Apply any set of electrodes as patient age is not relevant.
B. Ensure that the chest is dry and wiped clean of any medications.
C. Use the electrode to rip away excess chest hair if the electrode is not sticking well to the chest.
D. Ensure that no one is touching the patient when analyzing the patient's heart rhythm and when delivering
a shock.

10. Your neighbor tells you that he is concerned about his 70-year-old wife. She is at home recovering from a
heart attack that occurred about 3 weeks ago. He explains that over the last few days her ankles and lower legs
have been gradually swelling and are painful to touch. Today she seems to be a little short of breath. You
recommend that your neighbor seek immediate medical attention because these findings are suggestive of:
A. Tension pneumothorax
B. Congestive heart failure
C. An acute myocardial infarction
D. Pericarditis

11. You are called to assist a 62-year-old female who is complaining of fatigue, jaw discomfort, and excessive
sweating. When asked, she denies having chest pain or shortness of breath. She tells you that she has a history of
hypertension and takes a blood pressure pill. She also has nitroglycerin. Which statement is inappropriate for this
event?
A. "I am going to check her blood pressure.”
B. "She is not short of breath, so we don't need to transport her.”

C H A P T E R 15
C. "I am not going to attach the AED because she is conscious and has a pulse.”
D. "I am going to call for ALS because she could be having a cardiac event.”

12. Which of the following actions is most important before assisting a patient to take prescribed nitroglycerin?
A. Determine whether the patient has a headache.
B. Attach the patient to an AED.
C. Determine whether the patient has any allergies.
D. Evaluate the patient's blood pressure.

13. Hypotension, distended neck veins, and muffled or distant heart tones are hallmark signs of:
A. Aortic aneurysm
B. Pericardial tamponade
C. Congestive heart failure
D. Pericarditis

14. Which of the following structures help direct blood flow forward and prevent the backflow of blood within
the heart?
A. Septa
B. The atrioventricular valves
C. The coronary arteries
D. The pulmonary vein and the pulmonary artery

15. You are in the first-aid area assisting a patient who just arrived with chest pain. He relates a history of angina
that usually resolves with nitroglycerin. The patient states that the pain started when he was skiing through the
bumps. Over the last 20 minutes, he has taken three nitroglycerin tablets and rested, but the pain has not gone
away. Your next action would be to:
A. Offer an aspirin.
B. Try one more nitroglycerin tablet because nitroglycerin usually relieves his pain.
C. Attach an AED in case he goes into cardiac arrest.
D. Place him on high-flow oxygen, and call for transfer to medical facility.

16. You are caring for a patient with chest pain who has just taken his nitroglycerin. The patient's chest pain has
been alleviated. You understand that nitroglycerin is effective in treating chest pain because it:
A. Increases the heart rate and gets more blood to the heart muscle.
B. Dilates the arteries and veins, which decreases the workload on the heart.
C. Has a sedative effect, which makes the patient less aware of the pain.
D. Causes the body to relax, decreasing the need for oxygen.

17. Blood in the right ventricle is pumped into the:


A. Left ventricle.
B. Left atrium.
C. Pulmonary artery.
D. Aorta.

18. Before nitroglycerin is administered, you should ensure that:


A. The patient's systolic blood pressure is at least 110 mmHg.
B. Nitroglycerin is in fact prescribed for the patient.
C. The patient is standing.

C H A P T E R 15
D. The patient rates his chest pain as at least 7 on a scale of 1 to 10.

19. When performing cardiac compressions on a 12-year-old child, you would:


A. Administer cycles of 30 chest compressions followed by two breaths.
B. Place the heel of your hand on the upper section of the sternum.
C. Recheck the pulse every four cycles.
D. Compress the chest about 1 inch.

20. You have just arrived with an AED at a scene where fellow OEC Technicians are treating a patient in cardiac
arrest. EMS has been contacted and is about 1 minute away. Which of the following actions should you take?
A. Help count compressions.
B. Apply and use the AED.
C. Go outside to direct ALS when they arrive.
D. Let the OEC Technicians continue CPR and wait for ALS.

21. You are assessing a 38-year-old woman who is complaining of the sudden onset of chest pain and dyspnea.
She describes the pain as sharp and states that it increases when she takes a deep breath. You note that she is
tachypneic. Her BP is 130/82 and her pulse is 100. Based on her presentation, you suspect that she may have:
A. pulmonary embolism.
B. an aortic dissection/aneurysm.
C. unstable angina.
D. pericardial effusion.

22. Which of the following actions is not an action you should take when using an AED?
A. Use the electrode to rip away excess chest hair if the electrode is not sticking well to the chest.
B. Ensure that the chest is dry and wiped clean of any medications.
C. Ensure that no one is touching the patient when analyzing the patient's heart rhythm and when delivering
a shock.
D. Apply any set of electrodes as patient age is not relevant.

23. A patient with chest pain informs you that he has a bad heart. You think he may have congestive heart failure.
When performing your secondary assessment, which of the following signs or symptoms does not indicate that
the patient is in congestive heart failure?
A. Swollen ankles
B. Bubbling breathing noises
C. Bradycardia
D. Shortness of breath

24. After jogging with your friend for about 30 minutes, she stops and asks why your heart rate increases and
your heart feels like it is pumping harder after you exercise for a while. Based on your understanding of the
cardiovascular system, your best response would be which of the following statements?
A. "You're out of shape. Those things wouldn't happen if you exercised more."
B. "Exercise makes your body need more nutrients and oxygen; your heart pumps harder and faster to deliver
more blood containing those things to your muscles.”
C. "Exercise pushes more blood into your heart, so your heart has to work harder and faster to pump the
blood out so you don't go into heart failure.”
D. "When you exercise, your blood pressure increases, so your heart has to work harder to overcome the
higher blood pressure.”

C H A P T E R 15
25. A 62-year-old man presents with a complaint of abdominal pain radiating to his groin. He also complains of
dizziness. On examination, his abdomen is tender and you feel a large pulsating mass. His signs and symptoms
are suggestive of:
A. Acute embolism.
B. Cardiogenic shock
C. An abdominal aortic aneurysm
D. A ruptured spleen

26. Which of the following statements made by a patient would cause you to suspect an aortic abdominal
aneurysm?
A. "The pain is sharp and spreads to both arms.”
B. "My heart is beating so hard, you can feel it on my chest.”
C. "I have really bad belly pain that I also feel in my back.”
D. "I seem to be vomiting up some blood.”

27. A patient tells you that he has been diagnosed with coronary artery disease. You recognize this diagnosis to
mean that the patient has:
A. A history of acute myocardial infarction.
B. A failure of the heart to efficiently pump blood.
C. Narrowing of the small arteries that supply blood and oxygen to the heart.
D. Had an operation known as coronary artery bypass grafting?

28. One complication of an acute myocardial infarction may be cardiogenic shock. Based on your training, you
would define cardiogenic shock as a (n):
A. Absolute or relative shortage of blood supply to a tissue due to a blood clot.
B. Condition whereby tissues of the body are oxygen deprived due to the heart's inability to adequately pump
blood.
C. Abrupt cessation of the regular electrical activity of the heart.
D. Acute illness in which the cardiac muscle does not get enough oxygen.

29. Which of the following statements concerning the flow of blood to and through the right side of the heart is
correct?
A. Blood reaches the right atrium from the aorta, is pumped through the mitral valve into the right ventricle,
and is then pumped through the pulmonary veins to the lungs.
B. Poorly oxygenated blood reaches the right atrium from the venae cavae, is pumped through the tricuspid
valve into the right ventricle, and is then pumped through the pulmonic valve and on to the lungs through
the pulmonary arteries.
C. Oxygenated blood reaches the right atrium from the pulmonary veins, is pumped through the tricuspid
valve into the right ventricle, and is then pumped into the aorta and throughout the body.
D. Poorly oxygenated blood reaches the right ventricle from the venae cavae, is pumped through the pulmonic
valve into the pulmonary artery, and then enters the right atrium and is pumped into the lungs.

30. Which of the following is not a reason to stop CPR?


A. You are ready to use an AED.
B. The patient has the spontaneous return of a pulse and breathing.
C. Rescuers are too tired to continue administering CPR.
D. It has been 25 minutes, so brain damage has occurred.

C H A P T E R 15
31. In performing CPR on an infant:
A. Place two fingers on the sternum just beneath an imaginary line between the nipples, and make
compressions that are approximately one-third the depth of the chest.
B. Check the carotid artery for a pulse before beginning compressions.
C. Place the heel of one hand on the sternum just above an imaginary line between the nipples, and make
compressions that are about 1/2 inch deep.
D. Stop to check for a pulse after administering about 100 compressions.

32. After jogging with your friend for about 30 minutes, she stops and asks why your heart rate increases and
your heart feels like it is pumping harder after you exercise for a while. Based on your understanding of the
cardiovascular system, your best response would be which of the following statements?
A. "Exercise pushes more blood into your heart, so your heart has to work harder and faster to pump the
blood out so you don't go into heart failure.”
B. "You're out of shape. Those things wouldn't happen if you exercised more.”
C. "When you exercise, your blood pressure increases, so your heart has to work harder to overcome the
higher blood pressure.”
D. "Exercise makes your body need more nutrients and oxygen; your heart pumps harder and faster to deliver
more blood containing those things to your muscles.”

33. You are assessing a 38-year-old woman who is complaining of the sudden onset of chest pain and shortness of
breath. She describes the pain as sharp and states that it increases when she takes a deep breath. You note that
she is breathing rapidly. Her BP is 130/82 and her pulse is 100. Based on her presentation, you suspect that she
may have:
A. Unstable angina
B. Pericardial effusion
C. An aortic dissection/aneurysm
D. Pulmonary embolism

34. Aspirin may be used in a patient who has chest pain suggestive of a heart attack because aspirin may:
A. Relieve the headache that is caused by the administration of nitroglycerin.
B. Break up a blood clot that is occluding a coronary artery.
C. Decrease the chest pain caused by the possible heart attack.
D. Decrease the ability of to form clots.

35. When assisting a patient to take a nitroglycerin tablet, it is:


A. Swallowed with only an ounce of water.
B. Placed under the tongue.
C. Placed between the cheek and teeth.
D. Chewed and then allowed to dissolve slowly.

36. Which of the following signs or symptoms is suggestive of right-sided congestive heart failure?
A. An irregular heart rate
B. Elevated blood pressure
C. Edema in the legs and feet
D. A bounding pulse

37. A hardening and loss of elasticity of the arterial walls is known as:

C H A P T E R 15
A. High blood pressure.
B. Arteriosclerosis.
C. Atherosclerosis.
D. Angina pectoris.

38. Which of the following statements concerning the cardiovascular system is true?
A. The heart is a muscular organ located behind and to the right of the sternum.
B. An electrical impulse that starts in the right atrium stimulates contraction of the heart muscle.
C. The subclavian arteries supply blood to the pelvis and legs.
D. It consists of four major components: the heart, the lungs, the blood vessels, and blood.

39. You are assessing a 68-year-old man who is complaining of severe abdominal pain that radiates to his back.
He tells you that he has a history of an abdominal aortic aneurysm. Based on this history and his complaints, on
your physical exam you would look for:
A. Abdominal pain that is relieved when you place the patient on his side.
B. A pulsating mass in his abdomen.
C. Blood pressure that is higher in one arm than in the other.
D. Bruising on his abdomen.

40. An alert and oriented 65-year-old woman presents with shortness of breath, noisy respirations, jugular vein
distention, and edema in her feet and ankles. Her pulse is 132, her respirations are 24, and her blood pressure is
86/68. Based on these findings, you would suspect that the patient:
A. Has an aortic dissection/aneurysm.
B. Has pericardial tamponade.
C. Is having a hypertensive emergency.
D. Is experiencing congestive heart failure.

41. Three commonly used cardiac medications are:


A. Lipitor, Dilantin, and Tenormin
B. Coumadin, Zesteril, and an Epi-pen
C. Plavix, Nitroglycerine, and Albuterol
D. Nitroglycerine, Coumadin, and Lasix

42. Which of the following statements concerning arteries is true?


A. They always carry oxygenated blood.
B. They carry blood away from the heart.
C. They have a lower pressure than veins.
D. They drain into the venae cavae.

43. The walls of the capillaries are only one cell thick. This allows the exchange of nutrients and oxygen and
waste products. The capillaries serve as the bridge between:
A. Arteries and veins.
B. The atrium right and the left atrium.
C. The pulmonary vein and the pulmonary artery.
D. Arterioles and venules.

44. The left side of the heart:


A. Pumps oxygenated blood to the body.

C H A P T E R 15
B. Pumps poorly oxygenated blood to the lungs.
C. Contains the pulmonic and tricuspid valves.
D. Receives poorly oxygenated blood from the body.

45. You are out shopping when a middle-aged man suddenly falls to the ground. You quickly run to assist him
and note that he is not breathing and does not have a carotid pulse. You begin CPR because you know that
prompt intervention with an AED is needed. Which of the following statements is not a reason that prompt
intervention with an AED is needed?
A. Two common arrhythmias that are associated with sudden cardiac death - ventricular fibrillation and
pulseless ventricular tachycardia - can respond to AED use.
B. Asystole, which can accompany an acute MI, is often reversed by the use of an AED.
C. Studies show that after 10 minutes of pulseless cardiac arrest due to V-fib or V-tach, the heart is less
responsive to electrical stimulation.
D. Since AED was first used by trained and lay rescuers, patient survival from sudden cardiac arrest has
doubled.

Answer Key: Chapter 15 - Test


1=B, 2=D, 3=B, 4=D, 5=B, 6=D, 7=D, 8=C, 9=A, 10=B, 11=B, 12=D, 13=B, 14=B, 15=D, 16=B, 17=C,
18=B, 19=A, 20=B, 21=A, 22=D, 23=C, 24=B, 25=C, 26=C, 27=C, 28=D, 29=B, 30=D, 31=A, 32=D,
33=D, 34=D, 35=B, 36=C, 37=B, 38=B, 39=B, 40=D, 41=D, 42=B, 43=D, 44=A, 45=B

C H A P T E R 15
C H A P T E R 16
“Gastrointestinal/Genitourinary Emergencies”

QUIZ

1. A middle-aged male presents with severe abdominal pain; nausea; vomiting; cool, moist skin; and uneven
femoral pulses. This patient is most likely suffering from what potentially life-threatening condition?
A. Cholecystitis
B. Appendicitis
C. An aortic aneurysm.
D. Pancreatitis

2. In which position should most patients complaining of GI/GU pain be transported?


A. In the Trendelenburg position
B. In the Fowler's position
C. In the NATO position
D. On their back with their knees flexed

3. Which of the following are solid organs?


A. Small bowel, large bowel, and ureters
B. Ureters, duodenum, and transverse and ascending colon
C. Liver, pancreas, spleen, and ovaries
D. Gallbladder, urinary bladder, and appendix

4. Which statement about acute abdomen is NOT true?


A. It is defined as slowly developing, initially minor pain that becomes worse over several hours or days.
B. It is more important for an OECT to recognize that a patient is ill rather than to try and pinpoint the exact
cause of the problem.
C. It is often difficult to identify the source of the problem.
D. It is defined as sudden, severe, unexplained pain.

5. When assessing a patient with a potential acute abdomen, the OECT should do all of the following EXCEPT:
A. Assess all vital signs, including blood pressure, at 5-minute intervals.
B. Offer the patient small sips of water to determine whether or not doing so decreases pain or nausea.
C. Palpate all four abdominal quadrants.
D. Assess the patient's femoral pulses, bilaterally.

Answer Key: Chapter 16 - Quiz


1=C, 2=D, 3=C, 4=A, 5=B

C H A P T E R 16
TEST

1. Abdominal aortic aneurysm (AAA) is a true emergency because it can lead to:
A. severe pain.
B. vomiting.
C. moderate blood loss.
D. sudden death.

2. Lower abdominal pain could indicate which of the following conditions?


A. A urinary tract infection
B. An irregular heart beat
C. A fractured rib
D. An auditory tract infection

3. When you are obtaining information for a medical history, which of the following items would be significant
for a patient with suspected cholecystitis?
A. The patient started taking a new medication for high blood pressure.
B. The patient ate fried chicken and potato salad for dinner.
C. The patient experienced a recent unexplained weight loss.
D. The patient has abstained from drinking alcohol over the past month.

4. Which of the following is the best sequence for examining the abdomen of a patient complaining of lower right
abdominal pain?
A. LLQ, RLQ, RUQ, LUQ
B. LUQ, LLQ, RUQ, RLQ
C. RUQ, RLQ, LUQ, LLQ
D. RLQ, RUQ, LUQ, LLQ

5. A 14-year-old girl sitting in the lodge is complaining of severe abdominal and flank pain. She tells you that her
symptoms started a couple of days ago but have worsened over the last few hours. She has chills and thinks she
might have a fever. Based on her complaints, you are concerned that she may have:
A. cholecystitis.
B. hepatitis.
C. nephrolithiasis.
D. pyelonephritis.

6. You suspect that a man experiencing right lower quadrant pain has appendicitis. He states that he has no
health insurance and wants to refuse care. He asks you what can happen if he has appendicitis but does not go to
the hospital. Which of the following statements would you make in reply?
A. "Take an antacid, and if the pain does not subside in an hour or two, then you should go to the hospital.”
B. "If you do not get care, blood may clot in your intestine and cause an obstruction.”
C. "Your appendix could rupture, causing a major infection, shock, and possible death.”
D. "The inflammation of the appendix will gradually subside, but in the meantime the pain can be terrible."

C H A P T E R 16
7. You are called to assist a woman complaining of abdominal pain. Assessment reveals her to be nauseated and
to have pain that is localized to the right lower quadrant of her abdomen. She states that she was not able to take
her blood pressure medication this morning, and that she wants to take it now with a small glass of juice. Until
she receives a further medical evaluation, you would encourage her to:
A. not take her medication or drink juice.
B. crush the medication and then mix it in a small amount of water.
C. take the medication with water instead of juice.
D. go ahead and take the medicine with juice.

8. What is typically the most comfortable position for a responsive patient who is suffering from abdominal pain?
A. Trendelenburg position
B. Supine with knees slightly flexed
C. Left lateral position
D. Supine with feet elevated 8 inches

9. Which of the following statements regarding acute abdomen is false?


A. OEC Technicians need to be able to pinpoint the source of an acute abdomen.
B. An acute abdomen can rapidly spiral into septic or hypovolemic shock.
C. OEC Technicians need only be able to recognize that the patient is very ill and needs to go the hospital.
D. Physicians accurately diagnose the source of an acute abdomen on first examination only about 50 percent
of the time.

10. A 28 year old healthy male presents with recent signs and symptoms of nausea and vomiting but very little
abdominal pain. Which one of the following is your best course of action?
A. You determine that the patient is having a silent myocardial infarction and call for ALS and transport the
patient immediately
B. Suggest the patient take some over the counter analgesics to see if they help and release back to skiing
C. The patient has indigestion; suggest that he follow up with his physician if the symptoms continue
D. Have the patient urinate in a cup and look for blood in the urine

11. An infection involving one or both kidneys and ureters is called:


A. a bowel obstruction.
B. a perforated bowel.
C. pyelonephritis.
D. melena.

12. A young patient is complaining of severe lower right quadrant pain with rebound tenderness. You would
advise them to do which of the following?
A. Urinate in a cup and look for blood in the urine
B. Go to the hospital by ambulance
C. Purchase some over the counter antacids and take them
D. Drink some milk to settle the stomach

13. When a patient has generalized abdominal pain and can only describe a larger area that is painful, it is known
as _____________ pain.
A. Rebound
B. Visceral
C. Celiac

C H A P T E R 16
D. Parietal

14. When a patient can point to an exact location of pain over their abdomen, this is known as _________________
pain.
A. Visceral
B. Diffuse
C. Referred
D. Parietal

15. The most common GI problem that OEC Technicians encounter is:
A. constipation.
B. gastroenteritis.
C. indigestion.
D. colic.

16. Which of the following statements regarding assessment of the abdomen is false?
A. One of the problems encountered while assessing abdominal complaints is a patient's reluctance to allow
the exam is due to embarrassment or pain
B. Abdominal pelvic organs are close to each other and problems affecting one system or organ can seriously
affect organs in another system
C. Assessment of the abdominal organs is straightforward because the abdominal organs are specifically
located and easily palpated
D. Assessing abdominal complaints can be difficult because the nerve pathways for the gastrointestinal,
urinary, and reproductive systems are in relatively close proximity to each other

17. Which one of the following statements regarding an acute abdomen is true?
A. Rarely is there nausea and vomiting accompanying an acute abdomen
B. An acute abdomen rarely results in sepsis or hypovolemic shock
C. OEC technicians need to be able to pinpoint the exact source of an acute abdomen
D. OEC technicians should be able to make a diagnosis that the patient is ill enough to warrant transport to a
hospital

18. A conscious patient with a GI/GU complaint is most often comfortable in:
A. the Trendelenburg position.
B. the recovery position.
C. a supine position with the feet elevated 12-18 inches.
D. a supine position with the knees slightly flexed.

19. Rebound tenderness is best described as:


A. severe pain that occurs after the sudden release of abdominal palpation pressure during an examination of
the abdomen.
B. a very useful finding for diagnosing a problem in a prehospital environment.
C. pain that radiates into an area other than the site of the affected organ.
D. a type of pain that is initially relieved by vomiting but returns within a short period of time.

20. Colic is best described as:


A. an inflammatory condition involving the lining of the stomach.
B. a violent contraction of the stomach muscles due to noxious stimuli.

C H A P T E R 16
C. bloody stool.
D. intermittent severe abdominal pain caused by the obstruction and distension of a hollow organ.

21. Which of the following statements regarding abdominal pain would lead you to suspect appendicitis?
A. "I had back pain for about a week, and now the pain is all throughout my belly."
B. "The pain started around my belly button and slowly spread down to the right side of my belly."
C. "The pain comes and goes and is more noticeable on the left side of my stomach."
D. "The stabbing pain started in the right side of my belly and spread up to my stomach."

22. Which of the following statements is false?


A. Assessing abdominal complaints can be difficult because the nerve pathways for the gastrointestinal,
urinary, and reproductive systems are in relative proximity to each other.
B. One of the problems encountered when assessing abdominal complaints is a patient's reluctance to allow
the exam due to embarrassment or pain.
C. Assessment of abdominal complaints is straightforward because the organs are specifically located and
easily palpated.
D. Problems affecting the organs in one system can seriously affect organs in another system.

23. Colic is often caused by which one of the following?


A. Passing a bloody stool
B. Contraction of the abdominal muscles from injury
C. Drinking too much water
D. Obstruction and distension of the colon

24. Which one of the following organs is not located in the abdomen?
A. Small intestines
B. Pancreas
C. Stomach
D. Kidney

Answer Key: Chapter 16 - Test


1=D, 2=A, 3=B, 4=B, 5=D, 6=C, 7=A, 8=B, 9=A, 10=C, 11=C, 12=B, 13=B, 14=D, 15=B, 16=C, 17=D,
18=D, 19=A, 20=D, 21=D, 22=C, 23=D, 24=D


C H A P T E R 16
C H A P T E R 17
“Principles of Trauma"

QUIZ

1. The "Golden Hour" of the post-injury phase refers to:


A. The time of transportation from accident site to the ambulance.
B. The time from dispatch to arrival on scene.
C. The time after injury in which timely interventions are critical to survival.
D. The time from impact to arrival at the hospital.

2. Classifications of mechanism of injury include all of the following EXCEPT:


A. Tearing injury.
B. Blunt injury.
C. Penetrating injury.
D. Crush injury.

3. The combination of which two factors has the most impact when considering the potential for injury severity?
A. Stopping distance and speed
B. Weight and reaction time
C. Reaction time and speed
D. Tissue density and weight

4. The trauma assessment primary survey includes:


A. Blood pressure, detailed physical exam, and medication history.
B. Abdominal exam, palpation of radial pulse, and capillary refill time.
C. Scene size-up, assessment of ABCs, and performance of life-saving interventions as needed.
D. Chest auscultation, spinal precautions, and respiratory rate.

5. The major considerations in the initial management of a trauma patient include:


A. Available resources, injury severity, mechanism of injury, and transport distance.
B. Airway adjuncts and status of oxygen tanks.
C. Medical insurance, informed consent, and patient confidentiality.
D. Parental permission to treat a minor and availability of warming equipment.

Answer Key: Chapter 17 - Quiz


1=C, 2=A, 3=A, 4=C, 5=A

TEST

C H A P T E R 17
1. A 49-year-old man has been stabbed in the lower right chest. After assessing him, you suspect that the knife
punctured the lung and is causing internal bleeding. In this situation, the mechanism of injury would be:
A. A penetrating injury
B. An assailant with a knife
C. Internal hemorrhage
D. Blunt trauma to the lungs

2. Trauma centers are specifically designed to:


A. Encourage patients with less urgent injuries to use lower-level facilities.
B. Be testing areas for physicians who wish to be "board certified" as trauma surgeons.
C. Provide training to physicians who will work with trauma patients.
D. Manage patients with multi-system trauma.

3. A rock climber falls when several boulders give way. His arm is pinned between two boulders. This MOI is best
described as:
A. Blunt
B. Crush
C. Rotational
D. Penetrating

4. Which of the following best defines mechanism of injury?


A. A set of problems associated with a specific source of trauma
B. Physical injury caused by an external force
C. The study of movement of body segments without consideration given to its mass or the forces making it
move.
D. The method that an injury occurs.

5. Your patient has been stabbed in the chest with a ski pole. The mechanism of injury is:
A. Rotational
B. Crush
C. Blunt
D. Penetrating

6. Dislocating a shoulder after catching a ski pole on a tree branch is an example of an injury caused by which
MOI?
A. Compressive injury
B. Rotational injury
C. Whiplash injury
D. Blunt injury

7. You arrive alone at a scene at which a 16-year-old skier has collided with a tree. He is unresponsive and has
blood flowing from his ears and nose. He was not wearing a helmet. Which of the following lists reflects the most
appropriate care of this patient?
A. Primary assessment, immobilization, bandaging of the ears to control bleeding, airway management
B. Immobilization, transfer to a sled, completion of the assessment in the aid room
C. Primary assessment, oxygen administration, rapid transport to the aid room, immobilization in the aid
room
D. Primary assessment, airway management, immobilization, transport as soon as possible

C H A P T E R 17
8. High-velocity injuries and low-velocity injuries are two types of which mechanism of injury?
A. Penetrating injury
B. Blunt trauma
C. Closed trauma
D. Blast injury

9. Trauma centers have five levels which are based on:


A. Number of doctors on staff
B. Insurance providers
C. Proximity to ski areas
D. Specialization, complexity, and availability of care

10. High-velocity penetrating, an injury is usually the result of:


A. The collision of cars traveling at greater than 65 mph
B. Stab wounds
C. The impact of a bullet from a high-powered rifle
D. An out-of-control skier hitting another skier

11. Assessment and management of the trauma patient should include:


A. Gathering information about the mechanism of injury
B. Initiating CPR on all patients with chest trauma
C. Transporting trauma patients as soon as the Scene Size Up is complete
D. Reassessing the patient every 30 minutes

12. You are called to the vehicle maintenance department on the mountain, where a fairly large explosion has
occurred. The first injured person you talk to tells you that he is having excruciating ear pain. You recognize that
his ear pain may be caused by:
A. A blast injury
B. A crush injury
C. A blunt injury
D. A penetrating injury

13. After determining the mechanism of injury, you are able to form an initial impression of what could be
injured and how bad the injury might be. This is called:
A. Kinetic energy
B. Multi-system trauma assessment
C. Index of suspicion
D. Kinematics

14. Your patient is a skier who tells you that she "caught an edge and twisted her leg. You later find out that the
skier suffered from a fractured tibia and fibula. This injury was the result of which mechanism:
A. Penetration
B. Crush
C. Blunt
D. Rotational

C H A P T E R 17
15. Your friends are discussing a hunting accident they heard about on the news. A hunter was shot with a high-
powered rifle, and the report indicated that he had damage to several internal organs. From your knowledge of
mechanisms of injury, you can tell your friends that this type of injury is called a:
A. High-impact injury
B. High-velocity penetrating injury
C. Projectile injury
D. Blast injury

16. Which of the following terms is not a "mechanism of injury"?


A. Crushed foot
B. Rotational spiral mid-shaft tibia fracture
C. Kidney stone
D. Blunt trauma to the abdomen

17. The term golden hour refers to the:


A. Amount of time it takes to stop the bleeding from a penetrating injury.
B. Hour before sunset, when most cases of on-hill trauma occur.
C. Amount of time you have to decide if a patient needs to be sent to a trauma hospital.
D. Time period when patient survival rates may be enhanced if critical injuries are identified and managed.

18. Your patient has been involved in a backcountry hiking accident and has multi-system trauma. It is most
important to consider which of the following when making your evacuation plans:
A. Location of the patient's home
B. Location of the patient's nearest family member
C. Location of the patient's primary doctor
D. Location of the nearest trauma center

19. A heavy-set snowboarder was critically injured when he crashed into a tree on a dimly lit trail. It was reported
that he appeared to be out of control while traveling at a high rate of speed down the expert trail. Which of the
following factors had the greatest impact on the extent of his injuries?
A. The skier's weight
B. The speed the skier was going
C. The dimly lit trail
D. The size of the tree he hit

20. Which of the following factors does not directly relate to the severity of bodily injury?
A. The amount of kinetic energy absorbed
B. The size of the injured person
C. The density of the structures impacted
D. The direction the kinetic energy travels

21. A patient involved in an altercation was struck in the ribs with a baseball bat. Your assessment reveals intact
skin with significant bruising to the right lateral chest. When palpating this area you note instability and crepitus
to the rib cage. An OEC Technician would recognize a:
A. Pulmonary injury caused by penetrating trauma.
B. Chest wall injury caused by acceleration forces.
C. Thoracic injury caused by penetrating trauma.
D. Chest injury caused by blunt trauma.

C H A P T E R 17
22. A 23-year-old skier falls about 20 feet from the chairlift. Aside from his complaints of soreness, you don't see
any obvious injuries. The skier states that as long as his legs aren't broken, he's going to continue skiing. Which
of the following statements would be your best response to him?
A. "You may feel OK now, but you will probably be sore later. You should go to the hospital and get some pain
medication."
B. "Since this happened at a ski area you should get evaluated. You may be able to sue.”
C. "From a fall of that height, you may have some injuries. You really should be examined.”
D. "Since you did not lose responsiveness, it's probably OK to go back to skiing. If your legs start to hurt, you
should probably get checked."

23. Your patient is an adult skier who collided with a snowboarder and was found to be unresponsive. The scene
is safe. After establishing the ABCs and stabilizing the patients head and spine, you will:
A. Reassess the patient every 5 minutes
B. Try to obtain a medical history
C. Take off a ski boot and see if you can find a pulse.
D. Perform a head-to-toe secondary assessment

Answer Key: Chapter 17 - Test


1=A, 2=D, 3=B, 4=D, 5=D, 6=B, 7=D, 8=A, 9=D, 10=C, 11=A, 12=A, 13=C, 14=D, 15=B, 16=C, 17=D,
18=D, 19=B, 20=D, 21=D, 22=C, 23=D


C H A P T E R 17
C H A P T E R 18
“Soft Tissue Injuries”

QUIZ

1. A tourniquet should be used when:


A. A bleeding patient is found to be taking anticoagulants.
B. The bleeding site is more than 3 inches distal to a joint.
C. Life-threatening extremity arterial bleeding continues despite correct application of direct pressure and
pressure dressings.
D. Patient packaging will obscure the bleeding site.

2. Which of the following is an essential element of treating a patient with soft tissue injuries?
A. The use of barrier dressings over hematomas
B. The correct use of appropriate personal protective equipment
C. The application of antibacterial ointment to abrasions
D. The use of sterile dressings to prevent infection

3. Functions of the skin include:


A. Glucose metabolism and blood oxygenation.
B. Water conservation and carbon dioxide excretion.
C. Hemostasis and clotting factor synthesis.
D. Temperature regulation and vitamin D synthesis.

4. Subcutaneous fat is useful for:


A. The dissipation of internal bleeding.
B. The containment of bone fragments after a closed fracture.
C. Insulation and energy storage.
D. The protection of internal organs from crush injury.

5. A pressure dressing applied to a laceration of the forearm has successfully stopped the previously brisk external
arterial bleeding. You would next:
A. Check the CMS in the hand.
B. Splint the joints above and below the laceration.
C. Apply additional dressings as needed for continued bleeding.
D. Obtain postural vital signs.

Answer Key: Chapter 18 - Quiz


1=C, 2=B, 3=D, 4=C, 5=A

C H A P T E R 18
TEST

1. Which of the following statements about amputations is true?


A. All amputated body parts should be preserved and sent to the hospital with the patient.
B. Surgeons can reattach an amputated part only if it has been without a blood supply for less than one hour.
C. A badly mangled or damaged amputated part should not be preserved or sent to the hospital because it
cannot be reattached.
D. Keep the amputated part at body temperature when sending it to the hospital with the patient.

2. Mechanical tattooing is defined as:


A. A type of self-mutilation requiring psychiatric follow-up for the patient.
B. A form of body art in which dye is injected under the skin.
C. The introduction of foreign debris such as dirt, gunpowder, or small rocks into the skin.
D. Multiple punctures made by a nail gun.

3. Which of the following statements about pressure dressings and bandages is true?
A. Bandages should be applied in a distal-to-proximal manner.
B. All pressure dressings must be airtight to prevent contamination.
C. Dressings are placed over bandages.
D. Gauze should be placed in an open wound before a pressure dressing is applied.

4. A patient has had part of his right thumb amputated in an accident. Friends have retrieved the thumb and
wrapped it in a towel. When you arrive, you would demonstrate appropriate handling of the amputated part by:
A. Wrapping the thumb in a moist sterile dressing, placing it in a sterile bag, and keeping it cool.
B. Placing the thumb in a plastic bag filled with ice.
C. Placing the thumb in a container of sterile saline or sterile water.
D. Keeping the thumb in the towel.

5. Which of the following injuries is most likely to result in capillary bleeding only?
A. A laceration
B. An avulsion
C. An abrasion
D. A penetration

6. For which of the following injuries may elevation be used to control bleeding?
A. For an abrasion on the lower leg
B. For no injuries; elevation has not been shown to help control bleeding.
C. For a laceration to the forearm
D. For a puncture wound to the foot

7. Which of the following statements about a high-pressure injection injury is false?


A. The presenting wound is typically large.
B. The injury may require surgical amputation of the injured part.
C. The injury involves the introduction of a liquid or gas into the body from a pressurized source.
D. The injury typically involves the limbs.

C H A P T E R 18
8. Which of the following lists correctly names the layers of the skin?
A. Dermis, subcutaneous, muscle
B. Subcutaneous, epidermis, dermis
C. Epidermis, subcutaneous, muscle
D. Dermis, muscle, epidermis

9. Which of the following assessment findings best indicates that a patient's injury should be classified as an
abrasion?
A. Intact skin with ecchymosis noted; patient describes minor pain
B. Intact skin with a large accumulation of dark blood underneath
C. Red scrapes involving the outermost layer of skin only
D. Jagged breaks in the skin; bleeding moderate but controllable

10. You are assessing a patient who received multiple injuries in a fight. Which of the following injuries would
present the greatest danger for infection?
A. A contusion to the chin
B. A closed crushing injury to the thigh
C. An abrasion on the right side of the face
D. A hematoma in the left foot

11. You respond to a patient who has an avulsion on the left arm. When you assess this patient, you would expect
to find:
A. A loose flap of torn skin on the left arm.
B. Bruised skin with a piece of bone protruding from the left arm.
C. Abraded skin with an underlying bone fracture.
D. A long and deep laceration on the left arm.

12. A 49-year-old man was climbing on rocks when he fell backward to the ground. He presents with a two-inch
linear wound on the top of his head. Bleeding has been controlled. You would document this type of injury as a
(n):
A. Contusion
B. Avulsion
C. Abrasion
D. Laceration

13. A patient has a laceration on his arm from a table saw. Assessment reveals dark red blood flowing slowly from
the wound. You would recognize this type of bleeding as:
A. Venous bleeding
B. Insignificant bleeding
C. Lymphatic bleeding
D. Arterial bleeding

14. Using a very narrow material such as wire or rope for a tourniquet can cause:
A. Serious tissue damage
B. Inadequate pressure to control bleeding
C. Venous pooling
D. Worse bleeding

C H A P T E R 18
15. Which of the following phrases best describes a closed-tissue injury?
A. A wound that has scabbed over or healed
B. A wound with no obviously visible opening, such as a needle stick
C. A wound in which underlying tissues are damaged but the overlying skin remains intact
D. A wound in which platelets and proteins have formed a plug

16. Assessment of a patient who fell reveals bruising to her right buttock. Her skin is intact and she complains of
tenderness in the area when you palpate it. You would recognize which of the following types of injury?
A. An abrasion
B. An avulsion
C. A contusion
D. An open injury

17. You are having a difficult time controlling bleeding from a small skin avulsion on a patient's ankle. Which of
the following statements made by the patient best explains why the bleeding has been so difficult to control?
A. "I drank a lot of alcohol last night."
B. "My blood pressure sometimes runs a little high."
C. "I take Coumadin (an anticoagulant) for my irregular heartbeat."
D. "I take steroids for my lung disease."

18. What is the function of subcutaneous fat?


A. It produces glucose needed for energy.
B. It insulates the body and stores energy.
C. It always projects underlying organs from trauma.
D. It has no real function and is unhealthy and detrimental to survival.

19. Which of the following statements indicates that the speaker understands the application of a tourniquet?
A. "A tourniquet should be applied only when directed by medical control.”
B. "Arteries run close to the body's surface in a joint, so the best place to apply a tourniquet is over the joint
just above the artery.”
C. "If the tourniquet appears to have stopped the bleeding, you should loosen it slightly to allow a small
amount of blood to perfuse the tissues.”
D. "A tourniquet should be applied tightly enough so that arterial blood flow distal to the tourniquet is
completely stopped."

20. You are responding to a call to aid a patient who has cut himself with a knife. Reportedly arterial bleeding is
involved. Given this information, which of the following findings do you expect?
A. Spurting dark-red blood that is difficult to control
B. Oozing bright-red blood that is easy to control
C. Flowing dark-red blood that is difficult to control
D. Bright-red blood that is spurting with each beat of the heart

21. A patient has been stabbed in the chest with a 6-inch-long ice pick. When you assess the wound, you find it
to be very small and insignificant in appearance, with minimal bleeding. Which of the following assumptions can
you correctly make about this injury?
A. Because the entrance wound is small, severe bleeding is highly unlikely.
B. The wound is most likely superficial.
C. Not enough information is available to make any assumptions.

C H A P T E R 18
D. Damage to the underlying structures is likely.

22. You are treating a patient who has cut herself while working in the lodge kitchen. She has a jagged laceration
on her left forearm that is bleeding steadily and heavily. While attempting to control the bleeding, you should
first:
A. Wrap the injury tightly with roller gauze and apply a tourniquet proximal to the laceration.
B. Place a sterile dressing over the site and hold steady pressure on the dressing with your gloved hand.
C. Apply direct pressure to the elbow pressure point to slow the flow of blood into the arm.
D. Tightly wrap roller gauze around a large sterile dressing placed over the injury.

23. Which of the following actions should you take next when direct pressure, elevation, and femoral artery
pressure have failed to control arterial bleeding on a patient lower leg?
A. Apply a tourniquet below the knee
B. Apply a tourniquet above the knee and splint the extremity.
C. Provide oxygen at 15 LPM with a nonrebreather mask.
D. Immediately transport the patient by ambulance.

24. You have applied a pressure dressing to the calf of a woman who suffered a deep laceration from a piece of
broken glass. Which of the following actions should you take next?
A. Apply a splint
B. Check CMS in the patient's foot
C. Apply a tourniquet above the laceration
D. Obtain a follow-up blood pressure reading

25. You have just arrived on scene, where a young girl has pushed her arm through a plate glass window. She is
responsive and has a patent airway. Bright red blood is spurting from a large laceration on her right upper arm.
When your partner tells you to apply direct pressure to the laceration to prevent the further blood loss, you
would:
A. Place a tourniquet proximal to the laceration, being careful not to get blood on your hands.
B. Apply direct pressure to the site with your hands until someone wearing gloves can take over.
C. Put gloves on before applying direct pressure over the injury site.
D. Control the bleeding by pushing the edges of the laceration together until you can put gloves on.

26. The skin performs which of the following functions?


A. Dissipation of internal cold
B. Prevention of blood loss
C. Elimination of carbon dioxide
D. Regulation of temperature

27. A patient has suffered abrasions to the legs after falling on a moving treadmill. In treating this injury, you
would be most concerned about:
A. Swelling and bruising
B. The loss of body heat
C. The formation of scar tissue
D. The risk of infection

28. The most effective method for treating a contusion is to apply a (n):
A. Ice pack

C H A P T E R 18
B. Tourniquet
C. Bandage
D. Pressure dressing

29. Which of the following statements about an impaled object is true?


A. OEC Technicians may remove objects that are impaled in the chest or abdomen.
B. OEC Technicians may remove only objects that are impaled in an extremity.
C. The primary treatment goal in the field is to stabilize the object in the position found.
D. The primary treatment in the field is to remove the object without causing further bleeding or causing as
little bleeding as possible.

30. During your secondary assessment, a patient with a stab wound to the abdomen tells you that he is taking
Coumadin, a blood thinner. Which of the following implications would taking this medication have for this
patient?
A. The medication would cause this patient's blood to clot more quickly, thereby minimizing any bleeding
that might occur.
B. The medication could cause the patient to bleed more profusely.
C. The medication would not affect the patient or the care you provide.
D. The medication would reduce the patient's blood pressure, so bleeding would be less profuse.

31. Which of the following statements indicates that the speaker understands bandaging?
A. "After you apply a bandage, a dressing should always be put over it."
B. "Bandaging material must always be sterile."
C. "Bandages are sterile gauze pads used to stop bleeding."
D. "Bandaging material is used to secure a dressing in place."

32. When managing a patient with soft-tissue injuries, it is essential that you:
A. Cover any hematomas with an occlusive dressing.
B. Control any bleeding with sterile dressings only.
C. Thoroughly clean all lacerations.
D. Use the appropriate personal protective equipment.

33. Severe external bleeding should be controlled during what phase of a patient assessment?
A. The rapid secondary survey
B. The primary assessment
C. The reassessment
D. The scene size-up

34. Which of the following statements indicates that the speaker understands dressing and bandaging a wound?
A. "If bleeding from a wound cannot be stopped initially with direct pressure, apply a bandage so you can take
care of other injuries.”
B. "Leave the tips of the fingers or toes exposed when bandaging an arm or a leg so you can continue to assess
CMS.”
C. "Place a bandage across the joint proximal to the wound because this helps control bleeding by applying
pressure to the underlying blood vessels.”
D. "When applying a bandage over a dressing, apply it very tightly so the bandage won't fall off."

C H A P T E R 18
35. Which of the following instructions is appropriate for helping a new patroller care for a patient with a very
large abrasion on his arm?
A. "It is best to let the wound bleed some. This is the body's natural way of cleaning the wound.”
B. "Do not cut away the sleeve over the top of the laceration because that will restart the bleeding.”
C. Wash the gross contamination off pouring on sterile saline, put on a sterile dressing, and recommend the
patient go to the hospital for further care.
D. "When you are done cleaning the abrasion, apply some sterile gauze soaked in sterile saline."

36. A young boy was out riding his bike in the neighborhood when he fell onto a stick protruding from the
ground. The stick impaled him in the neck. The boy immediately pulled the stick out. Assessment reveals a
gaping wound to the boy's right neck. Which of the following types of dressing would you use on this injury?
A. An occlusive dressing that is taped on all sides
B. A sterile dressing placed over the wound and bandaged with roller gauze around the neck
C. Sterile gauze carefully placed inside the open wound to control bleeding
D. A moist sterile dressing that is taped to the neck

37. Which of the following statements about the use of a tourniquet is true?
A. On an arm or leg, it should be placed 1 inch from either the elbow or knee.
B. It can be used on most any part of the body except the neck.
C. It should be placed as distal as possible but at least several inches proximal to the wound.
D. Tourniquets are a last resort and should be used rarely, if ever.

38. You arrive at a scene at which a 16-year-old girl who tells you she has cut her wrist with a knife. She appears
to be pale, and blood is spurting from the cut. Which of the following actions should you take first after making
the scene safe?
A. Obtain a blood pressure reading
B. Apply direct pressure to the cut
C. Apply oxygen at 15 LPM via a nonrebreather mask
D. Ask her how the injury happened

39. Which of the following statements about impaled objects not affecting the airway is true?
A. Allow only the patient to pull the impaled object out because that will hurt less.
B. The impaled object should be left in place during transport.
C. An impaled object never causes serious underlying tissue damage.
D. A wound made by an impaled object always bleeds profusely.

40. When assessing a patient you note a bruise on the chest. Another word for documenting a bruise is:
A. Erythematic
B. Hematoma
C. Cyanosis
D. Ecchymosis

Answer Key: Chapter 18 - Test

C H A P T E R 18
1=A, 2=C, 3=A, 4=A, 5=C, 6=B, 7=A, 8=B, 9=C, 10=C, 11=A, 12=D, 13=A, 14=A, 15=C, 16=C, 17=C,
18=B, 19=D, 20=D, 21=D, 22=B, 23=A, 24=B, 25=C, 26=D, 27=D, 28=A, 29=C, 30=B, 31=D, 32=D,
33=B, 34=B, 35=C, 36=A, 37=C, 38=B, 39=B, 40=D


C H A P T E R 18
C H A P T E R 19
“Burns”

QUIZ

1. Strong alkaline chemical burns tend to be more extensive than acid burns because:
A. Acid agents cling to the skin less than alkaline agents.
B. They are highly viscous.
C. Alkaline agents penetrate deeper into the skin.
D. Commercial cleaning agents are more concentrated than household ones.

2. A burn patient has a coarse and husky sounding voice. Based on this, you would be concerned:
A. About a depressed level of consciousness.
B. That the patient may be allergic to the smoke from the fire.
C. About the possibility of an airway burn.
D. That there may be a foreign body in the mouth.

3. The management of electrical burns includes:


A. A diligent search for internal injuries.
B. Moist dressings.
C. Ice packs on the entrance wound.
D. Antibiotic ointments.

4. You are called to the lodge kitchen where a 26-year-old cook has spilled boiling water on his left hand. He has
diffuse redness involving the entire hand and all but the tips of the fingers. Blisters are beginning to form.
Appropriate prehospital care for this injury includes all of the following EXCEPT:
A. Elevation of the wound above the level of the heart.
B. The application of a dry, sterile dressing.
C. Cooling the burn with cool water.
D. The application of antibiotic ointment.

5. You respond to the scene of a smoky fire in a maintenance shop, where you encounter the supervisor in the
parking lot. He tells you that he stayed behind to ensure that all his personnel made it out (which they did) and
made prolonged efforts to extinguish the fire. On exam, he has no evidence of burns but is complaining of
headache and nausea and seems a bit confused. You would first:
A. Interview the other personnel to find out how long the fire has been burning.
B. Look for evidence of substance abuse.
C. Enter the shop to be sure the fire is out.
D. Administer high-flow oxygen to the supervisor because of concern for carbon monoxide exposure.

C H A P T E R 19
Answer Key: Chapter 19 - Quiz
1=C, 2=C, 3=A, 4=D, 5=D

TEST

1. Alternating (bidirectional) current - current that pulses 60 times per second - is more dangerous than direct
current because the pulses:
A. Cause external injuries
B. Create higher voltage
C. can cause the victim to remain in physical contact with the source of the current for a longer time leading
to more significant damage
D. Produce higher amperage

2. In treating a burn patient, you should cool and irrigate the burned tissue with:
A. tap water warmed to 95 degrees F for 10 minutes.
B. chilled tap water for 5 minutes.
C. tap water warmed to 102 degrees F for 15 minutes.
D. room-temperature tap water for 15 minutes.

3. Another simple method for estimating the total percentage of area burned is the ______ approach
One percent of body area is roughly the size of a _____
A. A peach
B. Fist
C. Three-fingers
D. Palm

4. Which of the following statements concerning a patient who received electrical burns to his hand after
grabbing a live wire is true?
A. Electrical burns are a minor injury.
B. The patient will remain an electrocution hazard to rescuers for several minutes after being removed from
the source of electricity.
C. The burning will continue for hours, perhaps days.
D. The extent of tissue damage may be much greater than it appears on the surface.

5. A burn extending through the epidermis and dermis layers into the subcutaneous layer would be classified as a:
A. Full-thickness burn (third or fourth degree burn).
B. Deep partial-thickness burn (second degree burn).
C. Superficial partial-thickness burn (second degree burn).
D. Superficial burn (first degree burn).

6. For the OEC, the first priority in managing a patient with a thermal burn is:
A. assessing for airway compromise
B. applying dry sterile dressings
C. completing a secondary survey
D. stopping the burning process to prevent further injury

C H A P T E R 19
7. Thermal burns caused by the skin directly contacting hot water or hot steam:
A. Is the most common type of burn
B. Result from tanning beds
C. Is the least common type of burn
D. Result from exposure to Gamma radiation

8. When treating a patient exposed to strong acids or bases, which of the following devices offers the least
protection to an OEC Technician?
A. A face shield
B. Exam gloves
C. An impervious apron
D. Goggles

9. For the OEC, the first priority in managing a patient with an electrical burn is:
A. Assessing for airway compromise
B. Assessing for potential c-spine injury
C. Ensuring the power source has been turned off
D. Ensuring you have an AED available if needed

10. In treating a burn patient, you should cool and irrigate the burned tissue with:
A. Room-temperature tap water for 15 minutes.
B. Chilled tap water for 5 minutes.
C. Tap water warmed to 102 degrees F for 15 minutes.
D. Tap water warmed to 95 degrees F for 10 minutes.

11. When treating a patient burned by a dry chemical, it is important to brush away any dry residue before
flushing with water because:
A. Flushing will cause the skin's pores to open.
B. Flushing will expose other areas around the burn.
C. A containment area needs to be set up to catch the runoff water.
D. Some dry chemicals are activated by water.

12. Thermal burns result from:


A. Direct contact between a heat source and the skin
B. Indirect contact between a heat source and the skin
C. A chemical reaction stimulated by a heat source
D. Transmitted Gamma radiation

13. Your patient is a 35-year-old woman who spilled a cup of hot coffee on herself two hours ago. An area on her
right thigh that is about twice the size of the palm of her hand is red and painful but has no blisters. Which of the
following actions would be appropriate for an OEC Technician to take?
A. Apply a plastic bag full of ice to the skin of the affected area
B. Apply a dry sterile dressing
C. Apply an antibiotic ointment
D. Apply a lotion containing a topical anesthetic and aloe vera

14. Your patient is a 40-year-old man who was burned when he spilled gasoline on his pants while he was
standing near the pilot light of his hot-water heater. He has partial thickness burns from his feet to just above his

C H A P T E R 19
knees, circumferentially around both legs. According to the Rules of Nines, the man burned ____ of his body
surface.
A. 4.5 percent
B. 18 percent
C. 36 percent
D. 9 percent

15. When treating a patient burned by a dry chemical, it is important to brush away any dry residue before
flushing with water because:
A. flushing will expose other areas around the burn.
B. some dry chemicals are activated by water.
C. a containment area needs to be set up to catch the runoff water.
D. flushing will cause the skin's pores to open.

16. Which of the following sentences best describes a partial-thickness burn?


A. The skin is charred or blackened and lacks sensation.
B. The skin is red and moist, and blisters have formed.
C. The skin is white and dry with no sensation of pain.
D. The skin is red, but dry, no blisters formed and painful.

17. When treating a patient exposed to strong acids or bases, which of the following devices offers the least
protection to an OEC Technician?
A. Goggles
B. A face shield
C. Exam gloves
D. An apron

18. Chemical burns result from exposure to:


A. Caustic substances
B. Steam
C. Hot light bulbs
D. Ultraviolet light

19. Which of the following substances cause the most serious chemical burns?
A. An acid with a pH of 1
B. An alkaline chemical or base with a pH of 8
C. A chemical with a pH of 7
D. An acid with a pH of 6

20. Hoarseness or voice changes in a burn patient should alert an OEC Technician that the:
A. patient most likely is experiencing an inhalation injury causing swelling to the airway, a true emergency
B. patient may be extremely anxious about the injury.
C. Patient should be treated as a pediatric patient.
D. Patient has altered mental status.

21. Your patient is a 10-year-old boy who was exposed to a dry chemical powder and is complaining of severe pain
at the site of contact on both of his hands. There is no decontamination shower on site. Which of the following
actions would be the best way to manage this situation?

C H A P T E R 19
A. Brush away as much of the powder as possible and then have the patient hold his hands under running
water from a faucet or garden hose.
B. Brush away as much powder as possible and then pour a bottle of sterile saline solution over the patient's
hands.
C. Have the fire department connect to a hydrant and spray the patient down from head to toe.
D. Brush away the powder and then bandage the patient's hands in a position of function.

22. Exposure to direct or alternating electrical current produces severe external AND internal burn injuries
because:
A. The skin is a poor conductor with a high resistance, and blood vessels are good conductors with low
resistance.
B. Both the skin and blood vessels are good conductors with low resistance.
C. The skin is a good conductor with a high resistance, and blood vessels are good conductors with low
resistance.
D. Skin is a poor conductor with a low resistance, and blood vessels are good conductors with low resistance.

23. Your patient is a 25-year-old man who has a reddened area with blisters across the palm of his hand after
grabbing the handle of a very hot iron skillet. Which of the following actions must be avoided in the prehospital
management of this wound?
A. Applying a dry, sterile dressing
B. Monitor for shock
C. Cooling the burn with cool water
D. Applying an antibiotic ointment

24. According to the Rule of Nines, the percentages of total body surface area that are taken up by the heads
(front and back) of adults and children are:
A. 9 percent and 18 percent, respectively
B. 4.5 percent and 9 percent, respectively
C. The same
D. 18 percent and 36 percent, respectively

25. When managing electrical burns, OEC Technicians should always:


A. Use antibiotic ointments
B. Assess for signs and symptoms of internal injuries
C. Use a topical ointment on the skin
D. Use moist dressings

Answer Key: Chapter 19 - Test


1=C, 2=D, 3=D, 4=D, 5=A, 6=D, 7=A, 8=B, 9=C, 10=A, 11=D, 12=A, 13=B, 14=B, 15=B, 16=B, 17=C,
18=A, 19=A, 20=A, 21=A, 22=A, 23=D, 24=A, 25=B


C H A P T E R 19
CHAPTER 20
“Musculoskeletal Injuries”

QUIZ

1. While attempting to place a deformed limb back into the anatomic position, the patient screams and begs you
to stop because of the pain. Your best response is to:
A. Stop and splint the limb in the current position.
B. Ignore the patient and finish the alignment.
C. Apply additional tension and make another attempt.
D. Have the patient take a deep breath and continue the reduction as quickly and efficiently as you can.

2. The purpose of field splinting an MS injury includes all but which of the following?
A. Preventing further injury and blood loss
B. Decreasing pain at the fracture site
C. Allowing the bone ends to knit back together
D. Helping with muscle spasms by placing the limb in an anatomic position

3. Hinge, gliding, pivot, and suture refer to what?


A. Types of joints
B. Types of irregular and flat bones
C. Types of fracture immobilization devices
D. Types of fractures

4. A patient has stretched and torn the medial malleolar ligament. This means the patient has:
A. Sprained her ankle.
B. Torn the meniscus in her knee.
C. Torn the rotator cuff in her shoulder.
D. Damaged the popliteal joint.

5. A patient has suffered a femur fracture. The greatest danger with this injury is:
A. Amputation.
B. Destruction of the vastus muscle.
C. Severe blood loss.
D. Permanent disability.

6. A patient exhibits the following sings and symptoms: respiratory difficulty, difficulty swallowing, upper
extremity and facial vein engorgement, red face, and shock. What injury is most likely causing these signs and
symptoms?
A. A sternal fracture
B. A heart attack
C. A scapulary fracture

CHAPTER 20
D. A posterior sternoclavicular dislocation

7. The most important component of field management for musculoskeletal injuries is:
A. Correctly immobilizing the injury.
B. Not exacerbating the injury.
C. Correctly diagnosing the injury.
D. Personal safety.

8. The axial skeleton is responsible for ________________, while the appendicular skeleton provides
__________________________.
A. Strength and endurance, for upright positioning
B. Spinal stability, for the movement of hinge joints
C. The upright position of the body, the manipulation of objects or locomotion
D. Production of red blood cells, production of white blood cells

9. A patient with a dislocated elbow has no CMS in his hand. One attempt at realignment fails to regain
circulation. Your next best course of action would be to:
A. Gently force the arm into the anatomic position, apply heat packs to the hand, splint the arm with a rigid
splint, sling and swathe as appropriate, and transport your patient immediately.
B. Make a second attempt.
C. Secure the arm as best as you can with a rigid splint, sling and swathe as appropriate, administer oxygen,
and transport rapidly.
D. Secure the arm to a tension splint and extricate your patient.

10. Which of the following most dramatically slows the healing time of bone?
A. Smoking
B. Aging
C. Drinking
D. Nutrition

11. One of the most common assessment mistakes is to:


A. Misdiagnose an injury.
B. Focus on the less-serious "obvious" injury and miss a potential life threat.
C. Underestimate the severity of an injury.
D. Incorrectly identify an anatomical landmark.

12. Skeletal and smooth refer to what?


A. Joints
B. Tendons
C. Ligaments
D. Muscles

13. A patient has suffered an obvious femur fracture at the proximal end of the femur, near the femoral head. The
proper splint for this type of injury would be a:
A. Traction splint.
B. Kendrick extrication device (KED) or SAM splint.
C. Quick splint.
D. Long spine board.

CHAPTER 20
14. You notice crepitus when attempting to move a patient's injured wrist back into an anatomically correct
position. This is indicative of what type of injury?
A. A dislocation
B. A sprain
C. A strain
D. A fracture

15. You are caring for a patient with an open humeral fracture. Extrication to definitive medical care will take at
least 12-14 hours. The proper method of care for this injury includes:
A. Wash the fracture site, apply tension until the bone end has slipped back into the body, and then bandage
and splint as appropriate.
B. Secure the humerus with a rigid splint, leave the bone end exposed, and cover it lightly with sterile gauze.
C. Treat the exposed bone end as you would an impaled object and bandage it in place, moisten the dressing
with sterile saline, and then splint the arm.
D. Place a cup over the exposed bone, secure it in place with roller gauze or Kling, and then splint as usual.

16. The MOST important step in applying any splint is to:


A. Apply appropriate tension when reducing the fracture.
B. Pad the splint appropriately.
C. Place the limb in the anatomically correct position.
D. Check distal CMS before and after splinting.

17. Which of the following injuries is generally the most serious?


A. A clavicular fracture
B. A dislocated patella
C. A dislocated knee
D. An anterior shoulder dislocation

18. A patient tells you she felt a "pop" behind her right ankle but there is little or no pain, and she cannot move or
walk on the foot. This is most likely indicative of what type of injury?
A. A strain
B. A sprain
C. A fibula fracture
D. A ruptured tendon

Answer Key: Chapter 20 - Quiz


1=A, 2=C, 3=A, 4=A, 5=C, 6=D, 7=D, 8=C, 9=C, 10=A, 11=B, 12=D, 13=D, 14=D, 15=A, 16=D, 17=D,
18=D

TEST

1. Which of the following terms does not describe a type of bone?


A. Flat

CHAPTER 20
B. Short
C. Long
D. Irregular

2. Your friend tells you that he wants to try snowboarding. He is concerned about his safety and asks you which
type of fracture is most common in snowboarders. You learned in your OEC training that this injury involves the:
A. radius.
B. elbow.
C. tibia.
D. humerus.

3. A fracture that has three or more fragments is called a (n):


A. Impacted fracture
B. Oblique fracture
C. Comminuted fracture
D. Pathologic fracture

4. Which of the following statements about the axial skeleton is true?


A. It has 126 bones consisting of the skill, vertebrae, and pelvis.
B. It has 80 bones consisting of the skull, vertebrae, and pelvis.
C. It has 60 bones consisting of the skull, vertebrae, and thoracic cage.
D. It has 80 bones consisting of the skull, vertebrae, and thoracic cage.

5. You are evaluating a new OEC instructor who is teaching a class on displaced humerus fractures. Which of the
following statements by the instructor would require your correction?
A. "Gentle longitudinal tension should be applied down from the shoulder in line with the normal axis of the
humerus while the upper arm lies along the side of the patient's body.”
B. "The splint should be long enough to reach the distal palm crease, which immobilizes the wrist.”
C. "The patient's forearm should be placed with the elbow at a 90-degree angle and in contact with the
patient's abdomen.”
D. "The fracture should be reduced to achieve better pain control and packaging."

6. The finding that upon muscle contraction the active motion of a joint is reduced or lost suggests:
A. A ruptured tendon
B. Torn cartilage
C. A dislocation
D. A ligament strain

7. A vacuum splint works because:


A. air expands the beads, forming a rigid splint.
B. temperature changes to the inside air space cause the unit to shrink and become rigid.
C. the splint becomes rigid when air is sucked out of it.
D. air causes a chemical reaction involving the beads inside, causing the unit to become rigid.

8. A common injury to the carpal bones among snowboarders who fall forward on an outstretched hand is a
fractured:
A. Ulna
B. Radius

CHAPTER 20
C. Thumb
D. Scaphoid

9. A patient for whom you recently provided care stops by to thank you. He reports that he was diagnosed with a
strain to his lower right leg. Which of the following structures is the primary structure affected in that injury?
A. A bone
B. A muscle
C. A ligament
D. A joint

10. A fracture of a bone will cause bleeding, and a hematoma forms around the fracture site. Over the next several
weeks, this hematoma organizes into a substance called:
A. cartilage.
B. calcium.
C. a synovium.
D. a callus.

11. Treatment of an anterior S/C dislocation is best managed using a:


A. sling and swathe.
B. rigid splint.
C. figure eight splint.
D. blanket roll.

12. One method for immobilizing a clavicle fracture is a figure eight splint. Which of the following statements
about a figure eight splint is false?
A. It should not be used for fractures of the lateral one-third of the clavicle.
B. It is used to immobilize fractures of the proximal and middle thirds of the clavicle.
C. This splint is useful for acromioclavicular (A/C) injuries.
D. This splint may immobilize the clavicle better than a sling and swathe does.

13. Which of the following musculoskeletal tissues does not heal?


A. A muscle
B. A cartilage
C. A tendon
D. A ligament

14. Your patient was struck in the right upper arm with a baseball bat. Which of the following signs or symptoms
indicates the highest probability that the humerus has been fractured?
A. Pain in the right upper arm
B. Swelling in the upper arm
C. Feeling or hearing bone ends moving on palpation
D. Decreased sensation in the right hand

15. You and a fellow patroller are applying a blanket roll to a 48-year-old patient with an anteriorly dislocated
shoulder. While your partner holds the blanket roll in position under the affected shoulder, which of the following
actions should you not take in securing the roll?
A. Tie one of the cravats over the affected shoulder.
B. Stabilize the patient's hand and forearm on the blanket using two cravats.

CHAPTER 20
C. Tie one of the cravats over the patient's opposite shoulder and around the neck.
D. Tie one of the cravats around the patient's waist.

16. Displacement of the bones of a joint is known as:


A. A sprain
B. A strain
C. A fracture
D. A dislocation

17. A fracture of a bone will cause bleeding, and a hematoma forms around the fracture site. Over the next several
weeks, this hematoma organizes into a substance called:
A. Calcium
B. A synovium
C. Cartilage
D. A callus

18. Which of the following statements indicates that an OEC Technician has a good understanding of what a joint
dislocation is?
A. "A distal pulse is absent, only in the case of a fracture."
B. "Patients with a joint dislocation experience very little pain."
C. "If a joint is dislocated the patient can move it without pain."
D. "A dislocation is a separation or displacement of the bones of a joint."

19. A fracture that has three or more fragments is called a(n):


A. impacted fracture.
B. oblique fracture.
C. comminuted fracture.
D. pathologic fracture.

20. You are caring for a patient who has an angulated lower leg fracture. To splint this injury you should:
A. splint the leg in the position found.
B. use gentle longitudinal tension to align the fracture.
C. apply 5-10 lbs of traction by pulling on the patient's foot.
D. apply an air splint, which when inflated will align the leg.

21. The most commonly broken bone in the body is:


A. a rib.
B. the clavicle.
C. the ulna.
D. the tibia.

22. The inside layer of a joint capsule where cells make a viscous fluid for lubricating the joint is the:
A. fascia.
B. cartilage.
C. callus.
D. synovium.

23. The outermost part of a bone is a tough lining known as:

CHAPTER 20
A. Articular cartilage.
B. The cortex.
C. Synovium.
D. The periosteum.

24. Which of the following functions is not one of the several functions of bones?
A. Providing structure and form for body tissues
B. Protecting vital organs
C. Producing red blood cells
D. Responding to sensory nerve stimulation to protect skin and other tissues

25. Which of the following terms does not describe a type of joint?
A. Ball and socket
B. Post
C. Gliding
D. Hinge

26. An airplane splint works well because:


A. it is waterproof and thus suitable for outdoor use.
B. it can be moved to conform to the angle of any injured joint.
C. it is a rigid, fixed stabilizing unit.
D. its design provides traction.

27. The _____________ can be injured when a hip is dislocated, resulting in numbness or paralysis of a lower
extremity.
A. Cranial nerve
B. Sciatic nerve
C. Central nervous system
D. Tibial nerve

28. A snowboarder's fall over the toe side of the board onto an outstretched hand is known as the:
A. "drop back."
B. "mousetrap."
C. "twist."
D. "dog drag."

29. Which of the following pairs of proteins causes muscle tissue to shorten or contract?
A. Leucine and lysine
B. Actin and myosin
C. Glutamine and glycine
D. Alanine and arginine

30. Which of the following statements about the appendicular skeleton is true?
A. It has 80 bones consisting of the shoulder, arm, pelvis, and legs.
B. It has 126 bones consisting of the shoulder, arm, pelvis, and rib cage.
C. It has 126 bones consisting of the shoulder, arm, pelvis, and legs.
D. It has 126 bones consisting of the arm, pelvis, rib cage, and legs.

CHAPTER 20
31. You are working with a candidate OEC Technician who is practicing the application of splints. When checking
the sling and swathe that the candidate applied to an upper extremity, you note that it is not applied correctly.
You point this out to the candidate and tell him that the purpose of a sling and swathe is to:
A. prevent further movement of an injured extremity.
B. prevent the patient from using an injured arm when getting into a toboggan.
C. hold an injured arm up so that the patient doesn't need to support it.
D. hold a wooden splint in place.

32. An air splint is a dual-walled, tube-shaped device used to temporarily immobilize a long bone. Which of the
following statements about air splints is false?
A. They are inexpensive and easier to clean than other types of splints.
B. They are made of a tough fibrous material that is difficult to puncture.
C. They are most often used to immobilize injuries located distal to the elbow or the knee.
D. They may help control bleeding by applying constant, external pressure to a wound.

33. The type of muscle found in the forearm is:


A. Skeletal muscle
B. Smooth muscle
C. Contractor muscle
D. Cardiac muscle

34. Which of the following splints should not be used for a wrist fracture?
A. A SAM splint
B. A Thomas Splint
C. A soft splint
D. An air splint

35. A snowboarder's fall over the toe side of the board onto an outstretched hand is known as the:
A. "Mousetrap"
B. "Twist"
C. "Drop back"
D. "Dog drag"

36. To properly apply a figure eight splint, an OEC Technician should:


A. not use a cravat that is wider than 1 inch.
B. tighten the figure eight until the patient's shoulders are pulled back tightly.
C. tighten the cravats so that the position of the shoulders is the same as if the patient were sitting normally.
D. place the cravat directly over the fracture to splint it.

37. A 42-year-old man has fallen 25 feet while rock climbing. He hit the ground feet first and suffered open
fractures to both tibias, which are protruding through the skin. When you arrive, you note the scene is safe, the
patient is responding to painful stimuli. His airway is open, his breathing is adequate, his radial pulse is strong
and rapid and your patient is alert and oriented. At this point in his care, it is a priority for you to:
A. Obtain a medical history from his friends.
B. Cover him to help prevent shock.
C. Preform a secondary survey to look for other injuries.
D. Immobilize the fractures.

CHAPTER 20
38. The most frequent injury in skiing is a:
A. Concussion
B. Knee sprain
C. Fractured radius
D. Thumb strain

39. The structure responsible for transmitting the force of a contracting skeletal muscle to a bone is a:
A. Ligament
B. Tendon
C. Joint
D. Cartilage

40. A common injury to the carpal bones among snowboarders who fall forward on an outstretched hand is a
fractured:
A. thumb.
B. scaphoid.
C. ulna.
D. radius.

41. The finding that upon muscle contraction the active motion of a joint is reduced or completely lost suggests:
A. a dislocation.
B. a ruptured tendon.
C. torn cartilage.
D. a ligament strain.

42. Care of an anterior dislocated shoulder:


A. can be achieved using a quick splint.
B. is easier than is care for any other upper extremity injury.
C. requires patience and sometimes creativity.
D. requires only a sling and swathe to keep the arm in the position found.

43. Can a Colles' fracture or "silver fork" wrist fracture result from falling on a clenched fist?
A. Yes
B. No

44. You respond to an accident and find a young man lying prone but in a neutral, anatomic position, except that
his head is turned to the side. Using the principles of Jams and Pretzels, you would say that this patient is in
position:
A. 3A.
B. 2A.
C. 1A.
D. 3.

45. Which of the following splints should not be used to treat proximal forearm fractures?
A. A ladder splint that goes up to the axilla and down to the palm crease
B. A sugar tong splint wrapped around the elbow and extending down to the palm crease
C. A splint that does not immobilize the elbow
D. A splint that prevents rotation of the forearm at the elbow

CHAPTER 20
46. Which of the following terms describes a type of joint except?
A. Hinge
B. Post
C. Gliding
D. Ball and socket

47. An injured forearm, shoulder, clavicle, or scapula should be cradled in a sling:


A. at the same level as the elbow.
B. with its weight evenly distributed.
C. at a 45-degree angle to the chest.
D. with the splinted extremity elevated at the level of the clavicle.

48. The outermost part of a bone is a tough lining known as:


A. the cortex.
B. the periosteum.
C. synovium.
D. articular cartilage.

49. You respond to a 9-year-old boy who was injured in the terrain park. He is complaining of pain in his right
wrist. Upon examination you note deformity and swelling of the right wrist, pinkness of the right hand, and a
strong radial pulse. Which of the following actions would be part of the proper care for this boy?
A. Straighten the wrist to promote blood flow to the hand.
B. Maintain the wrist below the level of the heart to decrease swelling.
C. Apply a cold pack to the wrist to reduce swelling.
D. Massage the wrist gently to decrease the pain.

50. You are treating a patient who is complaining of moderate pain in the right knee. The knee is swollen,
discolored, and flexed about 45 degrees. CMS is intact. How would you treat this patient?
A. Apply a traction splint and straighten the knee until the pain is decreased.
B. Straighten the injured knee and then immobilize it by attaching it to the other leg.
C. Splint the knee in the position found before moving the patient.
D. Place the patient on a long spine board and transport to the aid room, where you can examine the injury
more efficiently.

51. While you are putting wood into your wood stove, your hand comes in contact with the hot surface. Your
body responds by pulling your hand away from the heat. How is this action accomplished?
A. One or more skeletal muscles receive a signal from the brain to contract.
B. Extension of the ligaments in the elbow joint pulls the lower arm up.
C. The smooth muscles in the arm automatically respond to heat.
D. You use conscious thought to pull your hand from the heat.

52. A sling and swathe can be used for a:


A. potential ulna fracture without the need for a separate splint.
B. rib fracture.
C. potential wrist fracture without the need for a separate splint.
D. humerus fracture if combined with a separate splint.

CHAPTER 20
53. A former patient returns to the Ski Patrol treatment room to thank you for the treatment he received. He
informs you that he has a popiteal fossa injury. With your OEC knowledge, you realize that his injury is related to
which of the following structures?
A. The wrist
B. The back of the knee
C. The lumbar muscles
D. The ankle

54. Which of the following musculoskeletal tissues does not heal?


A. A muscle
B. A tendon
C. A cartilage
D. A ligament

55. Your secondary assessment of a patient who is complaining of pain in his right leg reveals an opening in the
skin where the fractured tibia broke through the skin and retreated back into the leg. You would recognize this
injury as a (n):
A. Closed fracture
B. Partial fracture
C. Open fracture
D. Laceration

56. patient for whom you recently provided care stops by to thank you. He reports that he was diagnosed with a
strain to his lower right leg. Which of the following structures is the primary structure affected in that injury?
A. A bone
B. A muscle
C. A ligament
D. A joint

57. Joints provide different degrees of movement. There are five different types of joints. An example of a "pivot"
joints is the:
A. Vertebrae of the spine.
B. Knee.
C. Acromio-clavicular joint of the shoulder.
D. Radioulnar joint of the elbow.

58. Should a fractured humerus be splinted using a soft splint and a sling and swathe?
A. Yes
B. No

59. The structure responsible for transmitting the force of a contracting skeletal muscle to a bone is a:
A. joint.
B. ligament.
C. tendon.
D. cartilage.

60. Joints provide different degrees of movement. There are five different types of joints. An example of a "pivot"
joints is the:

CHAPTER 20
A. radio-ulnar joint of the elbow.
B. vertebrae of the spine.
C. acromio-clavicular joint of the shoulder.
D. knee.

61. When it becomes apparent that the arm distal to an elbow injury has a CMS deficit, OEC Technicians should:
A. make one attempt to realign and restore CMS before splinting.
B. attempt to realign the injury when final definitive care by a physician is less than two hours away.
C. use only a sling so that no pressure that further compromises CMS is put on the elbow.
D. splint the arm with the elbow slightly flexed.

62. During your SAMPLE inquiry, a patient reports that he had surgery to repair a torn ligament. Based on your
training, you know that a ligament is:
A. A tissue that stabilizes two contiguous bones ends.
B. Connective tissue that connects muscle and bone.
C. A thick muscle that surrounds a joint.
D. A muscle that connects to a bone.

63. You are demonstrating boot removal to a class of OEC candidates. While explaining Rescuer #1's role, you
indicate that his first task is to:
A. unbuckle or unlace the boot.
B. remove the boot.
C. check the patient's CMS.
D. stabilize the patient's leg and ankle

64. Which of the following statements would you make to the parent of a 10-year-old snowboarder who took a
hard fall and complains of some discomfort in a wrist that is slightly swollen but has no deformity?
A. "This is probably a pathologic fracture."
B. "This looks like an open fracture."
C. "It's probably just a sprain."
D. "The child needs to be medically evaluated to ensure there is not a fracture."

65. You are dispatched to aid a snowboarder who has sustained a shoulder injury. The injured man is sitting on
the side of the trail and reports that because of past dislocations of the shoulder he has a subluxation, and that
the shoulder is okay. Which of the following statements best describes his condition?
A. The shoulder has an anterior dislocation.
B. The shoulder joint partially dislocates and then returns to a normal anatomic position.
C. The shoulder has become dislocated but with can be managed with prescription pain drugs.
D. The shoulder has a posterior dislocation.

66. While you are putting wood into your wood stove, your hand comes in contact with the hot surface. Your
body responds by pulling your hand away from the heat. How is this action accomplished?
A. Extension of the ligaments in the elbow joint pulls the lower arm up.
B. The smooth muscles in the arm automatically respond to heat.
C. You use conscious thought to pull your hand from the heat.
D. One or more skeletal muscles receive a signal from the brain to contract.

67. Which of the following statements about skeletal (voluntary) muscles is true?

CHAPTER 20
A. They generally connect to the skeletal system.
B. They are mostly located on the torso of the body.
C. They have the ability to cause movement by extending.
D. They make up less mass in the body than do involuntary muscles.

68. In order to make a SAM splint


font size decreased by 1 Superscript font size decreased by 1 font size decreased by 1 TM
TM a "rigid" splint, you need to:
A. fold it in a double thickness.
B. use it in combination with a wooden splint.
C. use it in a sugar-tong configuration.
D. make a "T" shape lengthwise in its middle.

69. Which of the following statements about musculoskeletal injuries is true?


A. A fracture occurs when a tendon connecting a muscle to a bone is overstretched and becomes injured.
B. A sprain is an injury to a joint that involves the stretching or tearing of ligaments.
C. A dislocation occurs when a bone that forms part of a joint is broken.
D. Strains are injuries that occur to ligaments and to the joints to which they are attached.

70. The most frequent injury in skiing is a:


A. fractured radius.
B. knee sprain.
C. concussion.
D. thumb strain.

71. The ideal position for patients before you transfer them to a long spine board for immobilization is:
A. on one side in a neutral anatomic position, with the back straight, the eyes facing forward, the lower arm
extended above the head, and the upper arm at the side with the palm against the thigh.
B. supine, in a neutral anatomic position, with the back straight, the eyes facing forward, and the extremities
straight with the palms against the thighs.
C. prone in a neutral anatomic position, with the back straight, the eyes facing forward, and the extremities
straight with the palms against the sides of the thighs.
D. on one side in a neutral anatomic position, with the back straight, the eyes facing forward, and the
extremities straight with the palms against the sides of the thighs.

72. Which of the following terms describes a type of bone except?


A. Long
B. Irregular
C. Flat
D. Short

73. The inside layer of a joint capsule where cells make a viscous fluid for lubricating the joint is the:
A. Cartilage
B. Callus
C. Synovium
D. Fascia

CHAPTER 20
74. Your friend tells you that he wants to try snowboarding. He is concerned about his safety and asks you which
type of fracture is most common in snowboarders. You learned in your OEC training that this injury involves the:
A. Humerus
B. Tibia
C. Radius
D. Elbow

75. An airplane splint is useful for immobilizing certain types of musculoskeletal injuries. Which of the following
statements concerning an airplane splint is not correct?
A. It is considered a rigid splint.
B. It can be used to immobilize either shoulder injuries or knee injuries.
C. It is used only for injuries that must be maintained at a 90-degree angle.
D. It is generally considered more difficult to use than a blanket roll.

76. Because of the abundant blood vessels and nerves in the elbow, an injury to a child's elbow should be splinted
in:
A. a 90-degree angle.
B. the position found.
C. a 45-degree angle.
D. an elevated position using a blanket roll.

77. The National Ski Patrol's current recommendation regarding ski boot removal when applying a traction splint
is:
A. to leave the boot on if the patient is to be placed in a traction splint in the outdoor environment, unless the
local medical director approves doing otherwise.
B. to leave the boot on until you have at least two other patrollers to help with its removal.
C. that removal is determined by the weather conditions and the extent of the patient's injuries.
D. that removal should occur in a warm environment, so it should occur as soon as possible after you reach
the first aid room.

78. Which of the following statements would you make to the parent of a 10-year-old snowboarder who took a
hard fall and complains of some discomfort in a wrist that is slightly swollen but has no deformity?
A. "This is probably a pathologic fracture."
B. "It could be a nondisplaced fracture and needs to be medically evaluated."
C. "This looks like an open fracture."
D. "It's probably just a sprain."

79. You are caring for a 26-year-old woman with a probable wrist fracture. Before applying a splint you ask her to
remove her large engagement ring. She hesitates and asks why she should do this. Which of the following
responses would be an OEC Technician's best response?
A. "You should remove the ring now because your fingers may become very swollen."
B. "The metal in the ring will interfere with taking X-rays at the hospital."
C. "You don't want to lose your ring when they are caring for you at the hospital."
D. "The diamond could get caught in the splint and ruin the ring."

80. The most commonly dislocated joint in the body:


A. The elbow
B. The shoulder

CHAPTER 20
C. The thumb
D. A knee

Answer Key: Chapter 20 - Test


1=B, 2=A, 3=C, 4=D, 5=D, 6=A, 7=C, 8=D, 9=B, 10=D, 11=A, 12=C, 13=B, 14=C, 15=A, 16=D, 17=D,
18=D, 19=C, 20=B, 21=B, 22=D, 23=D, 24=D, 25=B, 26=C, 27=B, 28=B, 29=B, 30=B, 31=A, 32=B, 33=A,
34=B, 35=A, 36=C, 37=C, 38=B, 39=B, 40=B, 41=B, 42=C, 43=N, 44=D, 45=C, 46=B, 47=B, 48=B, 49=C,
50=C, 51=A, 52=D, 53=B, 54=C, 55=C, 56=B, 57=D, 58=N, 59=C, 60=A, 61=A, 62=A, 63=D, 64=D,
65=B, 66=D, 67=A, 68=D, 69=B, 70=B, 71=B, 72=D, 73=C, 74=C, 75=C, 76=B, 77=A, 78=B, 79=A, 80=B


CHAPTER 20
C H A P T E R 21
“Head and Spine Injuries”

QUIZ

1. Motor, balance, movement, and posture are controlled by the:


A. Thalamus.
B. Cerebrum.
C. Cerebellum.
D. Brainstem.

2. The terms linear, depressed, and basilar refer to which of the following conditions?
A. Neurologic function
B. Skull fracture
C. TBI
D. Stroke

3. Cerebral edema is most likely caused by:


A. Compression of white or gray matter.
B. Leaking of CSF from the ears or nose.
C. An increase in volume (blood or swelling) within the skull.
D. An excess of cerebrospinal fluid (CSF).

4. "Two temporal events" of brain injuries refers to which of the following?


A. Being struck or injured on both hemispheres of the brain at once, such as with bullet penetration or a
crushing injury
B. A secondary, fatal minor concussion on top of an earlier, more significant one
C. A coup-contra-coup injury
D. The primary injury of a collision and the secondary injury of hypoxia and increasing ICP

5. All of the following concerning the CNS are true EXCEPT:


A. Central nervous system injuries have a higher incident of mortality or morbidity than peripheral nervous
injuries.
B. The largest collection of neurons is located in the spinal column.
C. Gray matter involves regions of the brain that control muscle activity, sensory perception, hearing,
memory, emotions, and speech.
D. White matter is largely composed of long nerves and regulates autonomic functions such as temperature,
heart rate, and blood pressure.

C H A P T E R 21
Answer Key: Chapter 21 - Quiz
1=C, 2=B, 3=C, 4=D, 5=B

TEST

1. You suspect that an unhelmeted climber who struck his head during a fall may have a basilar skull fracture. As
you conduct your assessment, which of the following findings reinforces this suspicion?
A. Paralysis of the left arm and left leg
B. Dilation and sluggish response to light of the right pupil
C. Blood coming from both ears
D. Clear fluid coming from the right ear and left nostril

2. Based on the structure of the nervous system, which of the following statements is true?
A. A deep laceration to an arm can sever peripheral nerves.
B. Numbness of a leg secondary to a fracture is caused by a central nervous system injury.
C. A gunshot wound to the spinal cord will directly damage the peripheral nerves.
D. A stroke represents an injury to the peripheral nervous system.

3. Which of the following signs is a characteristic sign that may be seen during a secondary assessment of
patients with spinal cord injury?
A. Normal skin color and complaint of headache and nausea
B. Flushed skin color at the level of pinpoint back pain
C. Flushed skin color below the level of the injury and impairment or absence of sensation and movement
D. Bruising on the back and on the abdomen at the level of the injury, with normal or impaired sensation

4. For a person to make a fist, which of the following must occur?


A. The central nervous system must send a message through the spinal cord and then through the peripheral
nerves.
B. The peripheral nervous system must send a message to the brain and then back through the peripheral
nervous system.
C. The muscles must signal the brain to send a message through the autonomic nervous system.
D. The central nervous system must send a message through the spinal cord and then through the autonomic
nervous system.

5. For which of the following patients involved in a motor vehicle collision is the use of a vest-type short
immobilization device indicated?
A. A 33-year-old female in the backseat who complains of midline posterior neck pain and a headache
B. A 15-year-old female complaining of neck and back pain who got out of the car and is standing next to it
C. A 25-year-old restrained driver who is responsive and has a history of diabetes
D. A 42-year-old male who was ejected from the vehicle and is lying supine in the roadway complaining of
back pain

6. Which of the following statements about removing a helmet in a prehospital setting is correct?
A. Helmets should be removed only if they are too tight or if spinal immobilization is required.
B. It is acceptable to leave a helmet on a patient if the patient has no airway or breathing problems.

C H A P T E R 21
C. Helmets should never be removed, so OEC Technicians must be creative in working around the obstacle of
a helmet while providing care.
D. Any patient wearing a helmet should have it removed so that the airway and breathing can be properly
assessed.

7. Based on the anatomy of the spine, which one of the following injuries is possible?
A. A compression fracture of lumbar vertebra 12
B. A dislocation of lumbar vertebra 7
C. A disk injury between thoracic vertebrae 11 and 12
D. A fracture of the ninth cervical vertebra

8. A 22-year-old female rock climber has fallen 25 feet to the trail below. The primary assessment shows her to be
confused and to have an open airway and shallow breathing. Her pulse is 72 beats per minute, and her blood
pressure is 78/50 mmHg. She has no motor ability or sensation in her legs. Which of the following conditions is
the most likely cause of this patient's presentation?
A. Hemorrhagic shock
B. Internal bleeding
C. Hypoglycemia
D. Spinal cord injury

9. A patient with a high probability of a cervical spine injury has just been fully immobilized to a long backboard.
Which of the following instructions should you provide to your team next?
A. "Loosen the collar so you can palpate the back of the neck."
B. "Maintain manual in-line spinal stabilization until he is on the toboggan."
C. "Check for CMS before we move him to the toboggan."
D. "Remove the chest straps so he can breathe more easily now that his head and legs are secured."

10. Which of the following findings is most consistent with a skull fracture?
A. Bruising behind the ear that develops several hours after the injury
B. A hematoma in the occipital area of the head
C. Bilateral dilated pupils that do not respond to light
D. Altered mental status

11. You are assessing a patient who tells you she had a previous head injury in which a portion of the cerebellum
was destroyed. Which of the following signs and symptoms would you expect to find in relation to this injury?
A. Inability to move or feel sensations in one side of the body
B. Inability to remember information such as a past medical history
C. Problems with the regulation of heart rate and blood pressure
D. Poor coordination when walking

12. A patient was killed immediately following a self-inflicted gunshot wound to the head. Which portion of the
central nervous system was most likely damaged in order to cause the rapid death of the patient?
A. The cerebrum
B. The brain stem
C. The cerebellum
D. The meninges

13. Which of the following findings is most consistent with a skull fracture?

C H A P T E R 21
A. A hematoma in the occipital area of the head
B. Bilateral dilated pupils that do not respond to light
C. Altered mental status
D. Bruising behind the ear that develops several hours after the injury

14. Which of the following findings indicates that a patient who received a blow to the head is suffering from
something other than a concussion/TBI?
A. He asks the same questions over and over.
B. His heart rate is 92 and his blood pressure is 144/86.
C. He cannot remember what happened.
D. His pupils are noticeably unequal.

15. You are notified that a patient is coming to the first-aid room with bruising to the lumbar area of the back.
Based on this statement, you would expect to find bruising in which area?
A. The lower back
B. The upper back
C. The buttocks
D. The neck

16. Which of the following signs or symptoms might be evident for an injury to the autonomic nervous system?
A. Inability to move facial muscles
B. Trouble remembering one's own name
C. Vomiting
D. Elevated heart rate

17. You are assessing 30-year-old man who has fallen about 20 feet. He is alert and oriented but states that he
cannot move or feel his legs. Additionally, because his blood pressure is 82/48 mmHg, you suspect neurogenic
shock. Which of the following additional assessment findings reinforces your suspicion of neurogenic shock?
A. Warm and dry skin
B. Shallow rapid respirations
C. Heart rate of 144
D. Heart rate of 44

18. A patient with a head and midline posterior neck injury was sitting found leaning against a tree. You have
placed a vest type short immobilization device on the patient. To transport the patient you would:
A. Secure the patient with the vest-type short immobilization device on a stretcher in a supine position with
the feet elevated.
B. Place the patient in a Semi-Fowler's position on the stretcher for transport to the hospital.
C. Immobilize the patient with the vest-type short immobilization device to a long backboard.
D. Remove the vest-type short immobilization device and secure the patient to a long backboard.

19. You and your partner are preparing to backboard a patient using a vest type short spine immobilization
device. After ensuring that inline manual spinal stabilization is being maintained, your next instruction should
be:
A. "Let's place the vest on the backboard and carefully move him onto it."
B. "Let's get the cervical collar applied before we place the vest."
C. "Let's turn the patient away from the tree before applying the vest."
D. "Let's check motor, sensory, and pulses in the arms and legs."

C H A P T E R 21
20. You are called to aid a 61-year-old man who has fallen off a second-floor lodge patio. He is unresponsive and
has very slow, shallow breathing at a rate of 6 breaths per minute. When you arrive, a fellow patroller has already
opened the airway and has stabilized the head. Which of the following actions should you take immediately?
A. Place the patient in the shock position.
B. Apply a cervical collar.
C. Assist ventilation with a BVM.
D. Transfer the patient to a stretcher for emergency transport.

21. When explaining the priorities of helmet removal to a candidate patroller, you would stress:
A. removing the helmet to ensure the patient's proper alignment on a backboard.
B. sliding the fingers under the occiput so that the head doesn't drop backward.
C. applying a cervical collar before removing the helmet to help stabilize the head.
D. sliding the helmet rearward off the head so that you don't hit the patient's nose.

22. Which of the following assessment findings is most indicative of increasing pressure within the skull from a
closed head injury?
A. Heart rate of 132 beats per minute
B. Pupils equal and reactive
C. Blood pressure of 110/50
D. Blood pressure of 192/82 mmHg

23. A primary injury to the central nervous system would be caused by:
A. A brachial plexus injury
B. A laceration of the spinal cord
C. A hematoma
D. Drowning

24. A young female was involved in a motor-vehicle collision in which her car struck the rear end of another
vehicle. As a result of the impact, her neck muscles were overstretched and torn. She is complaining of neck pain.
Based on this mechanism, the most likely type of injury she has is:
A. Neural ischemia
B. Translational strain
C. Whiplash
D. A Jefferson fracture

25. Which of the following statements indicates that OEC Technicians are correctly using a vest type short spine
immobilization device?
A. The head is secured to the device after the chest has been secured.
B. The cervical collar is applied after the chest has been secured to the device.
C. The head is secured to the device immediately after the cervical collar is applied.
D. The head is secured first, followed by the torso and legs.

26. A 41-year-old male snowboarder struck his head on a metal rail. As you approach him, you note that he
appears confused and has blood on the left side of his face and head, and on his shirt. His breathing appears to be
labored. Which of the following actions should you perform immediately?
A. Apply oxygen at 15 LPM via nonrebreather mask.
B. Check his pupils for equality and reactivity.

C H A P T E R 21
C. Expose the chest to look for possible bleeding.
D. Apply manual in-line spinal immobilization.

27. A patient involved in a motor vehicle collision has a deformity to the left side of the head. The skin overlying
the deformity is still intact. Based on these assessment findings, which of the following conditions would be your
greatest concern?
A. The potential for infection of the brain
B. A spinal cord injury
C. Soft tissue trauma to the scalp
D. A possible brain injury

28. Which of the following findings indicates that a patient who received a blow to the head is suffering from
something other than a concussion/TBI?
A. He asks the same questions over and over.
B. His pupils are noticeably unequal.
C. His heart rate is 92 and his blood pressure is 144/86.
D. He cannot remember what happened.

29. A 2-year-old boy fell down a flight of stairs. Which of the following instructions would you provide to other
patrollers who are immobilizing the patient?
A. "It is better if the collar is a little too big because it will be more comfortable for him."
B. "Let's place a folded towel under his shoulders to help maintain his head alignment."
C. "Just apply a cervical collar and then place him directly on the stretcher."
D. "After he is on the board, place a towel behind his head to keep his airway open."

30. While performing a primary assessment on a patient who has an isolated spinal cord injury, you note that he
is in severe respiratory distress and struggling to breathe. Where should you suspect that the spinal cord injury
has occurred?
A. The diaphragm
B. The lumbar spine
C. The cervical spine
D. The thoracic spine

31. When explaining the priorities of helmet removal to a candidate patroller, you would stress:
A. sliding the helmet rearward off the head so that you don't hit the patient's nose.
B. removing the helmet to ensure the patient's proper alignment on a backboard.
C. applying a cervical collar before removing the helmet to help stabilize the head.
D. sliding the fingers under the occiput so that the head doesn't drop backward.

32. A mechanical injury to the brain that results in a short-term and/or a long-term neurologic deficit is:
A. A subdural hematoma
B. Neural ischemia
C. A traumatic brain injury
D. A repetitive head injury

33. Which of the following instructions from one OEC Technician to another describes the appropriate
application of a cervical spine immobilization collar?

C H A P T E R 21
A. "Keep his head in neutral position while I apply a cervical collar."
B. "I need you to extend the patient's chin backward a little so I can fit the collar under his chin."
C. "Carefully flex his head forward a little so I can pass the collar underneath his neck."
D. "Let's log roll the patient to one side so I can apply a cervical collar."

34. You are called to the lodge by a family who is concerned that their mother is not acting right. The family tells
you that they are concerned because their mother has been complaining of a headache and is very confused today.
Assessment reveals a bruise on the right side of the head that the family states she got about a week ago when
she fell. Which of the following conditions would you be concerned about based on the history and assessment
findings?
A. A scalp laceration
B. A subdural hematoma
C. A concussion
D. A cerebral contusion

35. You are participating in training OEC candidates and are asked about cerebral contusions. You would explain
to the class that a cerebral contusion is:
A. An open skull fracture with increased pressure within the skull.
B. The formation of a pocket of blood within the brain tissue.
C. Bruising and swelling of the brain tissue.
D. Active bleeding between the brain and the skull.

36. Which of the following statements about the care and treatment of a patient with a spinal injury in a
prehospital setting is true?
A. If a patient with a possible spinal injury is in shock, it is permissible to forego immobilization because
doing this uses up time during the golden hour.
B. Before transporting a critically injured patient with a spinal injury, OEC Technicians must perform a
detailed head-to-toe neurological assessment.
C. Prehospital care for a patient with a spinal injury involves correcting life-threatening injuries and limiting
secondary injury.
D. It is important to identify the site of spinal injury so that proper prehospital care can be rendered.

37. A young intoxicated male patient cannot move his lower extremities after diving into the shallow end of a
pool and hitting the bottom with his head. Which of the following mechanisms is most likely to be responsible
for this injury?
A. Compression
B. Rotation
C. Penetration
D. Distraction

38. You should recognize a possible spinal column injury with no spinal cord involvement when you discover
which of the following assessment findings?
A. Intact sensation to all four extremities with the loss of motor ability in the arms
B. Intact motor ability to all four extremities with the loss of sensation to the legs
C. Tenderness to the thoracic spine with intact motor and sensory function in each extremity
D. Pain in the cervical spine with the loss of sensation to the right arm and leg

C H A P T E R 21
39. A 68-year-old female patient is complaining of a headache and generalized weakness. Her husband informs
you that she was with a friend yesterday and struck her face on the dashboard during a motor vehicle collision.
Her past medical history includes a stroke that caused right arm weakness and high blood pressure. When
assessing this patient, which of the following findings should concern you the most?
A. A complaint of pain when she moves her jaw
B. Bruising behind her left ear
C. Weakness in the grip of the right hand
D. A contusion to her left cheek

40. Your patient is a young female who was thrown from her mountain bike and is now confused. Assessment
findings include an open airway, adequate breathing, and a strong radial pulse. Which of the following questions
would be the most important to ask her friends who were with her?
A. "Is she allergic to anything?"
B. "Are all of her immunizations up to date?"
C. "Has she ever needed to be hospitalized before?"
D. "Did she lose responsiveness?"

41. Which of the following signs or symptoms best indicates that a patient has suffered an injury to the thoracic
spine?
A. Cool and diaphoretic skin
B. Tingling in the legs
C. Altered mental status
D. Paralysis of the arms

42. A male soccer player was struck in the head with a soccer ball. Players state that he was dazed for several
seconds following the impact and then asked the same questions over and over. He is currently conscious and
oriented to person but is confused as to place and time. He also has a reddened area on the side of his head and
face. As you proceed with your assessment, his memory continues to improve. Based on these findings, you
would suspect which of the following injuries?
A. An open head injury
B. A concussion/TBI
C. A cerebral contusion
D. An epidural hematoma

43. An elderly patient has fallen down a flight of stairs and is complaining of neck and back pain and weakness to
both legs. Your primary assessment reveals no life threats to the airway, breathing, or circulation. Manual in-line
spinal stabilization is being maintained. Which of the following actions should you take next?
A. Place a cervical collar on the patient and immobilize him to a long spine board.
B. Complete a secondary assessment looking for injuries.
C. Apply high-flow oxygen and move the patient to a stretcher for transport.
D. Place an oral airway and begin assisting ventilation.

44. You respond to an accident at which a male patient is complaining of severe back pain. He informs you that
his back pain is coming from a recent fracture of his coccyx. Based on this information, what area of the back
would the pain be emanating from?
A. The upper back
B. The tailbone
C. The lower back

C H A P T E R 21
D. The neck

45. A patient is complaining of back pain and numbness in both legs after being thrown from a bicycle. When
should you check the motor function, sensory function, and pulses in this patient?
A. During the secondary assessment
B. Immediately after applying a cervical collar
C. Immediately after achieving manual in-line spinal immobilization
D. During the primary assessment

46. You are maintaining manual in-line stabilization of the cervical spine for a patient being log rolled,
transferred, and secured to a long backboard. At what point should you release the manual in-line stabilization?
A. After the body has been fully strapped and the head immobilized with blocks and straps
B. Once the patient has been log rolled onto the long backboard
C. Once the patient has been placed in the waiting ambulance
D. After the patient's head, chest, and knees have been secured with straps

47. You are securing a patient to a long backboard. Which of the following lists indicates the appropriate order for
securing the straps?
A. Head, torso, pelvis, legs
B. Head, legs, torso, pelvis
C. Legs, head, torso, pelvis
D. Torso, pelvis, legs, head

48. A patient involved in a motor-vehicle collision has suffered the separation of a rib from a spinal vertebra.
Based on the anatomy of the spine, where has this injury occurred?
A. The middle spine
B. The cervical spine
C. The thoracic spine
D. The lumbar spine

49. Which of the following questions or statements indicates that an OEC Technician is correctly assessing motor
function in the arms of a patient with potential spine injuries?
A. "Can you hold up two fingers on each hand?"
B. "I am going to feel for a pulse in your wrist."
C. "I am going to move your arm; tell me if it hurts."
D. "Can you tell me what finger I am touching?"

50.A patient has just been log rolled and positioned onto the long backboard. Which of the following actions
should be performed next?
A. Secure the patient's chest with straps
B. Secure the patient's head
C. Apply a properly sized cervical collar
D. Release manual in-line stabilization

C H A P T E R 21
Answer Key: Chapter 21 - Test
1=D, 2=A, 3=C, 4=A, 5=A, 6=B, 7=C, 8=D, 9=C, 10=A, 11=D, 12=B, 13=D, 14=D, 15=A, 16=D, 17=D,
18=C, 19=D, 20=C, 21=B, 22=D, 23=B, 24=C, 25=A, 26=D, 27=D, 28=B, 29=B, 30=C, 31=D, 32=C,
33=A, 34=B, 35=C, 36=C, 37=A, 38=C, 39=B, 40=D, 41=B, 42=B, 43=B, 44=B, 45=D, 46=A, 47=D,
48=C, 49=A, 50=A


C H A P T E R 21
CHAPTER 22
“Face, Eye and Throat Emergencies"

QUIZ

1. A trauma patient tells you that it hurts to talk. This could be indicative of what?
A. A viral or bacterial throat infection
B. Impending airway compromise
C. Nothing potentially serious
D. There is not enough information to make this determination

2. Which of the following is true about aqueous and vitreous humor?


A. Aqueous humor is located in the posterior eye chamber, while vitreous humor is found in the anterior.
B. Once lost through penetrating eye injury, aqueous humor cannot be replaced.
C. Once lost through penetrating eye injury, vitreous humor can be replaced.
D. The combined volumes of vitreous and aqueous humor give the eyes their globe-like shape.

3. A patient with facial trauma is complaining of double vision. This is most likely caused by:
A. Damage to the sclera.
B. Leaking of aqueous or vitreous humor from the globe.
C. A detached retina.
D. Muscles entrapped within an orbital fracture.

4. A patient appears to have suffered fractures of the mandilla and maxilla. Appropriate care would include all of
the following EXCEPT:
A. Assess and maintain the patient's airway.
B. Secure the patient to a long spine board, including securing the head per usual protocol.
C. Administer high-flow oxygen.
D. Firmly palpate the facial bones to determine integrity of the ocular orbit.

5. When assessing a patient's neck, the OECT should NOT:


A. Assess for breath sounds.
B. Assess for the patient's ability to speak or swallow.
C. Palpate the larynx and trachea.
D. Assess for clear, gray, or green matter covering the tonsils.

Answer Key: Chapter 22 - Quiz


1=B, 2=D, 3=D, 4=D, 5=D

CHAPTER 22
TEST

1. You are caring for a 15-year-old male with a laceration of the left ear. Which of the following actions would you
not take in dressing this injury?
A. Wrapping gauze loosely around the head and under the opposite axilla
B. Securing the dressing with gauze that is wrapped securely around the head
C. Using direct pressure to control the bleeding
D. Placing a bulky dressing between the ear and the scalp

2. You have responded to a scene at which a young man tells you he splashed a chemical into his eye. He is
complaining of burning and pain in his right eye. After performing a primary assessment and finding no life-
threatening conditions, you would:
A. Instruct the patient to keep his eyes closed and not rub them while you arrange for emergency transport.
B. Identify the exact chemical involved so that you can get the antidote.
C. Obtain a set of vital signs and SAMPLE.
D. Hold the patient's injured eye open and flush it with large amounts of sterile water from the nose to the
outer edge of the eye.

3. A 10-year-old boy arrives in the aid room bleeding from his nose. He states that he did not injure himself and
just started bleeding while he was skiing, and that he can't get it to stop. Based on your OEC training, which of
the following statements about treating epistaxis is false?
A. The nares, or nostrils should be pressed together just below the bony prominence of the nose.
B. Your initial treatment is to pack the nose with tightly rolled gauze and then apply direct pressure to the
nose.
C. Have the patient lean forward to allow blood and secretions to drain out of the nose and mouth.
D. You should maintain pressure up to 15 minutes before evaluating ongoing bleeding.

4. Which of the following statements concerning the eye socket or orbit is false?
A. It protects the fluid-filled eyeball.
B. It anchors the muscles that control voluntary eye movement.
C. It is formed by the skull and facial bones.
D. It is easy to injure the orbital bones.

5. The part of the eye that focuses light on the retina is called the:
A. Iris
B. Lens
C. Vitreous humor
D. Sclera

6. Which of the following structures works like a camera shutter to adjust the amount of light entering the eye?
A. The iris
B. The posterior chamber
C. The lens
D. The sclera

CHAPTER 22
7. A young girl is complaining of eye pain after having bleach thrown in her face. When assessing her eyes, you
note redness and significant tearing in the right eye. You would treat this girl by:
A. Covering both eyes immediately.
B. Flushing the eye with sterile water.
C. Gently wiping the inner eye with pieces of sterile gauze.
D. Stopping the tearing by applying gentle pressure to the eye.

8. You are assessing a patient who was punched in the eye. You note blood in the white part of his eye. This area
of the eye is known as the:
A. Sclera
B. Retina
C. Lens
D. Cornea

9. A 15-year-old girl struck in the mouth with a baseball bat has lost her front teeth and is splitting a significant
amount of blood. Your primary concern in treating this patient is:
A. Observing her for possible airway compromise.
B. Checking her for a jaw fracture.
C. Saving her teeth so they can be reimplanted.
D. Watching her for seizure activity.

10. You and your partner are caring for a patient who tells you that she wears contact lenses. Your partner wants
to remove the lenses. You remind him that the removal of contact lenses is indicated in all of the following
situations except:
A. eye edema.
B. eye trauma.
C. significant altered level of responsiveness.
D. any facial trauma.

11. When treating a deep laceration to the neck, bandaging should be applied over the pressure dressing by:
A. wrapping roller gauze loosely around the neck and then firmly through the opposite axilla.
B. taping the dressing firmly to the neck.
C. wrapping the bandages around the neck securely enough to maintain pressure on the dressing.
D. wrapping roller gauze tightly around the neck and then firmly under the axilla on the same side.

12. You are on a snowmobiling trip with friends when you see members of the lead group hollering and waving
their hands up ahead. When you get to the scene you note that the lead rider apparently did not see a chain
across the road, and he struck the chain with his neck so forcefully that he was thrown backward off his machine.
Someone in the group is maintaining manual in-line stabilization. When you assess the injured person, which of
the following signs would indicate the injury that must be addressed and managed first?
A. An open fracture of the left arm
B. Difficulty speaking
C. A partially avulsed ear
D. A laceration to the forehead

13. During an altercation, your patient was cut with a sharp knife. Your assessment reveals a laceration across the
right eyelid down to the right cheek that is oozing dark red blood. It also appears that the patient's eye was cut
with the knife. You would:

CHAPTER 22
A. Apply cold packs to the eyelid to decrease swelling.
B. Cover both eyes with a sterile dressing.
C. Apply direct pressure to the eyelid and the eye to control the bleeding.
D. Clean the incisions with sterile water.

14. While running outside the lodge, a teenage girl tripped and fell. She hit her face and mouth, knocking one of
her top front teeth from its socket. Although she is upset, you have assessed no threats to life, and the bleeding
has been controlled. Your partner finds the tooth and asks you what to do with it. Which of the following
responses would be best?
A. "Avoid touching the root of the tooth. We will gently irrigate it and place it back in its socket."
B. "Just put it in dry gauze and bring it with you. They probably won't be able to do anything with it."
C. "Wrap the tooth in dry gauze and put it in a plastic bag placed in cool water."
D. "Scrub the base of the tooth to be sure it's clean and then we'll place it back in its socket."

15. The lower jaw, the only "movable" bone of the face, is hinged on both sides of the skull by the:
A. Maxillary/occipital joints
B. Maxillary sutures
C. Temporomandibular joints
D. Occipital/temporal joints

16. A mother brings her 7-year-old child to the aid room and tells you that fluid is draining from the child's right
ear. She tells you that the child has had a fever since this morning and was complaining of right ear pain. Based
on your OEC training, you would:
A. Put a pressure dressing over the ear to stop the drainage.
B. Arrange for ALS transportation immediately.
C. Have the boy lie on his right side to allow the fluid to drain.
D. Use a 4×4 to pack the ear and then bandage it securely using gauze around the head.

17. You are assisting another OEC Technician to treat a young male with a deep laceration on his neck. Your
partner appears to have controlled the bleeding. He asks you to apply an occlusive dressing. In addition to helping
to control the bleeding, you recognize that another important benefit to this type of dressing on a neck wound is:
A. Preventing a cervical collar from irritating the wound.
B. Providing neck stabilization.
C. Preventing the entry of air into the circulatory system.
D. Promoting clot formation to prevent further bleeding.

18. You respond to the vehicle maintenance department, where a middle-aged man is flushing his eyes with tap
water. He tells you that acid splashed in his eyes and that they are burning. Which of the following questions is
the most important one to ask right away?
A. "Have you ever had a previous eye injury?"
B. "What is your past medical history?"
C. "Do you take any medications?"
D. "Are you wearing contact lenses?"

19. A young boy was running through the lodge with a pencil and tripped. The pencil impaled the boy's left eye
and remains lodged in place. Appropriate care for the eye would include:
A. Applying gentle pressure to the pencil and the eye to keep the pencil from moving.
B. Quickly removing the pencil from the eye to prevent the loss of vision.

CHAPTER 22
C. Applying ice to the affected eye to decrease pressure and reduce subsequent damage.
D. Stabilizing the impaled object and providing immediate transport.

20. Light stimulates nerve endings in which of the following structures to send signals to the brain via the optic
nerves?
A. Lachrymal glands
B. Retina
C. Iris
D. Lens

21. The sternocleidomastoid muscle is responsible for:


A. Protecting the larynx from injury.
B. Securing the clavicle to the shoulder.
C. Rotation of the head.
D. Anatomically dividing the neck into right and left sides.

22. A patient who was hit in the face with a cloud of dust while working in an industrial setting is complaining of
pain and discomfort to his left eye. While performing your assessment on the eye, you note some redness of the
globe but do not see any obvious foreign object. The most appropriate care you can provide to this patient would
be to:
A. Flush eye with copious amounts of water.
B. Carefully wipe the eye with a soft piece of sterile gauze.
C. Have the patient keep his eye open during transport to a medical facility.
D. Gently massage the eye to promote tearing.

23. You are on a snowmobiling trip with friends when you see members of the lead group hollering and waving
their hands up ahead. When you get to the scene you note that the lead rider apparently did not see a chain
across the road, and he struck the chain with his neck so forcefully that he was thrown backward off his machine.
Someone in the group is maintaining manual in-line stabilization. When you assess the injured person, which of
the following signs would indicate the injury that must be addressed and managed first?
A. A partially avulsed ear
B. An open fracture of the left arm
C. A laceration to the forehead
D. Difficulty speaking

24. What instructions would you give to an OEC candidate who asks how to treat a patient who has visible blood
in the anterior chamber of the eye?
A. "It is important to keep the pressure in the eye low, so we will transport this patient in a sitting position."
B. "Apply a cold pack to the eye and transport this patient immediately."
C. "Put a shield over the eye and bandage tightly to apply some pressure to the eye."
D. "Put a shield over the eye and cover both eyes before immediately transporting this patient."

25. The condition characterized by pupils of unequal size is:


A. retinal detachment.
B. anisocoria.
C. hyphema.
D. malocclusion.

CHAPTER 22
26. You are caring for a 15-year-old male with a laceration of the left ear. Which of the following actions would
you not take in dressing this injury?
A. Placing a bulky dressing between the ear and the scalp
B. Using direct pressure to control the bleeding
C. Securing the dressing with gauze that is wrapped securely around the head
D. Wrapping gauze loosely around the head and under the opposite axilla

27. A mother brings her 7-year-old child to the aid room and tells you that fluid is draining from the child's right
ear. She tells you that the child has had a fever since this morning and was complaining of right ear pain. Based
on your OEC training, you would:
A. put a pressure dressing over the ear to stop the drainage.
B. arrange for ALS transportation immediately.
C. use a 4x4 to pack the ear and then bandage it securely using gauze around the h
D. have the boy lie on his right side to allow the fluid to drain.

28. The largest bone of the face, which forms the upper jaw, the hard palate, the floor of the nose, and the lower
portion of each eye socket, is the:
A. Maxilla
B. Frontal bone
C. Mandible
D. Zygoma

Answer Key: Chapter 22 - Test


1=A, 2=D, 3=B, 4=D, 5=B, 6=A, 7=B, 8=A, 9=A, 10=D, 11=A, 12=B, 13=B, 14=A, 15=C, 16=C, 17=C,
18=D, 19=D, 20=B, 21=C, 22=A, 23=D, 24=D, 25=B, 26=D, 27=D, 28=A


CHAPTER 22
CHAPTER 23
“Thoracic Trauma”

QUIZ

1. During inhalation, the diaphragm:


A. Expands and pulls the intercostal muscles downward.
B. Relaxes and returns to its resting position.
C. Activates the phrenic nerve.
D. Contracts and descends into the abdomen.

2. All of the following are true about the anatomy and physiology of the chest EXCEPT:
A. The mediastinum consists of the heart, aorta, vena cava, trachea, esophagus, thymus gland, and nerves.
B. The thoracic cage encompasses the heart, lungs, and mediastinum.
C. The thorax is enclosed by the bony rib cage, sternum, thoracic spine, and diaphragm.
D. The thorax includes the area from the mandible to the costal margin.

3. All of the following statements about thoracic trauma are true EXCEPT:
A. Patients with chest trauma may initially appear very stable.
B. Ongoing assessment is vital for patients with either major or minor traumatic chest injuries.
C. Skiers outnumber snowboarders almost 2:1 for incidences of traumatic chest injuries.
D. 10-30% of all chest trauma requires surgical intervention.

4. Beck's triad consists of which three significant clinical findings?


A. Absent pulse, diminished lung sounds, and rapidly dropping BP
B. Distended neck veins, muffled heart sounds, and pulsus paradox
C. Decreased pulse, increased respirations, and widening systolic diastolic readings
D. Irregular pulse, respirations <10, and stable blood pressure

5. A skier collides chest first with a tree at a high rate of speed. Based on this mechanism of injury alone, the
OECT should be suspicious of all of the following injuries EXCEPT:
A. Hemo, pneumo, or hemopneumothorax.
B. A triple A.
C. A lacerated liver or ruptured spleen.
D. Pericardial tamponade.

Answer Key: Chapter 23 - Quiz


1=D, 2=D, 3=C, 4=B, 5=B

CHAPTER 23
TEST

1. Which of the following actions occurs when the diaphragm and the intercostal muscles relax?
A. Release
B. Exhalation
C. Inhalation
D. Inspiration

2. You have placed an occlusive dressing on a puncture wound on the right side of the chest of a 33-year-old
woman. During your ongoing assessment your primary concern is monitoring the injury for:
A. Ecchymosis at the wound site
B. Infection at the wound site
C. Arterial bleeding
D. Tension pneumothorax

3. You are instructing a class in Outdoor Emergency Care when a student asks you to explain the L.A.P. method of
examining the thorax. Your best reply to this question would be which of the following statements?
A. "The L.A.P. method is a rapid assessment for deformities of the chest."
B. "The L.A.P. method can be used under any conditions and in any environment."
C. "The L.A.P. method divides the chest into three exam areas: the lateral, anterior, and posterior thorax."
D. "The L.A.P. method directs you to look, auscultate, and palpate the chest."

4. Your patient was involved in a serious motor vehicle collision. Which of the following assessment findings best
helps to determine that the patient has a flail chest segment?
A. Pain upon inspiration
B. Chest wall bruising
C. Shortness of breath
D. Paradoxical chest wall movement

5. The heart and lungs are located in the:


A. Pleural cavity
B. Mediastinum
C. Pericardial cavity
D. Thoracic cavity

6. You are instructing a class in Outdoor Emergency Care when a student asks you to explain the L.A.P. method of
examining the thorax. Your best reply to this question would be which of the following statements?
A. "The L.A.P. method can be used under any conditions and in any environment."
B. "The L.A.P. method directs you to look, auscultate, and palpate the chest."
C. "The L.A.P. method is a rapid assessment for deformities of the chest."
D. "The L.A.P. method divides the chest into three exam areas: the lateral, anterior, and posterior thorax."

7. Your 35-year-old patient has received major chest trauma. He complains of shortness of breath and pain. Your
assessment reveals distended neck veins and pulsus paradoxus. Based on these signs and symptoms, you believe
the patient to be suffering from:
A. Acute myocardial infarction

CHAPTER 23
B. Pericardial tamponade
C. Pleural effusions
D. Aortic rupture

8. You are following the L.A.P. method to examine the chest of a 38-year-old trauma patient. To assess for thoracic
cage instability, you would apply moderate downward pressure on the sternum while asking the patient to take a
deep breath, and then you would:
A. "Walk" the patient's clavicles with your fingertips while the patient exhales.
B. Have the patient exhale slowly while you listen to the breath sounds.
C. Palpate the upper thorax for subcutaneous emphysema.
D. Apply moderate inward pressure on the lateral walls of the rib cage and ask the patient to take a deep
breath.

9. The diaphragm separates the:


A. Thoracic cavity from the abdominal cavity
B. Heart and lungs from the spine
C. Lungs from the heart
D. Right lung from the left lung

10. A patient has sustained an injury to his mediastinum. Based on the anatomy of his chest, which of the
following structures may have been injured?
A. The diaphragm
B. The lungs
C. The esophagus
D. A bronchus

11. A 39-year-old man has been stabbed once in the anterior chest. When notifying the patrol aid room by radio,
you inform them that the patient has suffered what possible type of injury?
A. Evisceration
B. Flail chest
C. An open chest injury
D. lacerated spleen

12. You are assisting at an accident scene and are asked to apply a dressing over a sucking chest wound on the
patient's left anterior chest wall. Which of the following items would you use?
A. A dry sterile gauze dressing
B. A clean cravat
C. Vaseline gauze
D. Sterile gauze soaked in sterile water

13. You are following the L.A.P. method to examine the chest of a 38-year-old trauma patient. To assess for
thoracic cage instability, you would apply moderate downward pressure on the sternum while asking the patient
to take a deep breath, and then you would:
A. "walk" the patient's clavicles with your fingertips while the patient exhales.
B. palpate the upper thorax for subcutaneous emphysema.
C. have the patient exhale slowly while you listen to the breath sounds.
D. apply moderate inward pressure on the lateral walls of the rib cage and ask the patient to take a deep
breath.

CHAPTER 23
14. You are listening to two patrollers discuss the accident to which they just responded. Which of the following
injuries would lead you to believe that one of the patients had suffered from traumatic asphyxia?
A. A bilateral femur deformity
B. Abdominal bruising and distention
C. A chest wound that makes a sucking sound
D. Bluish discoloration of the neck and face

15. When assessing a patient, which of the following signs or symptoms is most indicative that the patient is
suffering from a tension pneumothorax?
A. Decreased breath sounds in the right lung
B. Excessive sweating
C. Crepitus in the abdomen
D. Severe heartburn

16. You have applied a nonporous dressing to a puncture wound on a patient's chest. An OEC candidate asks you
why the dressing was taped only on three sides. Your best response is that taping the dressing on three sides:
A. Decreases the chances that air will enter the chest upon exhalation.
B. Allows trapped air to escape upon exhalation.
C. Permits oxygen to enter the lungs.
D. Is less painful for the patient.

17. You are treating a 26-year-old man with a pneumothorax. The patient's breathing has been inadequate, and
you have been assisting his ventilations with a bag valve mask. The patient's condition seems to be worsening.
Because you understand the pathology involved in a pneumothorax, you are most concerned that:
A. You may have missed an injury.
B. You are not maintaining an adequate seal around the facemask, and thus the patient is not getting enough
oxygen.
C. The patient's airway is not being maintained, so you should insert an oropharyngeal airway.
D. The pressure from the BVM is causing more air to enter the pleural space and is causing a tension
pneumothorax.

18. You treated a 19 year old with a puncture wound to the chest by covering the wound with an impermeable
dressing. Now, as you perform your ongoing assessment, you note that the patient is tachypneic and is
complaining of difficulty breathing. Breath sounds on the side of the injury are also diminished. Which of the
following actions should you take immediately?
A. Assist his breathing with a bag-valve mask.
B. Add another layer to the dressing.
C. Insert an oropharyngeal airway
D. Lift a corner of the dressing from the wound for a few seconds; then reapply and check his breath sounds.

19. The thorax is enclosed by the:


A. Ribs, clavicles, diaphragm, and mediastinum.
B. Thoracic vertebrae, diaphragm, mediastinum, and clavicles.
C. Ribs, sternum, thoracic spine, and diaphragm.
D. Clavicles, sternum, diaphragm, and thoracic spine.

CHAPTER 23
20. You are treating a 26-year-old man who fell about 20 feet while rock climbing. He is complaining of right
sided chest discomfort and dyspnea. You note that his respiratory rate is 28 and shallow. Based on this
assessment, the best emergency care would be:
A. placing the patient in a position of comfort to help breathing, performing an L.A.P. exam of the thorax, and
administering high-flow oxygen.
B. performing a rapid body survey and assisting ventilations with BVM and high-flow oxygen.
C. achieving spinal immobilization, administering high-flow oxygen, and then performing "load and go.”
D. achieving spinal immobilization, splinting the affected area of the chest, and administering high-flow
oxygen.

21. Which of the following phrases best describes a flail chest segment?
A. Multiple rib fractures with bruising of the underlying lung
B. Fractured ribs that have collapsed the underlying lung
C. Multiple fractures to the rib cage caused by blunt trauma
D. Two or more adjacent ribs that have been broken in two or more places.

22. An OEC candidate asks you to explain a pneumothorax. Your response should be that it occurs when:
A. Air enters the lung alveoli through a traumatic opening in the chest wall.
B. Air accumulates between the inner chest wall and the outside of the lung, causing the lung to collapse.
C. The lung becomes overinflated with air and then collapses.
D. The trachea becomes obstructed, causing the lungs to collapse.

23. You are treating a 26-year-old man with a pneumothorax. The patient's breathing has been inadequate, and
you have been assisting his ventilations with a bag valve mask. The patient's condition seems to be worsening.
Because you understand the physiology involved in a pneumothorax, you are most concerned that:
A. the patient's airway is not being maintained, so you should insert an oropharyngeal airway.
B. you are not maintaining an adequate seal around the facemask, and thus the patient is not getting enough
oxygen.
C. you may have missed an injury.
D. the pressure from the BVM is causing more air to enter the pleural space and is causing a tension
pneumothorax.

24. Injuries at which level of the spinal cord can affect the phrenic nerves and therefore respiration?
A. T4-T6
B. T1-T3
C. C3-C5
D. C6-T1

25. The diaphragm plays a major role in respiration. During inhalation, the diaphragm:
A. Contracts and creates a negative pressure in the thorax, which draws air into the lungs.
B. Elongates and descends into the abdomen, allowing air to enter the lungs.
C. Contracts and increases internal thoracic pressure, which pushes air out of the lungs.
D. Relaxes so that it can be stretched as you pull air into the lungs.

26. You are managing a patient with a large flail segment in the right lateral chest. The immediate threat to this
patient's life is:
A. Rib fractures
B. Hypoxia

CHAPTER 23
C. Infection
D. Blood loss

27. An OEC candidate indicates that she understands the difference between a pneumothorax and a tension
pneumothorax when she makes which of the following statements?
A. "A tension pneumothorax can cause cardiac output to decrease; a pneumothorax does not.”
B. "A tension pneumothorax causes blood to accumulate around the lung; a pneumothorax involves the
accumulation of air only around the lung.”
C. "A pneumothorax is caused by a closed chest injury; a tension pneumothorax is caused by an open chest
injury.”
D. "A pneumothorax describes a collapsed lung; a tension pneumothorax involves both a collapsed lung and
blood loss."

28. You are treating a 16-year-old male who was skiing out of control and ran off the trail and into the woods.
When you arrive, he is alert and complaining of pain in his right lower chest. Upon examination you note that he
has impaled himself with a tree branch. He tells you he will feel better if you just remove the branch. Based on
your training as an OEC technician, you would respond by making which of the following statements?
A. "We need to leave the branch in place until we get you to a hospital."
B. "I'm not supposed to remove the branch, but you can if you think it will make you more comfortable."
C. "I need to get a bulky dressing ready in case there is bleeding when I remove the branch."
D. "Once my helpers arrive and we are ready to transport you, we can remove the branch."

29. You suspect that a trauma patient is suffering from a hemothorax to the left lung. Which of the following
assessment findings would reinforce your suspicion?
A. Neck vein distention and absent breath sounds in the right lung
B. Bradycardia and hypertension
C. Respiratory distress and the signs and symptoms of shock
D. Cyanosis and a blood pressure of 210/100

30. Which of the following statements indicates that the speaker understands chest trauma?
A. "Open chest injuries are more serious than closed chest injuries because air and bacteria can enter the
chest through the open wound.”
B. "In general, an open chest injury caused by blunt trauma is more life threatening than a closed one due to
increased blood loss.”
C. "Internal chest injuries are more difficult to assess and manage than are the more obvious external chest
injuries.”
D. "Closed chest injuries are caused by penetrating trauma and cause more life-threatening injuries than do
open chest injuries."

31. You are treating a 16-year-old male who was skiing out of control and ran off the trail and into the woods.
When you arrive, he is alert and complaining of pain in his right lower chest. Upon examination you note that he
has impaled himself with a tree branch. He tells you he will feel better if you just remove the branch. Based on
your training as an OEC technician, you would respond by making which of the following statements?
A. "Once my helpers arrive and we are ready to transport you, we can remove the branch.”
B. "I need to get a bulky dressing ready in case there is bleeding when I remove the branch.”
C. "I'm not supposed to remove the branch, but you can if you think it will make you more comfortable.”
D. "We need to leave the branch in place until we get you to a hospital.”

CHAPTER 23
32. You are at the scene of a shooting. Your assessment reveals a 23-year-old man who has been shot twice. The
first wound is to the left lower quadrant of the abdomen and is bleeding. The second wound is to the left lateral
chest and makes a sucking sound every time the patient takes a breath. The initial action of an OEC Technician
should be which of the following actions?
A. Place direct pressure over the abdominal wound.
B. Place the patient on high-flow oxygen using a nonrebreather mask.
C. Cover the chest wound with a gloved hand.
D. Cover the chest wound with a gauze dressing.

Answer Key: Chapter 23 - Test


1=B, 2=D, 3=D, 4=D, 5=D, 6=B, 7=B, 8=D, 9=A, 10=B, 11=C, 12=C, 13=D, 14=D, 15=A, 16=B, 17=D,
18=D, 19=C, 20=B, 21=D, 22=B, 23=D, 24=C, 25=A, 26=B, 27=A, 28=A, 29=C, 30=C, 31=D, 32=C

CHAPTER 23
CHAPTER 24
“Abdomen & Pelvic Trauma”

QUIZ

1. The bones that protect the organs of the pelvic cavity include the:
A. Pubic symphisis, trochanter, and sacrum.
B. Lumbar spine, pelvic girdle, and trochanter.
C. Glenoid, sphenoid, and mesenteric.
D. Ilium, ischium, and pubis symphisis.

2. The superior and inferior boundaries of the abdominal cavity are:


A. The anterior and posterior ribs and the iliac crests.
B. The superior aspect of the liver and the urinary bladder.
C. The mediastinum and the symphisis pubis.
D. The diaphragm and the pelvic brim.

3. A patient was blindsided and struck from the side by a snowboarder who left the scene. The patient is rapidly
developing shock signs and symptoms and is complaining of left shoulder pain but you do not find any indication
of shoulder injury. This is most likely due to a:
A. Shoulder injury.
B. Ruptured diaphragm.
C. Ruptured spleen.
D. Lacerated liver.


4. You are assessing a normally healthy 29-year-old patient who is complaining of abdominal pain, nausea, and
vomiting. The patient's vital signs are indicating that the patient is growing shock-y. Which piece of SAMPLE
information would cause you the greatest concern?
A. The patient's wife and 2-year-old son both have the 'flu.'
B. The patient took a hard fall while skiing 2 days ago and landed on his chest and abdomen.
C. The patient is taking a beta blocker for high blood pressure.
D. The patient had appendicitis 12 weeks prior.

5. Which of the following statements about internal abdominal injuries is true?


A. Solid organ rupture releases toxins into the abdominal cavity, which can cause peritonitis.
B. Abdominal pain always directly pinpoints the location of an injury.
C. The greatest danger from a ruptured hollow organ is internal hemorrhage.
D. Blunt trauma tends to cause injury to solid organs, while penetrating trauma tends to injure hollow
organs.

6. The most common cause of vascular injury (i.e., aortal or vena cava rupture) within the abdominal cavity is
most often caused by:

CHAPTER 24
A. Low-velocity penetrating trauma (i.e., impalement by tree branch).
B. Blunt trauma.
C. Deceleration trauma.
D. High-velocity penetrating trauma (i.e., bullet).

Answer Key: Chapter 24 - Quiz


1=D, 2=D, 3=C, 4=B, 5=D, 6=C

TEST

1. You have been called to aid a patient with generalized abdominal pain. The patient states that he recently had
his gall bladder removed. You know that the gallbladder is located in the:
A. Lower right abdominal quadrant.
B. Lower left abdominal quadrant.
C. Left upper abdominal quadrant.
D. Right upper abdominal quadrant.

2. In most cases, the position of comfort in abdominopelvic trauma is:


A. in a right recumbent position with the knees bent.
B. in the Trendelenburg position.
C. in a left recumbent position with the knees bent.
D. supine, with the knees bent.

3. Because pelvic fractures can result in large blood losses, you should:
A. Give the patient liquids by mouth to replace lost body fluids.
B. Call for ALS to begin treatment ASAP to replace lost body fluids.
C. Apply O2 at 2 to 3 LPM via a non-rebreathing mask.
D. Give the patient glucose.

4. In most cases, the position of comfort in abdominopelvic trauma is:


A. In the Trendelenburg position.
B. In a left recumbent position with the back bent.
C. Supine, with padding under the knees to keep them slightly bent.
D. In a right recumbent position with the knees straight.

5. The spleen of a patient has ruptured. Because the spleen is a solid organ, you recognize that the primary threat
to life is most likely:
A. Pain
B. Infection
C. Blood loss
D. Inflammation

6. When taking care of a patient with an abdominal injury, your first priority is:
A. Keeping the scene, patient and all rescuers safe.

CHAPTER 24
B. Achieving proper airway management.
C. Stabilizing the patient.
D. Controlling massive bleeding.

7. The most common mechanism of injury to the pancreas is:


A. Direct trauma to the middle of the abdomen
B. Trauma to the left flank
C. Rotational trauma to the chest wall
D. Trauma to the right flank

8. The abdomen is located between the:


A. Diaphragm and the pelvic brim.
B. Base of the 5th rib and the pelvic base.
C. Nipple line and the coccyx of the spine.
D. Mid-chest and the level of the umbilicus.

9. A crying 42-year-old woman states that she has sharp abdominal pain that she localizes to her right lower
quadrant. When assessing this patient's abdomen, you would look for distention and symmetry and then:
A. Avoid palpating the entire abdomen.
B. Start the abdominal exam by palpating the right lower quadrant.
C. Avoid palpating the right lower quadrant.
D. Start the exam by palpating the abdomen in the quadrant farthest away from the one that is painful.

10. If a scene becomes unsafe for a patient with abdominopelvic trauma, OEC Technicians should:
A. Move the patient to a safer place, so long as doing so does not place rescuers at risk.
B. Call for more help.
C. Continue to care for the patient and then "load and go."
D. Leave the scene.

11. Upon entering the treatment room of a 62-year-old woman who has an unknown medical complaint, Which
one of the following clues observed during the scene size-up would increase your suspicion that the patient is
suffering from an acute abdominal condition?
A. A portable commode is next to the bed.
B. She is sweating even though the air conditioning is on "high."
C. She is wearing a nasal cannula attached to oxygen at 2 LPM.
D. She is lying on her side clutching her abdomen with her knees drawn up to her chest.

12. When palpating the anterior portion of a patient's abdomen, you note tenderness in the left upper quadrant.
You would recognize that which organ might be involved?
A. The kidney
B. The spleen
C. The liver
D. The gallbladder

13. To prepare a patient with a pelvic fracture for transportation, you should:
A. Apply a pelvic binder.
B. Place the patient in a supine position on a backboard.
C. Place the patient in a seated position in a toboggan.

CHAPTER 24
D. Place the patient on a scoop stretcher in the right recumbent position.

14. Because pelvic fractures can result in large blood losses, OEC Technicians should:
A. apply O2 at 6 LPM via a nonrebreather.
B. call for ALS to begin treatment to replace lost body fluids.
C. give the patient glucose.
D. give the patient liquids to replace lost body fluids.

15. Which of the following statements indicates that you correctly understand the goal of assessing and managing
a patient who has abdominal pain?

A. "It is more important to recognize a possible abdominal injury or emergency than it is to identify the exact
cause.”
B. "It is important to determine if the liver is the cause of pain, because that is the most life-threatening
condition.”
C. "If the patient has tenderness upon any palpation of the abdomen, OEC Technicians should assume that
bleeding is the cause.”
D. "To best treat the patient, OEC Technicians must identify the exact cause of the patient's pain."

16. To prepare a patient with a pelvic fracture for transportation, OEC Technicians should:
A. place the patient in a seated position in a toboggan.
B. place the patient in a supine position on a backboard.
C. place the patient on a scoop stretcher in the right recumbent position.
D. apply a pelvic binder.

17. Blunt trauma to the upper left abdominal quadrant can injure the:
A. Appendix
B. colon
C. Liver
D. Spleen

18. The function of organs in the abdomen is:


A. The storage of fat
B. The absorption of oxygen
C. Digestion, production of white blood cells, and the filtering of blood
D. Only digestion

19. An injury to the abdominal wall allowing the intestines to come out of the abdominal cavity is a:
A. Evisceration
B. Abrasion
C. Laceration
D. Contusion

20. In treating an evisceration, OEC Technicians should:


A. not put the protruding organs back inside the abdominopelvic cavity.
B. cover the wound with an occlusive flutter-valve dressing.
C. apply a sterile dressing and an ice pack.
D. apply a sterile dressing and direct pressure.

CHAPTER 24
21. A patient complains of severe and sharp pain in the right shoulder when palpating the right upper abdominal
quadrant. Based on this sign and location, what organ or structure is most likely involved?
A. Liver
B. Kidney
C. Spleen
D. Appendix

22. In which of the following injuries can the abdominal cavity migrate into the thoracic cavity?
A. A tear in the diaphragm
B. An intestinal tear
C. A rib fracture
D. An abdominal hernia

23. The pancreas is best described as a (n):


A. Solid organ
B. Fluid-filled organ
C. Hollow organ
D. Nonfunctional organ

24. You would recognize which one of the following situations as an acute abdominal emergency?
A. A woman who says she drank tequila last night and now has stomach pain with diarrhea
B. A patient who complains of stomach pain and has bad-smelling gas
C. A ski racer who hit a tree and has significant left flank pain prior to and during palpation.
D. A child whose sister has hepatitis and who has just vomited for the first time

25. Blunt trauma to the upper right abdominal quadrant can injure the:
A. Spleen
B. Pancreas
C. Small intestines
D. Liver

Answer Key: Chapter 24 - Test


1=D, 2=D, 3=B, 4=C, 5=C, 6=A, 7=A, 8=A, 9=D, 10=A, 11=D, 12=B, 13=A, 14=B, 15=A, 16=D, 17=D,
18=C, 19=A, 20=A, 21=A, 22=A, 23=A, 24=C, 25=D

CHAPTER 24
CHAPTER 25
“Cold Emergencies”

QUIZ

1. All of the following statements regarding thermoregulation are true EXCEPT:


A. Shivering is an ineffective means of heat production and is generally more harmful than beneficial to the
body.
B. The hypothalamus controls body temperature.
C. Shunting and constriction allow the body to minimize core heat loss while increasing the risk of frostbite
or frost nip.
D. Humans are far better developed physiologically for cooling rather than warming themselves.

2. The two chief factors that predispose an individual to cold injury are:
A. Age and alcohol.
B. Melanin and adipose.
C. Body fat and metabolic rate.
D. Health and fitness.

3. All of the following statements about cold water immersion deaths are true EXCEPT:
A. Immersion hypothermia is a rapid process that usually occurs in under 10 minutes.
B. Immersion hypothermia can result in severe after-drop even after rewarming measures are implemented.
C. Without a life jacket, most victims of cold water immersion die due to muscle fatigue and subsequent
drowning rather than hypothermia.
D. Immersion hypothermia is a more rapid process than nonimmersion hypothermia.

4. Frostbite damages tissue by:


A. Vascular rupture.
B. Sludging.
C. Cellular freezing and expansion with subsequent cellular wall rupture.
D. Intercellular ice crystals, dehydration, and cellular chemical imbalance.

5. The pulse rate for a hypothermic patient should initially be checked for:
A. 3-5 minutes.
B. 30 seconds.
C. 10 seconds.
D. 15 seconds.

Answer Key: Chapter 25 - Quiz

CHAPTER 25
1=A, 2=A, 3=A, 4=D, 5=C

TEST

1. Which of the following findings is a sign of severe hypothermia?


A. Slurred speech
B. Coma
C. Confusion
D. Active shivering

2. Your friend, who is a paramedic, is describing an emergency call he was on last weekend. He tells you that he
cared for a 78-year-old woman who had slipped while getting out of her bathtub and had been lying on a tile floor
with no clothing for almost 24 hours. When your friend arrived he found the woman confused and cold to the
touch. Although it was 80 degrees outside, the temperature in the apartment was only 68 because she had her air
conditioner running on high. Based on your OEC training, you recognize that this patient was probably suffering
from hypothermia due to:
A. Evaporation
B. Convection
C. Radiation
D. Conduction
E. All of the above

3. The major cause of death in avalanche victims is:


A. severe hypothermia.
B. bleeding.
C. trauma.
D. asphyxiation.

4. After successfully rewarming a foot that has frozen toes, an OEC Technician should:
A. Ambulate the patient to ensure return of adequate motor function.
B. Place the foot below the level of the heart to enhance circulation.
C. Gently massage the foot and toes.
D. Cover the foot and toes with dry dressings.

5. Which of the following signs would you observe earliest in a hypothermic patient?
A. Bradycardia
B. Confusion
C. Hypotension
D. Shivering

6. You and your friends are camping on a 20-degree day. Which of the following assessment findings best
illustrates that your bodies are no longer capable of compensating for the effects of the cold?
A. Cool and clammy skin
B. Loss of fine motor coordination
C. Shivering
D. Hypertension

CHAPTER 25
7. Which of the following instructions is most appropriate when moving a patient who has severe generalized
hypothermia?
A. "I want everyone to take extra care in moving her very gently to the stretcher; we do not want her to go
into cardiac arrest.”
B. "Let's secure her with her arms and legs extended because this position promotes the retention of body
heat.”
C. "Be careful when moving her to the stretcher; the cold makes her very prone to bone fractures.”
D. "Let's place her on her side on the stretcher because her body temperature contraindicates placement of a
nasal airway."

8. A patient with moderate hypothermia will have a:


A. high risk of ventricular fibrillation.
B. higher-than-normal blood pressure.
C. slow pulse and/or slow respirations.
D. core body temperature of 90–95 degrees F.

9. Which of the following findings best indicates that a patient with cold skin is suffering from moderate
hypothermia?
A. Respirations at 28-30 per minute
B. Cold waxy fingers
C. A pulse oximetry reading of 95 percent
D. A heart rate of 52 beats per minute

10. hen assessing the vital signs of a severely hypothermic patient, OEC Technicians should:
A. Monitor the patient's pulse and respirations for ten seconds.
B. Warm their own fingers for a full minute to ensure that they can feel the patient's pulse.
C. Vigorously shake the patient to assess the true level of responsiveness.
D. Check the patient's radial pulse to avoid exposing the patient's head and neck to the cold.

11. You are facilitating a talk on cold emergencies in the backcountry. You are asked when one should attempt to
rewarm a hand or foot that is frostbitten. Which of the following statements would be your best response?
A. "You never want to rewarm a frozen body part because doing so can cause additional damage."
B. "Rewarming should take place only when there is no chance that the tissue will refreeze."
C. "Attempt to rewarm a frozen body part only if the person still has some feeling in it."
D. "Rewarm the frozen body part if the patient needs to self evacuate"

12. Which of the following statements indicates that an OEC Technician understands the care of a patient with a
localized cold injury to the foot?
A. "You should avoid immobilizing the foot because doing so will further decrease the circulation of blood in
the affected area.”
B. "Gentle massage of the foot is beneficial because it enhances the circulation of warm blood in the affected
area.”
C. "Warming a frozen body part can cause severe pain”
D. "If the patient has blisters on his foot, it is beneficial to break them so that the fluid inside will not freeze
and cause additional injury."

CHAPTER 25
13. A young boy complains of pain in his fingers after spending several hours outside riding a sled in cold
temperatures. After ensuring that he has no life-threatening conditions, you turn your attention to his hands and
note that his fingers are cold and the skin is pliable to the touch. Which of the following signs or symptoms
would help confirm your suspicion that he is suffering from early or superficial frostbite?
A. Firm and numb cold skin on the fingers
B. Swelling of and blisters on the fingers
C. Soft and tingling skin on the fingers
D. A history of being exposed to temperatures in the 70s

14. A patient with moderate hypothermia will have a:


A. High risk of ventricular fibrillation.
B. Active shivering
C. Slow pulse and/or slow respirations.
D. Core body temperature. above 95

15. A patient who is conscious and breathing has been pulled from a stream of cold water. To decrease her loss of
heat via the mechanism of conduction, an OEC Technician should immediately:
A. Encourage her to stop shivering.
B. Provide positive-pressure ventilation.
C. Remove her wet clothing then cover her with blankets.
D. Cover her with a blanket.

16. Which of the following processes best describes the emission of infrared heat?
A. Evaporation
B. Respiration
C. Conduction
D. Radiation

17. When obtaining a history of a five-year-old boy with generalized hypothermia, which of the following
questions provides the most important information and should an OEC Technician ask first?
A. "Why was he outside without adult supervision?"
B. "Are all of his childhood shots up to date?"
C. "When did he last eat or drink?"
D. "Does he have any medical conditions?"

18. You have initiated the rewarming of a local cold injury in a patient's hand and fingers. On scene the patient
had no sensation in his hand or fingers. Which of the following statements is a cause of concern when made by
the patient?
A. "I still can't feel my fingers when I rub them with my other hand."
B. "The blisters on my fingers haven't broken yet."
C. "My fingers are really starting to hurt now!"
D. "My hand is feeling funny. It seems to be starting to tingle a little bit."

19. To prevent a continued drop in core body temperature after treatment for hypothermia has begun, the patient
should:
A. Have the extremities vigorously rubbed to move fluids into the body's core.
B. Immediately start exercising to produce body heat.
C. Be entirely immersed in warm water.

CHAPTER 25
D. Remain quiet and in a supine position.

20. It has taken you 45 minutes to extricate an avalanche victim. Based on your knowledge of hypothermia and
avalanche victims, you know that:
A. A pulseless victim can most likely be resuscitated after rewarming.
B. Hypothermia can occur after extrication due to a cold, windy environment
C. Hypothermia during an avalanche burial generally occurs quicker than during cold water immersion
D. A patient who is responsive is not hypothermic.

21. The initial priority for avalanche victims is:


A. Cervical stabilization
B. Treating for hypothermia
C. Ensuring adequate ventilation
D. Warming the extremities

22. Which of the following statements concerning immersion hypothermia is true?


A. Death will typically occur within a few minutes.
B. It occurs only when the entire body and head are submersed.
C. The function of arm and leg muscles will not affect the outcome.
D. It can take more than 30 minutes for an individual to become hypothermic.

23. Which of the following statements about immersion hypothermia is true?


A. It can take more than 30 minutes for a person who has fallen into very cold water to become hypothermic
B. Death typically occurs within a few minutes of submersion in very cold water.
C. Immersion hypothermia is most typically associated with traumatized or critically ill patients.
D. Protective winter clothing will always protect a person from immersion hypothermia

24. A young man who was angry with friends wandered away from a party and spent the night outside uncovered
in 40-50 degree temperatures. He is confused, has decreased but adequate breathing, and has a weak radial pulse.
His skin is cool, and capillary refill is delayed. When looking to see if he is shivering, you note that he is not.
Based on this presentation, OEC Technicians can safely conclude that the:
A. Cause of his confusion is not related to exposure to the low overnight temperatures.
B. Patient's body temperature is most likely dangerously low.
C. Patient is exhibiting signs indicating that he is ineffectively compensating and conserving heat.
D. Absence of shivering indicates that body temperature is almost back to normal.

25. Prevention of heat loss in the care of a patient with a cold injury should include:
A. Carefully removing wet clothing
B. Rubbing cold extremities
C. Placing the patient in a prone position
D. Placing the OEC technician's jacket over the patient

26. You are with a candidate OEC Technician who is attending to snowboarder who has fallen. The candidate is
very concerned that the snowboarder is shivering vigorously and asks you what this indicates. Which of the
following statements would be your best response?
A. Shivering indicates moderate hypothermia.
B. Shivering is a sign of mild hypothermia.
C. Individuals who are shivering cannot maintain their normal body temperature.

CHAPTER 25
D. It is good to try to get a shivering patient to stop because shivering burns energy.

27. Which of the following findings is not a sign of severe hypothermia?


A. Diminished pupillary eye reflexes
B. Coma
C. A palpable radial pulse
D. A core temperature below 82 degrees F

28. The continued drop in core body temperature after removal from exposure to the cold is known as:
A. Shock
B. After drop
C. Secondary hypothermia
D. Thermoregulation

29. You are with a candidate OEC Technician who is attending to snowboarder who has fallen. The candidate is
very concerned that the snowboarder is shivering vigorously and asks you what this indicates. Which of the
following statements would be your best response?
A. Shivering indicates moderate hypothermia.
B. It is good to try to get a shivering patient to stop because shivering burns energy.
C. Shivering is a sign of mild hypothermia.
D. Individuals who are shivering cannot maintain their normal body temperature.

30. You need to begin rewarming an unresponsive patient who has a core temperature of 93°F. Which of the
following measures would most benefit this patient?
A. Gently massage the patient's arms and legs.
B. Give the patient warm drinks to increase internal body temperature.
C. Place one extremity at a time into warm water.
D. Wrap the patient in several warm blankets.

31. Which of the following patients is losing body heat by the mechanism of convection?
A. A 56-year-old male who is confused, wet and perspiring heavily
B. A 30-year-old male exposed to a light breeze in a cold environment
C. A 63-year-old female who took a drug that depressed her brain function and is sitting in the lodge
D. A 41-year-old female who is intoxicated and has passed out and is buried in the snow

32. Which of the following actions should you take to most effectively decrease a patient's heat loss due to
radiation?
A. Turn off all fans in the room.
B. Apply warm packs to the patient's chest
C. Put a hat/cover on the patient's head.
D. Wipe the patient's skin down with warm water.

33. One of your friends on a hunting trip in Montana has suffered a severe localized cold injury to his hand and
fingers. He does not have any life-threatening conditions and you elect to rewarm the affected areas. Which of the
following actions is most appropriate for your friend?
A. Thaw and rewarm the tissue as quickly as possible.
B. Immerse the hand and fingers in water that is hotter than 120 degrees
C. Vigorously rub the hands and finger

CHAPTER 25
D. Slowly and gradually warm the affected hand and fingers

34. You have been called to aid an alert and oriented male patient whose friends state he spent several hours
locked out of his house in cold temperatures. Given that the primary and secondary assessments have ruled out
immediately life-threatening conditions, you have decided to rewarm the patient. Which of the following actions
would be most appropriate for this patient?
A. Massage the patient's arms and legs after applying warm blankets.
B. Blow hot air from a hair dryer over the patient's arms and legs.
C. Apply hot packs to the patient's chest, groin, and armpits.
D. Place the patient in a tub of warm water.

35. You are assessing a conscious but confused hiker who became lost in the woods on a cold day. Your
assessment shows that he has an open airway, adequate breathing, and a weak radial pulse. His skin is cold to the
touch and he is shivering. OEC Technicians should recognize that the:
A. Shivering actually represents a small seizure, which indicates that the brain is cold
B. Shivering is the only indicator that the patient's core body temperature has fallen below 85 degrees °F.
C. Cold caused the patient's brain to become dysfunctional, as evidenced by the shivering.
D. Shivering is a protective means by which the body is attempting to warm itself.

36. A severely hypothermic patient is in cardiac arrest. Most studies indicate that for patients who have been
submerged in cold water for more than one hour, you should:
A. Focus on rewarming and rapidly transporting the patient.
B. Start cardiopulmonary resuscitation.
C. Does nothing because there is no chance such patients will survive?
D. Provide ventilations but not compressions.

Answer Key: Chapter 25 - Test


1=B, 2=E, 3=D, 4=D, 5=D, 6=B, 7=A, 8=C, 9=D, 10=A, 11=B, 12=C, 13=C, 14=C, 15=C, 16=D, 17=D,
18=A, 19=D, 20=B, 21=C, 22=D, 23=A, 24=B, 25=A, 26=B, 27=C, 28=B, 29=C, 30=D, 31=B, 32=C,
33=A, 34=C, 35=D, 36=A


CHAPTER 25
CHAPTER 26
“Heat Emergencies”

QUIZ

1. A climber has been struck by lightning and is found pulseless and breathless. Which statement about his care
is correct?
A. He is electrically charged, so rescuers should not approach him.
B. CPR is generally helpful for lightning strike victims, although rescue breathing may need to continue
beyond compressions.
C. CPR is generally useless because the heart has probably been “fried."
D. Rescue breathing alone will often restart the heart of a lightning strike victim.

2. A patient presents with weakness, inability to keep up with her hiking group, headache, confusion, nausea,
slightly elevated body temperature, cool skin and is sweating profusely. This patient is most likely suffering from:
A. Heat syncope.
B. Heat cramps.
C. Heat stroke.
D. Heat exhaustion.

3. The usual sequence of heat injury from least to most severe is:
A. Cramps, exhaustion, stroke.
B. Exhaustion, cramps, syncope, stroke.
C. Cramps, stroke, exhaustion.
D. Syncope, exhaustion, cramps, stroke.

4. Multiple climbers were struck by the lightning, but everyone else is only suffering from burns or blindness. Per
protocol, the pulseless, breathless patient should be:
A. Treated after other patients are stabilized but before they are extricated.
B. Flagged as "black" and considered dead.
C. Treated last.
D. Treated first.

5. The heat index table is based on what?


A. The humidity index
B. The body heat index in relation to ambient air temperature and skin surface temperature
C. The highest predicted air temperature
D. The ambient air temperature in a higher-humidity environment and the concurrent health risk

6. Camp, which consists of cottonwood trees, a cold running creek, and plenty of shade, is just 20 more minutes
up the trail. Your best course of action for this patient is to:

CHAPTER 26
A. Stop immediately, hydrate her, create shade from clothing or a tarp, cool her actively with water, and fan
her skin. Do not proceed to camp until she recovers.
B. Keep going slowly but steadily toward camp where she can rest and rehydrate but only if she is sweating.
C. Stop immediately, create shade from clothing or a tarp, actively cool the patient with water, but do not
hydrate her until the nausea dissipates, and then continue on to camp.
D. Give her cool water to drink, let her rest 15 minutes, and continue.

7. A patient grows nauseated and restless, begins to sigh, and yawns. These are warning signs of what impending
condition?
A. Heat stroke
B. Heat exhaustion
C. Heat-related syncope
D. Heat cramps

8. Which statement is correct?


A. The body can physiologically adapt better to cold than heat.
B. The body can physiologically adapt better to heat than cold.
C. The body can adapt equally well to a hot or cold environment.
D. The human body is incapable of adapting to any level of temperature extremes.

9. Which statement is correct regarding adapting to a hot environment versus a cold one?
A. Prior heat exposure improves the body's ability to adapt to a hot environment.
B. Adaption to heat depends largely on modifications in behavior.
C. Adaption to heat depends largely on physiologic acclimatization and less on modifications in behavior.
D. Adaption to heat only works if the patient is young, physically fit, and well hydrated.

Answer Key: Chapter 26 - Quiz


1=B, 2=D, 3=A, 4=D, 5=D, 6=A, 7=C, 8=B, 9=C

TEST

1. You are at a football practice on a very hot summer day. A young player is panting and lying under a tree. His
skin is flushed, and his teammates are continually wiping the sweat off his face, arms, and chest with towels and
offering him fluids to drink. Which of the following statements is most appropriate for this situation?
A. "Let's put his shirt back on to keep him from sweating."
B. "It's OK for him to sweat. It is helping his body cool down."
C. "Don't give him fluids. It will only make him sweat more."
D. "Keep wiping. You are helping him retain water that his body needs."

2. Which of the following patients would have the highest priority for transport?
A. A 28-year-old who is responsive, has tachycardia, and has cool, moist skin
B. A 24-year-old who is intoxicated and is complaining of pain and spasms in his legs

CHAPTER 26
C. A 34-year-old who is responsive, has cool diaphoretic skin, and is complaining of nausea
D. A 19-year-old who is lethargic, has hot dry skin, and has tachycardia

3. You are in the aid room with a patient who is unresponsive and has hot, dry skin. His friends state that he has
been drinking and passed out in the hot sun for several hours. ALS has been called. Which of the following
actions would be most beneficial to this patient now?
A. Try to give the patient sugar because he could be having a diabetic reaction.
B. Prepare the patient for vomiting by placing him in a prone position.
C. Place cold packs on the patient's groin and armpits.
D. Gently pour cold water over the patient to rapidly cool him down.

4. Your friend, who is a landscaper, tells you that he got really sunburned at work today. He describes his torso
and face as being very red, and he has blisters on his back. He asks you what to do. Based on his description, your
recommendation would be to:
A. Take ibuprofen for the discomfort.
B. Cover the sunburned areas with zinc oxide.
C. Apply cool compresses and seek medical attention.
D. Apply a thick layer of an aloe-based lotion on the sunburned areas.

5. A group of three golfers have been struck by lightning on a golf course. Two of the golfers are responding; the
third is in cardiac arrest. You are the lone witness. When deciding which patient to treat first, you should
remember that:
A. You can't do everything by yourself; run and get help.
B. You must somehow provide head stabilization for all three victims.
C. In a lightning strike, patients who are apparently dead should be treated first.
D. To save the most people, the patient in cardiac arrest should be treated last.

6. A group of three golfers have been struck by lightning on a golf course. Two of the golfers are responding; the
third is in cardiac arrest. You are the lone witness. When deciding which patient to treat first, you should
remember that:
A. you must somehow provide head stabilization for all three victims.
B. you can't do everything by yourself; run and get help.
C. in a lightning strike, patients who are apparently dead should be treated first.
D. to save the most people, the patient in cardiac arrest should be treated last.

7. Your friend, who is a landscaper, tells you that he got really sunburned at work today. He describes his torso
and face as being very red, and he has blisters on his back. He asks you what to do. Based on his description, your
recommendation would be to:
A. apply a thick layer of an aloe-based lotion on the sunburned areas.
B. cover the sunburned areas with zinc oxide.
C. take ibuprofen for the discomfort.
D. apply cool compresses and consult a physician.

8. The body cools itself primarily by:


A. Radiation
B. Respiration
C. Conduction
D. Evaporation

CHAPTER 26
9. You are treating a patient with suspected heat cramps. Which of the following treatments is the usual
recommended treatment?
A. Have the patient drink one quart of cold tap water every 15 minutes until he has to urinate.
B. Have the patient drink a mixture of 1/4 to 1/2 teaspoon of table salt in a quart of cool water.
C. Have the patient drink a mixture of one tablespoon of table salt in one quart of Gatorade.
D. Administer salt tablets every 15 minutes until the cramping subsides.

10. You are treating a patient with suspected heat cramps. Which of the following treatments is the usual
recommended treatment?
A. Have the patient drink a mixture of 1/4 to 1/2 teaspoon of table salt in a quart of cool water.
B. Administer salt tablets every 15 minutes until the cramping subsides.
C. Have the patient drink one quart of cold tap water every 15 minutes until he needs to urinate.
D. Have the patient drink a mixture of one tablespoon of table salt in one quart of Gatorade.

11. You recognize that heat cramps are the probable cause of a patient's problem when the patient makes which of
the following statements?
A. "I cannot remember what day it is."
B. "I am having a hard time moving my fingers."
C. "My neck feels as though it is in spasm."
D. "I have pain in my belly and legs."

12. A man has been struck by lightning at a picnic. As you arrive on scene, his friend states that the patient was
in cardiac arrest after the strike, but after one minute of CPR he has a heartbeat and weak respirations. The
patient remains unresponsive. Which of the following actions should you take immediately?
A. Provide care to the burn injury on the patient's back.
B. Perform the jaw-thrust maneuver.
C. Attach an automated external defibrillator.
D. Initiate positive pressure ventilation.

13. The signs of heat exhaustion are:


A. Warm and very sweaty skin, elevated body temperature, headache
B. Hot dry skin, elevated heart rate, elevated body temperature
C. Diaphoretic skin, decreased heart rate, hunger
D. Cool and clammy skin, bradycardia, slow respirations

14. You are at a park with your family when you hear a call for help. There is a teenager who is "sick." On scene
you find a 16-year-old boy who is mildly confused. Friends state that they have been playing basketball for most
of the morning and afternoon. The temperature is in the 90s and the humidity is high. The boy's airway is patent
and his breathing rate is elevated but adequate. His pulse is weak and rapid. Observation of his skin reveals it to
be pale, cool, and moist. Which of the following would you do first for this patient?
A. Administer salt tablets by mouth every 15 minutes.
B. Lay him in the shade in a supine position, and elevate his feet.
C. Get some newspaper and fan him vigorously.
D. Lay him down in a side-lying position.

15. When cooling a patient with a possible heat stroke, which of the following findings would be of most concern
to a rescuer?

CHAPTER 26
A. Observing your partner applying an ice pack to the patient's neck
B. Observing your partner giving the patient two aspirin to bring down his temperature
C. An increase in blood pressure to 100/80
D. A decrease in heart rate from 140 beats per minute to 120 beats per minute

Answer Key: Chapter 26 - Test


1=B, 2=D, 3=C, 4=C, 5=C, 6=C, 7=D, 8=D, 9=B, 10=A, 11=D, 12=B, 13=A, 14=B, 15=B

CHAPTER 26
CHAPTER 27
“Plants and Animals”

QUIZ

1. A jellyfish sting can NOT be treated with:


A. Baking soda.
B. Fresh water.
C. Vinegar.
D. Isopropyl alcohol.

2. Ingested plants almost never cause:


A. Cardiogenic effects.
B. Kidney failure.
C. Spasms or seizures.
D. Unconsciousness.

3. Autumn crocus, lily of the valley, yew, foxglove, and belladonna can all cause:
A. Rashes.
B. Cardiogenic effects.
C. Salivation.
D. Seizures.

4. Which of the following diseases is transmitted by mosquitoes?


A. Hepatitis B
B. Lyme disease
C. Dengue fever
D. HIV

5. Which of the following mushrooms are least likely to cause death?


A. Green-spored Lepiota
B. Amanitas
C. False morels
D. Little browns

6. Jack-O-lantern mushrooms are most likely to cause:


A. Skin rashes.
B. Cardiac arrest.
C. Violent GI effects that are commonly fatal.
D. Violent GI effects that are rarely fatal.

CHAPTER 27
Answer Key: Chapter 27 - Quiz
1=A, 2=B, 3=B, 4=C, 5=A, 6=D

TEST

1. Which of the following actions is not an appropriate treatment measure for a sting by a marine creature?
A. Rubbing the affected area to remove any spines
B. Applying a tourniquet if required for controlling life-threatening hemorrhaging
C. Irrigating the affected area with fresh water
D. Rinsing the affected area with a vinegar solution

2. Its early fall and you have been backpacking on part of the Appalachian Trail with some friends. One member
of the group comes up to you holding some bright orange mushrooms that resemble pumpkins and asks if you
think they are safe to eat. Based on your training, you would respond with which of the following statements?
A. "Those are very toxic and would definitely kill you if you ate them."
B. "Those mushrooms can cause some pretty severe vomiting, diarrhea, cramps and loss of coordination."
C. "Most toxic mushrooms are brown, so those are probably all right to eat."
D. "If you just eat the mushroom cap, you should be OK.”

3. Attacks by which of the following creatures do not cause both soft-tissue injuries and fractures?
A. Swordfish
B. Crocodile
C. Alligators
D. Sharks

4. Most reptile-related injuries are caused by:


A. Crocodiles
B. Snakes
C. Alligators
D. Sharks

5. DEET (N, N-Diethyl-meta-toluamide) is quite often effective as a (n):


A. Antidote for snake bites.
B. Repellent for mosquitoes.
C. Anti-inflammatory medication for tick bites.
D. Analgesic for relieving pain caused by bee stings.

6. When a young child at a neighborhood cookout cries out that he was stung by a bee on his forearm, you
should:
A. Scrub the area with soap and water to remove the stinger.
B. scrape the stinger off the skin with a firm flat object such as a credit card and then apply ice to reduce the
swelling and pain.
C. ask if anyone has an epinephrine auto-injector and assist in giving the injection

CHAPTER 27
D. Immobilize the arm and place it at the level of the child's heart to prevent spread of the venom.

7. You and your friends have been examining cactus plants when your friend gets stuck by one of the plant's
spines. You recognize that a cactus spine can:
A. Inject a toxin that causes a systemic reaction.
B. get stuck in the skin and cause pain and redness in the affected area
C. Cause an immediate anaphylactic reaction in some individuals.
D. Just be uncomfortable but will not cause a reaction.

8. Which of the following actions is not an appropriate treatment measure for a sting by a marine creature?
A. Rubbing the affected area to remove any spines
B. Applying a tourniquet if required for controlling life-threatening hemorrhaging
C. Rinsing the affected area with a vinegar solution
D. Irrigating the affected area with fresh water

9. You are at a cookout with friends when one of the adults starts yelling excitedly that her 4 year old appears to
have been nibbling on the azalea bush in the front yard. Based on your knowledge of this plant's toxicity, you
would say which of the following things to the parent?
A. "Azaleas can be toxic. let's call poison control hotline right now."
B. "The worst that will happen is some nausea and vomiting that will teach your child not to eat azalea leaves
again.”
C. "You need to worry only if you see your child starting to get a rash.”
D. "You have nothing to worry about. Many people grow azaleas because they know they are safe."

10. A specific toxin or poisonous secretion of an animal that is usually transmitted by a bite or sting is known as:
A. Antigen
B. Nettle
C. Venom
D. Bullae

11. When caring for someone who has been attacked by a large animal such as a moose or bear, you should
assume that:
A. Unless there is a bite, there is no potential for infection.
B. The potential for spinal injury exists.
C. The animal probably has rabies or it would not have attacked.
D. Bleeding from claw injuries is the most dangerous injury.

12. Which one of the following groups is not included in the three main groups of mushrooms that are dangerous
to humans?
A. Amanitas
B. Big white mushrooms
C. Little brown mushrooms
D. False morels

13. Which of the following signs and symptoms is not a characteristic of a bee, wasp, or hornet sting?
A. Pain
B. Nausea and vomiting
C. Joint aching

CHAPTER 27
D. Swelling

14. When caring for someone who has been attacked by a large animal such as a moose or bear, you should
assume that:
A. the animal probably has rabies or it would not have attacked.
B. bleeding from claw injuries is the most dangerous injury.
C. unless there is a bite, there is no potential for infection.
D. the potential for spinal injury exists.

15. The phrase "red on yellow, kill a fellow; red on black, venom lack" is helpful in identifying which of the
following kind of poisonous snakes?
A. Coral snakes
B. Rattlesnakes
C. Pit vipers
D. Copperheads

16. You are working at the first-aid station at the local scout camp. A counselor brings in two young boys that he
thinks were playing in an area where poison oak was found. Which of the following actions would not be a part of
your treatment for the possible exposure to poison oak?
A. Using a commercial cleanser such as Oak-N-Ivy to wash the boys' hands and arms
B. Applying a topical cream such as bacitracin to the boys' hands and arms
C. Putting on non-latex gloves before examining the boys
D. Washing the boys' hands and arms with hot water and soap

17. The bite of which of the following spiders that are indigenous to Australia may produce significant toxicity
and thus should be treated as potentially life threatening?
A. Funnel web spider
B. Wolf spider
C. Bark scorpion
D. Black widow spider

18. You are creating a Power Point presentation on rattlesnakes. Which of the following statements would not be
included in your bullet points?
A. Rattlesnake venom can prevent blood from clotting.
B. The bite of a rattlesnake is typically painful.
C. Rattlesnake attacks are typically unprovoked.
D. Rattlesnake venom destroys local tissue.

19. Your neighbor has been cleaning up the wooded area around his yard and burning brush and weeds. He tells
you that he is trying to rid the area of poison ivy. You recognize that his actions could:
A. Cause significant respiratory problems for anyone exposed to the smoke.
B. Be a great solution that you should use around your house.
C. Temporarily solve his problem but will not permanently kill the poison ivy.
D. Encourage the poison ivy to grow into any area where the smoke travels.

20. On a camping trip with your family, your children ask if they can pick some plants for part of their wilderness
dinner. Based on your OEC training, you explain to your children that they need to be careful picking wild plants
and flowers, and that:

CHAPTER 27
A. Leaves may be poisonous but most berries found in the woods are safe to eat.
B. The worst that can happen from eating a bad plant is you will get sick to your stomach.
C. Plants that do not cause any local reaction when they are picked are safe to eat.
D. Some plants can be very toxic, so it is best not to eat anything unless you are sure what it is.

21. Plants most commonly cause toxic reactions through which of the following routes?
A. Ingestion and inhalation
B. Topical contact and injection
C. Inhalation and injection
D. Ingestion and topical contact

22. Marine life can cause varying degrees of injury and illness. Which of the following marine creatures can cause
respiratory failure and cardiovascular collapse in humans?
A. Moray eels
B. Biting fish
C. Jellyfish
D. Spiny fish

23. You have been doing some spring-cleaning in your yard, and the next day you notice a red, itchy rash
developing on your arms. Your doctor tells you that you have poison ivy. You recognize that this reaction occurs
because:
A. Your body was unable to create an antigen.
B. Venom from the plant reacted with your body.
C. Poison from the plant entered your bloodstream.
D. A toxin, plant oil, came in contact with your skin.

24. Which of the following statements regarding mammals is correct?


A. Mountain lions, lynx, and bobcats travel and attack humans in packs.
B. Grizzly bears will attack humans only when provoked.
C. Most injuries from moose and elk are caused when the animal charges a hiker.
D. Wild dogs, wolves, and coyotes usually attack humans only when they are sick, protecting their young, or
injured.

25. A spider bite that can resemble a "bull's-eye target" is most often caused by a:
A. Brown recluse spider
B. Black widow spider
C. Funnel web spider
D. Hobo spider

26. You are teaching a basic OEC class and need to explain toxins. Which of the following statements is the best
description of a toxin?
A. It is a poison made by a living creature, including plants and animals.
B. It is anything ingested that causes an anaphylactic reaction.
C. It is a usually transmitted by a bite or a sting.
D. It is a poison that comes from a chemical.

CHAPTER 27
27. You will be taking a group of scouts on a weekend camping trip. You recognize that ticks are typically active at
this time of year, so you want to teach the scouts about them. Which of the following statements about ticks is
false?
A. Campers should check their skin regularly, especially in the evening before going to bed.
B. A red circle with a blanched white center is characteristic of the rash caused by a tick bite.
C. The typical disease transmitted by ticks is Lyme disease.
D. Tick bites are painful, so you will know when you have been bitten.

28. You are reviewing care for snake bites before your wilderness trip with friends. You note that appropriate
treatment would include:
A. Applying ice to the bite wound to decrease pain and swelling.
B. Sucking the poison out of the bite wound.
C. Applying a tourniquet above the bite wound to prevent spread of the venom.
D. Wrapping the affected extremity snugly with an elastic bandage, proximal to distal, immobilizing the
extremity and then placing it at the level of the heart.

Answer Key: Chapter 27 - Test


1=A, 2=B, 3=A, 4=B, 5=B, 6=B, 7=B, 8=A, 9=A, 10=C, 11=B, 12=B, 13=B, 14=D, 15=A, 16=B, 17=A,
18=C, 19=A, 20=D, 21=D, 22=C, 23=D, 24=D, 25=B, 26=A, 27=D, 28=D


CHAPTER 27
CHAPTER 28
“Altitude Emergencies”

QUIZ

1. Which of the following is NOT a factor at high altitude that can cause altitude illness?
A. Reduced atmospheric humidity
B. Reduced partial pressure of oxygen
C. Fewer oxygen molecules per breath
D. Barometric pressure decrease

2. Which of these conditions at altitude should be presumed due to HACE?


A. Unsteadiness or inability to maintain one's balance
B. Loss of memory
C. Dilated pupils
D. Cough and shortness of breath

3. Which is the most effective treatment for altitude illness?


A. Descent
B. Rest and hydration
C. Continued slow ascent
D. Oxygen administration

4. Which of the following does NOT affect risk of altitude illness?


A. Family history
B. Use of alcohol or other sedatives
C. Rate of ascent
D. Hydration

Answer Key: Chapter 28 - Quiz


1=A, 2=A, 3=A, 4=A

TEST

1. The height or vertical elevation above a fixed point is known as:


A. Ataxia
B. Elevation
C. Altitude

CHAPTER 28
D. Ascent

2. You are with a group of hikers on the third day of an 11,000-foot mountain ascent. One of the hikers has not
been feeling well for a couple of days. Today, members of the group notice that he is having difficulty getting
dressed and speaking. From your training in outdoor emergency care, you recognize that these signs and
symptoms are most often associated with:
A. Frostbite
B. HACE
C. Acute mountain sickness
D. Peripheral neuropathy

3. Labored breathing at rest and audible chest congestion herald the development of a serious, potentially life-
threatening stage of what altitude-related condition?
A. Acute mountain sickness
B. Khumbu cough
C. Peripheral edema
D. HAPE

4. In an otherwise healthy individual, the presence of a headache and feelings of sickness at high altitude is
known as:
A. Acute mountain sickness
B. A high-altitude migraine
C. High-altitude pulmonary edema
D. Acute mountain encephalitis

5. You are asked to speak to a college group that is planning a ski trip to the Alps. In order to help them plan for
reducing their risks for developing altitude-related illnesses, you suggest all of the following except:
A. Sleep at an elevation that is lower than where they will be skiing.
B. Do a lot of heavy physical exertion early in the trip to help them acclimate.
C. Plan a layover day at between 6,000 feet and 8,000 feet.
D. Drink 3-4 quarts of fluid a day.

6. A patient at a mountain ski resort has notable shortness of breath. She denies any past medical history and
takes no medications. After applying oxygen, you realize that the fundamental treatment to helping this patient
improve is to:
A. Position her in a left lateral recumbent position.
B. Place her in a supine position with her legs elevated.
C. Administer a respiratory medication by a metered-dose inhaler.
D. Descend to a lower altitude.

7. In order to assess for a key symptom of HACE, you would ask patients to:
A. Read an eye chart.
B. Tell you if their rings are fitting tightly.
C. Walk a straight line heel to toe.
D. Take a deep breath and let you know if it hurts.

8. Your group of climbers has reached an elevation of 8,500 feet. One of the climbers is increasingly short of
breath and now has audible chest congestion. Your party has a limited supply of oxygen, which you immediately

CHAPTER 28
apply to the climber at 15 LPM via a nonrebreather mask. Based on your assessment that the climber has HAPE,
you recognize that the next necessary treatment is:
A. a rapid descent of at least 1,500 to 3,000 feet.
B. to administer diuretics such as Diamox to reduce the patient's pulmonary edema.
C. to get the patient to a physician who can administer dexamethesone.
D. to place the patient in the shock position.

9. Your group of climbers has reached an elevation of 8,500 feet. One of the climbers is increasingly short of
breath and now has audible chest congestion. Your party has a limited supply of oxygen, which you immediately
apply to the climber at 15 LPM via a nonrebreather mask. Based on your assessment that the climber has HAPE,
you recognize that the next necessary treatment is:
A. to place the patient in the shock position.
B. a rapid descent of at least 1,500 to 3,000 feet.
C. to administer diuretics such as Diamox to reduce the patient's pulmonary edema.
D. to get the patient to a physician who can administer dexamethesone.

10. You are working at the summit of a 9,000-foot mountain. A 60-year-old woman is brought to you complaining
of headache, fatigue, and shortness of breath. You prepare to do your assessment and recognize that your goal is
to:
A. Determine whether this is an emergent condition so that you can initiate life-saving treatment.
B. Find out if she has an inhaler she can use to relieve her shortness of breath.
C. Gets the patient lying down to relieve her presenting symptoms.
D. Give her some ibuprofen to ease her headache.

11. You are accompanying a team of hikers up a high mountain. The next morning you are summoned to a tent
and find one of the climbers confused and complaining of a headache. His airway is patent and his respirations
are 24 per minute. He has no medical history and was in good health until found ill this morning by his friend.
Suspicious of high-altitude cerebral edema (HACE), which of the following actions should you takes?
A. Immobilize the patient
B. Administer oral glucose
C. Insert an oropharyngeal airway
D. Provide high-flow oxygen

12. Given that 50 percent of patients with HAPE also have symptoms of AMS, it is important that you ask
patients if they have had which of the following groups of signs/symptoms?
A. Harsh cough, itchy skin, nausea
B. Difficulty sleeping, vomiting, ataxia
C. Headache, blurred vision, peripheral edema
D. Fatigue, nausea, difficulty sleeping

13. You are working at a mountain resort and are presented with a 50-year-old woman who states that she does
not feel well. From your assessment, which of the following findings would seemingly indicate that the patient is
being adversely affected by the high altitude?
A. Vomiting after eating large meals
B. A heart rate of 62 beats per minute
C. A complaint of feeling short of breath on exertion
D. An elevated blood glucose level with no history of diabetes

CHAPTER 28
14. The concentration of oxygen at sea level is approximately:
A. 40 percent
B. 35 percent
C. 21 percent
D. 43 percent

15. Which of the following processes does not occur during acclimatization?
A. Dilation of pulmonary blood vessels
B. An increase in respiratory rate and depth
C. An increase in red blood cell production
D. An increase in heart rate

16. You have a 32-year-old male patient who just arrived at the aid room on the top of a 9,000-foot peak. He is
exhibiting signs of altitude sickness and HAPE. You recognize that the final key to a successful outcome of
treatment is to:
A. Start him on O2 while waiting for him to be transported
B. Prevent further body cooling by covering him with blankets.
C. Get him to lie down so his symptoms do not worsen.
D. Recognize the patient's signs and descend to a lower elevation.

17. You are asked to speak to a college group that is planning a ski trip to the Alps. In order to help them plan for
reducing their risks for developing altitude-related illnesses, you suggest all of the following except:
A. sleep at an elevation that is lower than where they will be skiing.
B. do a lot of heavy physical exertion early in the trip to help them acclimate.
C. drink 3-4 quarts of fluid a day.
D. plan a layover day at between 6,000 feet and 8,000 feet.

18. In order to enjoy their upcoming skiing trip, your family has gathered information about risk factors for
high-altitude illnesses. Which of the following would not increase the possibility of developing altitude-related
illness?
A. COPD.
B. A history of angina.
C. The lack of regular exercise.
D. A diet that is high in carbohydrates.

19. What percentage of patients who have HAPE do not develop AMS first?
A. 10 percent
B. 20 percent
C. 50 percent
D. 40 percent

20. You are teaching a class on acute mountain sickness. When one of your students asks you to explain the term
acclimatization in relation to AMS, your best response would be which of the following statements?
A. "Physiologic changes that decrease the body's need for slow ascents."
B. "Physiologic changes that decrease the body's need for oxygen at high altitudes."
C. "Physiologic adjustments that increase the delivery of oxygen to cells."
D. "Physiologic changes that increase the body's ability to adjust to extreme changes in temperature."

CHAPTER 28
21. As altitude increases:
A. The partial pressure of oxygen decreases.
B. The concentration of oxygen per cubic foot decreases.
C. Air density becomes greater.
D. Barometric pressure increases.

22. Your family is planning a ski trip to a resort that is at about 10,000 feet of elevation. You suggest that you
spend the first two nights of the trip at a hotel where the elevation is about 6,500 feet. When your children
protest that they are in good shape and want to go immediately to the resort, you explain that you are concerned
about altitude sickness and that:
A. "Your mom and I will exercise more before the trip so that maybe we can reduce the delay to one night."
B. "Physical fitness does not necessarily prevent altitude sickness."
C. "Just because you children are fit and don't need the gradual change doesn't mean your mom and I don't
need it.”
D. "I'll see if I can get some medication so that we don't need to stop over at the lower elevation."

23. You have a 32-year-old male patient who just arrived at the aid room on the top of a 9,000-foot peak. You
think he may be exhibiting signs of altitude sickness. You recognize that the key to a successful outcome of
treatment is to:
A. recognize the patient's signs and descend to a lower elevation.
B. get him to lie down so his symptoms do not worsen.
C. start him on oxygen via a nasal cannula.
D. prevent further body cooling by covering him with blankets.

24. The most effective method for preventing high-altitude illness is to:
A. Make gradual ascents.
B. Avoid drinking alcohol.
C. Take medications such as Diamox.
D. Maintain adequate hydration and eat a high-carbohydrate diet.

Answer Key: Chapter 28 - Test


1=C, 2=B, 3=D, 4=A, 5=B, 6=D, 7=C, 8=A, 9=B, 10=A, 11=D, 12=D, 13=C, 14=C, 15=A, 16=D, 17=B,
18=D, 19=C, 20=C, 21=A, 22=B, 23=A, 24=A


CHAPTER 28
CHAPTER 29
“Water Emergencies”

QUIZ

1. The mammalian diving reflex is most prominent among which segment of the population?
A. Females
B. Geriatric
C. Males
D. Pediatric

2. What is the "mammalian diving reflex"?


A. A protective response during which the blood vessels constrict, blood is shunted to the heart and brain,
and the metabolic and heart rate slow considerably
B. An ability of all mammals to be able to submerse under water for a given amount of time dependent upon
health, fitness, and other cardiovascular factors
C. A gag reflect initiated by the epiglottis, which keeps water from entering into the lungs
D. A response to laryngospasm

3. Barotrauma is best defined as:


A. Any illness caused by deep-water submersion.
B. Tissue damage within any of the air-filled structures of the body.
C. A ruptured eardrum.
D. Nitrogen bubbles in the blood.

4. A diver returns to the beach and you notice that the sclera of his eyes are bright red and bloodshot. This is due
to:
A. Mask reverse squeeze.
B. Small particles in the water such as sand, which irritate the eyes.
C. A bacterial infection of the eye.
D. A symptom of narcosis.

5. Boyle's law explains which complication of diving?


A. Ruptured lungs
B. Narcosis
C. Barotrauma
D. Hypothermia

6. Drowning is best defined as:


A. Suffocation by submersion in liquid, resulting in death within 24 hours following removal from the liquid.
B. Immediate death due to laryngospasm or inhalation of liquid.
C. Death by wet or dry asphyxiation.

CHAPTER 29
D. Filling of the lungs with fluid.

7. A diver is exhibiting impaired judgment, irrational berhavior, and altered levels of consciousness. This is most
likely symptomatic of:
A. Oxygen narcosis.
B. Compression sickness.
C. Nitrogen narcosis or "rapture of the deep."
D. Barotrauma.

8. The difference between a 'dry' versus a 'wet' drowning includes which of the following?
A. Whether unconsciousness was due to asphyxia or aspiration
B. Whether the patient was conscious or unconscious when inhaled liquid entered into the lungs
C. Laryngospasm, which seals the airway, versus inhalation of fluid into the lungs
D. The location where the drowning occurred

9. A teenager suffered what appeared to be a dry drowning and was successfully resuscitated. He appears to be
fine. The next best course of action would be to:
A. Arrange for immediate transport to the nearest medical facility for follow-up care and observation.
B. Tell his friends to keep an eye on him but allow him to continue swimming.
C. Release him to his parents' custody and assure them he is fine but suggest they call the doctor if they are
worried.
D. Urge him to follow up with his family physician.

Answer Key: Chapter 29 - Quiz


1=D, 2=A, 3=B, 4=A, 5=A, 6=A, 7=C, 8=C, 9=A

TEST

1. For any submersion injury that is not a life-threatening water-related emergency, a complete secondary
assessment is performed using:
A. SAMPLE
B. AVPU
C. OPQRST
D. DCAP-BTLS

2. Assessment of patients with water-related emergencies is not remarkably different from assessment of patients
with other emergencies. In addition to BSI, OEC Technicians may:
A. need to swim in deep water to retrieve a patient.
B. need to wear an SCBA (Scott Air-Pak).
C. need to don a personal floatation device.
D. be required to wear a cold-water rescue suit and stand on a flotation ramp.

3. Which of the following strategies does not limit or mitigate risk in water-based activities?
A. Avoiding swimming alone

CHAPTER 29
B. Avoiding alcohol consumption around water
C. Checking ice thickness before traversing it
D. Entering swiftly moving water accompanied by a buddy

4. For any submersion injury that is not a life-threatening water-related emergency, a complete secondary
assessment is performed using:
A. OPQRST.
B. SAMPLE.
C. DCAP-BTLS.
D. AVPU.

5. Patients with arterial gas embolism or decompression sickness should be transported by ground to a facility
that:
A. is a trauma center with a rehabilitation wing.
B. has a hyperbaric chamber.
C. has advanced respiratory specialists.
D. is nearest to the scene.

6. Which of the following strategies does not limit or mitigate risk in water-based activities?
A. Avoiding alcohol consumption around water
B. Checking ice thickness before traversing it
C. Avoiding swimming alone
D. Entering swiftly moving water accompanied by a buddy

7. A patient who is conscious and breathing has been pulled from a cold stream. To decrease the patient's loss of
heat via the mechanism of conduction, OEC Technicians should immediately:
A. encourage the patient to stop shivering.
B. provide positive-pressure ventilation.
C. apply hot packs to the patient.
D. thoroughly dry the patient and then apply a blanket.

8. Patients with arterial gas embolism or decompression sickness should be transported by ground to a facility
that:
A. Has a hyperbaric chamber
B. Has advanced respiratory specialists
C. Is nearest to the scene
D. Is a trauma center with a rehabilitation wing

9. Decompression sickness can cause:


A. Severe muscle and joint pain
B. Swelling of the tongue
C. Swelling of the cricoid cartilage
D. Swelling of the vocal cords

10. The final event in the series of events in drowning is:


A. A seizure leading to unresponsiveness leading to respiratory arrest.
B. Unresponsiveness leading to respiratory arrest.
C. Altered mental status leading to respiratory arrest.

CHAPTER 29
D. Cardiac arrest.

11. Drowning is defined as:


A. Death by traumatic injury to the airway.
B. Fluid oxidation of the lungs.
C. Respiratory impairment due to trauma.
D. Suffocation by submersion in a liquid.

12. Trauma from a dive injury that results in tissue damage within any air-filled structure of the body is called:
A. Internal dive trauma
B. Barotrauma
C. Upper pressure trauma
D. Tonicity

13. A patient who appears dead due to a deep cold-water drowning should be:
A. transported to a medical facility while being given CPR.
B. transported to a medical facility while being given rescue breathing only.
C. warmed up prior to transport and then given continuing CPR.
D. left at the scene for the medical examiner to pick up.

14. Two of the most important conditions that can complicate drowning and near-drowning are:
A. Water depth and pH
B. Time of day and air temperature
C. Undertow and turbidity
D. Water temperature and salinity

15. Squeeze is a term for excessive external pressure on various parts of the body. Reverse squeeze:
A. Is pressure created when holding one's breathe?
B. Is too much pressure from within a body compartment or organ?
C. Is pressure experienced within a hyperbaric chamber?
D. Occurs when two body organs compress each other.

16. The mammalian diving reflex is most prominent in:


A. Adults in warm water
B. Experienced swimmers
C. Deep-water divers
D. Young children

17. A patient who appears dead due to a deep cold-water drowning should be:
A. Transported to a medical facility while being given CPR.
B. Warmed up prior to transport and then given continuing CPR.
C. Transported to a medical facility while being given rescue breathing only.
D. Left at the scene for the medical examiner to pick up.

18. During assessment of the water-related emergency for any submersion injury, care should be taken to:
A. assess the airway but not to treat the patient.
B. use AEIOU tips.
C. assess the water temperature.

CHAPTER 29
D. protect the patient's spine to prevent additional neurological injury.

19. Large nitrogen bubbles can act as a (n):


A. Carrier for glucose
B. Embolus that blocks blood flow
C. Oxygen inhibitor
D. Carbon dioxide carrier

20. Two types of submersion injuries are:


A. The mammalian diving reflex and near-drowning.
B. Diving and drowning.
C. Drowning and near-drowning.
D. Delayed submersion and advance drowning syndrome.

21. Patients who die 24-72 hours after a water emergency are said to die from:
A. Secondary drowning
B. Drowning
C. Near-drowning
D. Post-drowning

22. The cause of an arterial gas embolism (AGE) is:


A. A rapid ascent by a diver
B. A rapid descent by a diver
C. Diving below 100 feet for longer than 30 minutes
D. Diving below 100 feet

23. Assessment of patients with water-related emergencies is not remarkably different from assessment of
patients with other emergencies. In addition to BSI, OEC Technicians may:
A. Need to wear an SCBA (Scott Air-Pak).
B. Need to swim in deep water to retrieve a patient.
C. Need to don a personal floatation device.
D. Be required to wear a cold-water rescue suit and stand on a flotation ramp.

24. A patient who is conscious and breathing has been pulled from a cold stream. To decrease the patient's loss of
heat via the mechanism of conduction, OEC Technicians should immediately:
A. Provide positive-pressure ventilation.
B. Encourage the patient to stop shivering.
C. Thoroughly dry the patient and then apply a blanket.
D. Apply hot packs to the patient.

25. Decompression sickness (DSC) or the "bends" is a (n):


A. Buildup of nitrogen bubbles in the body
B. High level of carbon dioxide in the body
C. Excessive O2 Level in the body
D. Buildup of nitrous acid in the body

26. The distinction between dry drowning and wet drowning is based on:
A. The closing of the epiglottis.

CHAPTER 29
B. How much fluid enters the lungs.
C. The pressure against the cricoid cartilage.
D. The amount of trauma to the laryngopharynx.

27. When arterial gas embolism (AGE) occurs, the gas within the lungs:
A. Causes an open pneumothorax
B. Causes the trachea to close
C. Expands, rupturing alveoli
D. Contracts and the bronchioles start to spasm

28. During assessment of the water-related emergency for any submersion injury, care should be taken to:
A. Assess the water temperature
B. Assess the airway but not to treat the patient
C. Protect the patient's spine to prevent additional neurological injury
D. Use AEIOU tips

29. Pure water without salt is referred to as:


A. aquatonic.
B. hypertonic.
C. hypotonic.
D. aquafied.

Answer Key: Chapter 29 - Test


1=D, 2=C, 3=D, 4=C, 5=B, 6=D, 7=D, 8=A, 9=A, 10=D, 11=D, 12=B, 13=A, 14=D, 15=B, 16=D, 17=A,
18=D, 19=B, 20=C, 21=A, 22=A, 23=C, 24=C, 25=A, 26=B, 27=C, 28=C, 29=C

CHAPTER 29
CHAPTER 30
“Pediatrics”

QUIZ

1. OECTs should be familiar with the varying aspects of child growth and development for all of the following
reasons EXCEPT:
A. A child's normal response to an adult depends on the child's developmental age.
B. Assessment norms vary tremendously from one age child to another.
C. Younger children are easier to deal with than older children.
D. Your OEC instructors are sadists.
E. To determine how to elicit a child's cooperation for assessment or treatment.

2. A child's head is proportionally larger and heavier than an adult's, which can cause which of the following
problems?
A. Greater heat loss, greater risk of head injury, and greater risk for airway management with a spine-boarded
patient
B. Greater risk of heat loss and c-spine or thoracic trauma
C. Greater risk of c-spine trauma
D. Greater risk of spinal trauma and skull fracture

3. Which of the following pediatric trauma patients should cause rescuers the most concern?
A. The 12-year-old who is screaming at his friends for causing him to fall
B. The 4-year-old who is screaming in the aftermath of a fall
C. The 3-year-old who engages with her mother but refuses to look at you
D. The 5-year-old who is lying quietly and whimpering

4. Which statement about blunt chest trauma and children is true?


A. Because they are smaller, a child's ribs can fracture easily and penetrate underlying organs.
B. Children rarely suffer from blunt trauma injuries.
C. A child's ribs are less likely to fracture than an adult's but can generate force inward and cause internal
injuries.
D. Penetrating chest trauma is more likely to occur in children than blunt chest trauma.

5. The number one cause of death in children is:


A. Trauma.
B. Dehydration.
C. Disease.
D. Loss of the airway.

6. The most appropriate method to get a 6-year-old child to cooperate with treatment would be to:
A. Ask the child if you may splint the child's arm.

CHAPTER 30
B. Ask the child if he or she would prefer you or your partner to splint the arm.
C. Not ask the child's permission to splint the arm; just tell the parent what you are doing and why, and then
do it quickly and efficiently.
D. Ask the child's parent if you may splint the child's arm.

7. Which statement regarding pediatric patients is correct?


A. They are largely miniature adults.
B. The differences between children and adults are static and by-and large do not change until adulthood.
C. There are inherent differences in intellectual capacity, size, proportion, and metabolism that make pediatric
patients unique.
D. The basic difference between an adult and pediatric patient is the size of the body, structures, organs, and
organ systems.

8. The best method of assessing a 2-year-old includes:


A. Having the parent hold and distract the child during examination.
B. Having the child lie quietly during assessment.
C. Have the parent hold the child down during examination.
D. Ignoring the child and directing all questions directly to the parent.

9. The most important physiologic difference(s) between an adult and pediatric patient that an OECT needs to
keep in mind is:
A. The size of the airway and the mechanism of breathing.
B. The size, weight, and metabolic rate.
C. The head-to-torso ratio.
D. A child's limited intellectual capacity.

10. All of the following concerning hypovolemic shock in children are true EXCEPT:
A. Children's arteries are able to constrict more quickly than an adult's, so they can initially compensate
better for shock.
B. Children have proportionately less blood than adults yet can bleed out at the same rate.
C. A stable child can spiral rapidly into shock.
D. Hypovelmia in children can only be caused by direct blood loss and not indirect fluid loss such as through
vomiting or diarrhea.

11. You suspect that a child you are examining may be the victim of abuse by the parent. Your best course of
action is to:
A. Alert law enforcement or child protection authorities.
B. Confront the parent.
C. Alert a hospital social worker, the child's teacher, or a physician.
D. Say nothing and mind your own business.

12. A child's tongue is proportionally larger and more bulbous in relation to the oral cavity until about what age?
A. 12
B. 18
C. 8
D. 4

13. When assessing a child's breathing, the most important information to determine is:

CHAPTER 30
A. The respiratory rate.
B. Whether or not air is moving well.
C. The respiratory depth.
D. The presence of any wheezes.

14. A newborn's ability to breathe is unique in what way?


A. A newborn's breathing rate is initially slower than that of an adult.
B. A newborn breathes only through the nose.
C. A newborn uses primarily intercostal muscles to breathe.
D. A newborn is incapable of breathing through the nose.

15. Which statement about children and cardiac arrest is correct?


A. Children generally have very strong cardiovascular systems; cardiac arrest is usually precipitated by
respiratory arrest and exhaustion.
B. Children's underdeveloped cardiovascular systems render them subject to cardiac arrest and other heart-
related problems.
C. CPR and defibrillation are generally ineffective for pediatric patients.
D. Cardiac arrest is the second leading killer of children after trauma.

Answer Key: Chapter 30 - Quiz


1=D, 2=A, 3=D, 4=C, 5=A, 6=B, 7=C, 8=A, 9=A, 10=D, 11=A, 12=C, 13=B, 14=B, 15=A

TEST

1. Fontanelles, which are openings in the skulls of newborns that allow brain expansion, close when the child is
at approximately what age
A. 18- 20 months of age
B. 3-4 months of age
C. 1 year of age
D. 3 years of age

2. A mother brings her 3-year-old son into the aid room. She says he has a low grade fever and seems to be having
slight difficulty breathing. Which of the statements by the mother would make you think the child may have
croup?
A. "There are times when he continually drools."
B. "Last night, he had a barking-like cough."
C. "He has a rash on his chest."
D. "When I turn the air conditioner on, he gets more short of breath."

CHAPTER 30
3. When assessing a 3-year-old child for possible injuries after the mother's boyfriend said the child fell down a
flight of stairs, which of the following findings would raise your suspicion that the child may be a victim of
physical abuse?
A. The child has a broken clavicle.
B. The child cries when you palpate his arm.
C. Circumferential bruising on is apparent on the child's arm.
D. Several bruises are located on the child's knees and shins.

4. An adolescent is defined as someone who is:


A. 13-18 years old
B. 9-18 years old
C. 10-16 years old
D. 8-16 years old

5. You are assessing a 2-1/2-year-old child who was involved in a minor car collision. She is currently alert and
oriented. While you are assessing her for possible injuries, which of the following actions would be considered
most appropriate?
A. Allow the child to hold a favorite toy during the assessment.
B. Assess the child starting at her head and moving to her feet.
C. Examine possible painful sites first.
D. Be stern with the child and provide strict instructions on what you expect from her.

6. You are immobilizing a 4-year-old boy on a long spine board. Which of the following actions would be
appropriate when performing this intervention?
A. Avoid applying the chest strap across the patient's thorax.
B. Secure the patient's chest and legs to the board after the head has been secured.
C. Place padding between the patient's shoulders and the spine board.
D. Place a pillow under the patient's neck to keep the airway open.

7. Children exhibit great bursts of energy followed by sudden profound fatigue because:
A. They need to take a nap
B. They have attention deficit disorder
C. They have fewer energy reserves than adults
D. They lose interest in their current activity

8. Which of the following statements regarding the treatment of a pediatric patient in a prehospital setting is
true?
A. If the primary assessment of a sick pediatric patient reveals no deficits to the ABCDs, the patient is stable
and will not deteriorate.
B. If a child looks sick and is not getting better with care, assuming that the child is getting worse.
C. You must determine the exact illness in order to provide the most appropriate treatment.
D. You will get early warning if the child is going into shock because a child's vital signs change early.

9. You are night skiing when you are notified that a child has been involved in a collision on the intermediate
trail. As you approach the scene you are thinking about your assessment and realize that the cold and low light
may make assessing the child's circulation difficult. In this situation, which of the following is least likely to be
effective?
A. Fingertip capillary refill

CHAPTER 30
B. The patient's lips
C. The sclera of the eyes
D. The palms of the hands

10. Which of the following conditions is not a possible cause of seizures in young children?
A. Epilepsy
B. Hypothermia
C. Diabetes
D. Fever

11. You are assessing a 2-week-old baby who is sick. Assessment reveals that he has a fever and difficulty
breathing. Which of the following additional assessment findings would be most concerning to you given the age
of this patient?
A. A pulse of 116 beats per minute
B. A respiratory rate of 30 breaths per minute
C. Continual crying
D. Nasal passages that are occluded by mucus

12. Which of the following behaviors would an OEC Technician recognize as uncharacteristic of a conscious and
stable 2-year-old boy who fell and hurt his hand?
A. He cries any time you touch him.
B. He becomes upset when you lift his shirt to assess his abdomen.
C. He does not tell you where the pain is when you ask.
D. He does not cry or protest when you take him from his mother to assess him.

13. You must assess the pupils of a 5-year-old boy who fell. Which of the following statements would be most
appropriate for you to make before performing the assessment?
A. "I need to test your visual acuity by shining a light in your eyes."
B. "Open your eyes so that I can look into them."
C. "I am going to look into your pupils with my light."
D. "I am going to use this light to look into your eyes."

14. You are treating a child who has a minor head laceration that is bleeding profusely. Which of the following
statements would show your partner that you understand bleeding in children?
A. "Children have great compensating mechanisms, so bleeding from the head isn't serious."
B. "This is a minor head laceration, so we don't need to worry."
C. "Children have a smaller blood volume than adults, so this rate of bleeding is serious."
D. "As long as the child's vital signs are stable, we don't have to worry."

15. An infant who is short of breath is alert and has adequate respirations at a rate of 54 per minute. His skin
color is pink but slightly cool to the touch. When you place a pediatric mask on his face, he becomes very upset
and begins to physically struggle to remove it. In this situation you would:
A. Allow the mother to hold the infant and then provide blow-by oxygen therapy.
B. Omit the oxygen for now and continue to assess the infant every 5 minutes.
C. Gently restrain the infant's hands so that he cannot remove the mask.
D. Secure the mask to the infant's face using tape.

CHAPTER 30
16. A 9-month-old boy has been stung on his tongue after sucking on a bottle that had a bee on the nipple. In
comparison to the same injury in an adult, why would an OEC Technician be more concerned over this child?
A. A child's tongue is proportionally larger, increasing the chances of airway occlusion from even minor
swelling.
B. Children tend to be highly allergic to bee stings, whereas the same allergy in adults is rare.
C. A child's airway has more blood vessels than an adult's, making bleeding more of a concern.
D. When injured, a child's mouth produces more saliva, making choking a major concern.

17. You are treating a child who has a minor head laceration that is bleeding profusely. Which of the following
statements would show your partner that you understand bleeding in children?
A. "Children have a smaller blood volume than adults, so this rate of bleeding is serious.”
B. "As long as the child's vital signs are stable, we don't have to worry.”
C. "Children have great compensating mechanisms, so bleeding from the head isn't serious.”
D. "This is a minor head laceration, so we don't need to worry."

18. You are called to assist a toddler who has just had a seizure that lasted about 60 seconds. His mom reports
that the child has a history of seizures. Your assessment reveals that he is now responding to painful stimuli, is
breathing normally, and has a normal radial pulse. Which of the following actions would you take?
A. Place him in a recovery position and continue to assess him.
B. Insert an oral airway.
C. Gently try to arouse him and make him more alert.
D. Apply oxygen via nonrebreather at 15 LPM.

19. A 4-year-old girl is sitting upright on her mother's lap with her chin thrust forward. She looks lethargic and is
drooling. Her airway is open and she appears to be breathing adequately. Her mother says she has had a fever and
is complaining of a sore throat. You are very concerned that she may be developing:
A. Epiglottitis
B. Strep throat
C. Pneumonia
D. Meningitis

20. The American Academy of Pediatrics recommends the use of the Pediatric Triangle to quickly determine if a
child is "sick or not sick." This method allows rescuers to quickly assess all of the following about the patient
except his/her:
A. Appearance
B. History of illness or injury
C. Breathing
D. Circulation

21. You are treating a 2 year old who fell down a flight of stairs. You are concerned about a head injury and know
that toddlers have a higher risk for traumatic brain injury than adults because
A. During rapid deceleration, a toddler's head is propelled forward first, before the body.
B. A toddler's fontanelles will not close until the child reaches preschool age.
C. A toddler's balance is not fully developed.
D. A toddler has a smaller head than an adult.

22. Which of the following statements is most appropriate concerning dealing with caregivers and children during
a medical emergency?

CHAPTER 30
A. "I try to include caregivers in all that I do with their child so that the child and the caregivers are more
comfortable.”
B. "I include caregivers in the care until I get the information I need; then I remove the child and continue the
assessment in the first-aid room.”
C. "It is best to separate caregivers from the child so that proper assessment and care can be given.”
D. "I tell caregivers that everything will be okay so that they are calm and I am better able to help their child."

23. The most common source of burns in children is:


A. A scalding-hot liquid
B. Chemicals
C. A cigarette lighter
D. A candle

24. An OEC Technician asks you why you should not overextend the airway when performing a head tilt-chin lift
maneuver on a pediatric patient. Which of the following replies would you make?
A. "The tongue of a pediatric patient is proportionally larger and blocks the airway when the neck is
hyperextended.”
B. "The cartilage of the trachea is very soft and can compress if the neck is extended too far.”
C. "The esophagus in a pediatric patient is very thick and will occlude the airway if the neck is
hyperextended.”
D. "The pediatric cervical spine is delicate and can be injured if the neck is hyperextended."

25. Cardiac arrest in children is most commonly caused by:


A. Birth defects
B. SIDS
C. Trauma
D. Respiratory failure

26. While following the Pediatric Triangle of assessment, you observe an infant and note that she is paradoxically
irritable. This sign is often indicative that the child is:
A. A victim of neglect
B. Hungry
C. Very ill
D. Not seriously ill

27. You are reviewing psychosocial development of a preschooler in your OEC class. You recognize that
"preschooler" includes children who are:
A. 6-8 years old
B. 2-3 years old
C. over 9 years of age
D. 3-5 years old

28. You have been called to care for an unattended 6-year-old girl who has vomited once and is complaining of
mild abdominal pain. When you are performing the physical assessment and obtaining a SAMPLE history, which
of the following approaches would be considered most appropriate?
A. Never involve the caregiver in the SAMPLE questions
B. Using "baby talk" when asking her questions about her pain
C. Allowing her to play with your stethoscope before listening to her lungs

CHAPTER 30
D. Standing above her and smiling while you ask questions

29. When seen in a child, the "tripod" and "sniffing" positions are usually signs of:
A. respiratory distress.
B. hypovolemia.
C. an infection of the brain.
D. a neck or spine injury.

30. You are assessing an infant who has been ill and has had a fever for the past two days. As you approach the
infant you note that she has grunting respirations. Based on your OEC training you determine that grunting is:
A. often normal in a child with a cold.
B. a symptom of severe respiratory disease.
C. a symptom of significant dehydration.
D. a soothing mechanism for a sick child.

31. You are assessing an infant who has been ill and has had a fever for the past two days. As you approach the
infant you note that she has grunting respirations. Based on your OEC training you determine that grunting is:
A. A symptom of significant dehydration.
B. A symptom of severe respiratory disease.
C. Often normal in a child with a cold.
D. A soothing mechanism for a sick child.

32. When seen in a child, the "tripod" and "sniffing" positions are usually signs of:
A. Respiratory distress
B. An infection of the brain
C. Hypovolemia
D. A neck or spine injury

33. Which of the following instructions would you give an OEC Technician who is preparing to assess a stable 9-
month-old boy with a rash?
A. "Start at the head and slowly work your way to the feet."
B. "Have the mother hold him as you do the assessment."
C. "Make sure that you do not undress the baby for the assessment."
D. "Keep the baby calm; do not touch him during the assessment."

34. You read in the newspaper about a 9-month-old child who died from sudden infant death syndrome (SIDS).
From your training, you know that this means that the infant:
A. Died unexpectedly and of an undetermined cause.
B. Had a congenital heart defect.
C. Had a history of sleep apnea.
D. Suffocated in his crib blankets.

35. You have been called to assist a panicked mother who is worried about her son. When you arrive, she tells you
that her 4-year-old son has been quiet all morning and napping while the rest of her children were skiing. While
napping he began to "shake all over" for about 30 seconds. The child is now resting quietly and has adequate
respirations. His radial pulse is strong, and his skin is very hot and moist to the touch. Based on this presentation
and information, you assume that the seizure occurred secondary to:
A. Altered mental status

CHAPTER 30
B. A fever
C. Hypoglycemia
D. Hypoxia

36. Which of the following conditions is not a possible cause of seizures in young children?
A. Epilepsy
B. Diabetes
C. Hypothermia
D. Fever

37. You are caring for a 2-year-old girl whose mother reports that the child has been ill for two days. It is
important to remember that children in this age group generally:
A. Will be cooperative during your exam
B. Prefer to be addressed through their caregivers
C. Are fearful of strangers
D. Readily tolerate separation from their caregivers

38. You have been called to care for a 21-month-old girl who has been bitten by a dog. Given the patient's age,
you would appropriately classify the patient as:
A. A school-age child
B. A toddler
C. An infant
D. A preschooler

39. When assessing a 3-year-old child with a respiratory illness, which of the following assessment findings
would be least concerning to you?
A. Paradoxical breathing
B. A respiratory rate of 28 breaths per minute
C. Patient in the tripod position
D. Retractions observed above the clavicles

40. The increased pliability of the ribs of children makes them more prone to:
A. Over inflation of the lungs
B. Bruising of the lung
C. Rib fractures
D. Cardiac arrest

41. Which of the following statements about shaken baby syndrome is false?
A. It rarely occurs in upper-class families.
B. It often results in a traumatic brain injury.
C. Most cases are perpetrated by someone close to the child.
D. It is a form of child abuse.

42. Bilateral injuries, circumferential bruising, and pattern bruises are suggestive of:
A. Shaken child syndrome.
B. Infant neglect
C. Child abuse
D. Suicidal behavior

CHAPTER 30
Answer Key: Chapter 30 - Test
1=A, 2=B, 3=C, 4=A, 5=A, 6=C, 7=C, 8=B, 9=A, 10=B, 11=D, 12=D, 13=D, 14=C, 15=A, 16=A, 17=A,
18=A, 19=A, 20=B, 21=B, 22=A, 23=A, 24=B, 25=D, 26=C, 27=D, 28=C, 29=A, 30=B, 31=B, 32=A, 33=B,
34=A, 35=B, 36=C, 37=C, 38=B, 39=B, 40=B, 41=A, 42=C


CHAPTER 30
C H A P T E R 31
“Geriatrics”

QUIZ

1. All but which of the following may make assessment and care of an elderly patient difficult?
A. Seniors often have a greater tolerance for pain.
B. Seniors may practice polypharmacy, which can affect physiological compensatory mechanisms and mask
the signs or symptoms of illness or shock.
C. Seniors often have increased curvature or other changes of their cervical and thoracic spine.
D. Seniors' knowledge or beliefs are generally outdated and their ability to process cognitive information
reduces their level of understanding and intelligence.

2. A 74-year-old skier falls hard, landing belly first on compact snow. He tells you he thinks he is fine; that his
'belly hurts a little,' but he thinks he can just ski that off. His pulse is 64, respirations 24, and BP is 116/80. He
tells you he takes Coumadin and a beta blocker. Your best course of action is to:
A. Maintain a high index of suspicion and urge him to seek immediate medical care for follow up and
assessment.
B. Tell him to check in with the patrol or his physician if the pain does not improve or if it gets worse.
C. Tell him you are glad he is OK and ski away.
D. Talk him into allowing you to arrange transportation for him to the nearest trauma facility.

3. A 68-year-old patient takes a channel blocker, warfarin, and Lopressor. What affect could these medications be
likely to have on this patient's pulse and blood pressure?
A. Hypertension and bradycardia
B. Normal blood pressure and bradycardia
C. Hypotension and tachycardia
D. None

4. Which statement concerning the physiology of a senior's body is true?


A. Physiological differences tend to make seniors frail or helpless.
B. It is basically no different than that of a younger individual; just older and somewhat 'worn out.'
C. It is inherently different than that of a younger individual.
D. Healthy seniors are physiologically identical to younger individuals; unhealthy seniors tend to be
physiologically different from younger individuals.

5. Which statement about seniors' neurologic, cardiovascular, and/or respiratory systems is true?
A. Aging causes normal gradational increases in blood pressure, so a 78-year old with a BP of 200/120 would
not be cause for immediate concern.
B. Although lungs lose their elasticity, the brain shrinks in size and the heart loses its elasticity, a senior's
vital lung capacity remains the same, and the ability of the nerves to transmit data and the heart's ability to
pump blood effectively remain unchanged throughout life.

C H A P T E R 31
C. The brain and peripheral nerves are less effective at transmitting data, the ventricular walls are stiffer and
less efficient at pumping blood, and the lungs are less elastic with less vital capacity.
D. Although there is a reduction in neurotransmitters in the brain, the size of the brain remains the same.

Answer Key: Chapter 31 - Quiz


1=D, 2=A, 3=B, 4=C, 5=C

TEST

1. An advance directive is a:
A. Court order directing prehospital providers to administer care.
B. Legal document that provides medical direction for life-saving efforts.
C. Protocol directed by a paramedic on scene.
D. Directive provided by medical control.

2. When communicating with a geriatric patient, OEC Technicians should:


A. Touch the patient as they explain the patient's medical issues and affected body areas.
B. Use anatomical pictures and point to the affected areas as they explain.
C. Use medical terms to describe their findings and treatment plan.
D. Use layman's terms.

3. Because many elderly patients will not offer their medical history unless specifically asked to do so, OEC
Technicians may have to:
A. Ask open-ended questions.
B. Look for past surgical scars during a secondary assessment.
C. Search a patient's medicine cabinet and kitchen cabinets for prescription bottles.
D. Ask yes-or-no questions.

4. Which of the following conditions must OEC Technicians consider when evaluating hypotensive geriatric
patients with altered mental status who show no signs of external bleeding, chest injury, or abdominal trauma?
A. A tib/fib fracture
B. Dehydration
C. A hip or pelvic fracture
D. A genetic hypotensive condition

5. To identify all the drugs, prescriptions, herbal supplements, and over-the-counter drugs that a patient may be
taking in combination, OEC Technicians should use:
A. DCAP-BTLS.
B. AEIOU.
C. SAMPLE.
D. OPQRST.

C H A P T E R 31
6. Compared to younger individuals, geriatric patients who fall have a higher incidence of fractures to the _____
region of the spine.
A. T1-T2
B. C1-C2
C. C4-C5
D. L1-L2

7. When taking the pulse of an elderly patient, it is recommended that OEC Technicians:
A. Divert the patient's attention with conversation.
B. Repeat and document the pulse rate every 2 minutes.
C. Use electronic devices only to obtain the pulse.
D. Take the radial pulse on both arms and compare the two rates.

8. The lung capacity in a healthy 90-year-old patient is ________ that of a 30-year-old patient.
A. Equal to
B. Half of
C. One-quarter of
D. Three-quarters of

9. You have been called to an unknown medical emergency. On scene you are presented with an 84-year-old
patient who is confused and does not obey commands. Which of the following action would be most useful in
determining the patient's normal mental status?
A. Question the patient's family members.
B. Check the patient's vital signs.
C. Examine the patient's pupils.
D. Identify the patient's medications.

10. Family members tell you that three hours ago their 76-year-old mother suddenly became confused and had
great difficulty speaking. However, within 15 minutes she returned to normal. Based on this description, an OEC
Technician should be suspicious of:
A. Dementia
B. A transient ischemic attack
C. A stroke
D. Alzheimer's disease

11. Over 60 percent of abdominal pain problems in geriatric patients require:


A. A prescription for a narcotic drug.
B. Evaluation for hernia.
C. Surgical intervention.
D. A trip to the emergency department.

12. Geriatric patients may also have a prescription for a diuretic such as HCTZ or Lasix. The role of this type of
drug is to:
A. Increase the preload of the atria.
B. Increase a patient's respiratory rate.
C. Increase a patient's heart rate.
D. Decrease the volume of fluid circulating in the cardiovascular system.

C H A P T E R 31
13. In geriatric patients, strokes can be caused by a blockage or by:
A. A rupture of a cerebral blood vessel.
B. Trauma to the cortex.
C. A buildup of epinephrine.
D. A buildup of dopamine.

14. Which of the following strategies is best for OEC Technicians for communicating with a geriatric patient who
has diminished eyesight?
A. Write your questions in large print on paper.
B. Increase both the pitch and the volume of your voice.
C. Obtain needed information from family members.
D. Stand in front of the patient and talk calmly.

15. Correcting a life-threatening condition such as external bleeding in elderly patients can be more problematic
because:
A. The skin of elderly patients does not rebound as quickly as it does in younger patients.
B. The veins of elderly patients have weaker walls than those in younger patients.
C. Many elder patients are taking warfarin or other blood thinners.
D. Elderly patients may be on Lasix.

16. Falls result in ____ percent of all deaths in the geriatric population.
A. 72
B. 12
C. 40
D. 5

17. One group of drugs prescribed for geriatric patients is beta-blockers. This type of drug:
A. Manages cardiac arrhythmias.
B. Increases a patient's pulse rate.
C. Thins a patient's blood.
D. Increases a patient's cardiac preload.

18. Some elderly patients have osteoporosis, a condition that results in:
A. Decreased bone density
B. Decreased insulin production
C. Decreased renal output
D. Gall stone production

19. Many elderly patients participate in a practice called "polypharmacy," which is the:
A. Use of designer drugs.
B. Use of mail-order pharmacies.
C. Simultaneous taking of multiple medications.
D. Use of generic drugs.

20. An elderly woman has fallen down three steps and is complaining of back pain. After you perform a primary
assessment, which of the following questions should you ask to illicit the next important piece of information?
A. "Did you become dizzy before you fell?"

C H A P T E R 31
B. "Do you have a history of high blood pressure?"
C. "Why didn't you use the ramp on the other side?"
D. "Who is your family doctor?"

21. An effective way to interview a geriatric patient is to:


A. Write your questions down on paper and show them to the patient.
B. Offer possible responses to assist the patient before she answers.
C. Keep repeating your question if the patient is slows to respond.
D. Use active listening skills.

22. By age 85, the brain of a typical geriatric patient can shrink by as much as 10 percent due to:
A. Reduced use in learning.
B. Reduced fluid volume.
C. Stroke activity.
D. A decrease in brain cell numbers.

23. Which of the following statements about traumatic injuries in the elderly is true?
A. Hyperthermia is an early sign of trauma.
B. Motor-vehicle collisions are responsible for over 75 percent of deaths.
C. Injuries are most commonly caused by falls.
D. Head injuries are easier to detect in the elderly than in younger adults.

24. You and your partner respond to the lodge to aid an elderly woman who is not feeling well. When you
perform a secondary assessment:
A. You and your partner should take turns asking the patient questions.
B. You and your partner should ignore the patient and ask the family for information.
C. Only one of you should ask the patient questions to avoid confusion.
D. You should stand to the patient's side and raise your voice so the patient can hear you.

25. For many geriatric patients, food is less appealing because:


A. They engage in less physical activity.
B. They have less money for purchasing appealing food.
C. Their sense of smell is reduced and they have fewer taste buds.
D. They sleep more and eat less.

26. A hearing-impaired patient wearing a hearing aid is having a great deal of difficulty hearing your questions.
Which of the following actions should you take first?
A. Ensure that their hearing aid is turned on.
B. Increase the pitch of your voice.
C. Write your questions on paper.
D. Position yourself in front of the patient and shout.

27. You have been called by the caregiver of a 91-year-old woman with dementia. The patient is complaining of a
possible fracture to her upper left arm. As you talk to the caregiver and note various bruises on the patient's body,
you become suspicious of elder abuse. Which of the following actions is most appropriate at this time?
A. Ask the patient if she is being abused.
B. Treat the patient for a possible broken arm.
C. Confront the caregiver regarding the bruises on the patient's body.

C H A P T E R 31
D. Contact the police to report your suspicion.

28. In a typical geriatric patient, a reduction in cerebral blood flow can:


A. Reduce the amount of glucose and oxygen that reaches the brain.
B. Increase the patient's pulse rate.
C. increase the patient's respiratory rate.
D. decrease the patient's blood pressure.

29. The son of an 88-year-old patient states that he told his mother to increase the amount of an antibiotic she
was taking so she would "feel better faster." This represents a problem because the elderly have:
A. Decreased liver and kidney function
B. Decreased respiratory function
C. An increased risk for cancer
D. An increased risk for stroke

30. You have been asked to deliver a talk about the geriatric population to the new OEC class. Which of the
following points would you emphasize in your presentation?
A. Dementia is an inevitable part of aging.
B. Many elderly individuals have a combination of different diseases in various stages.
C. Assessment can be difficult because most individuals older than 70 suffer from dementia.
D. Geriatric patients actually account for a small number of emergency medical calls and transports.

31. A geriatric patient's cardiovascular response to stress, illness, and injury is different than that of younger
patients. Which of the following statements is false?
A. As the heart ages, the ventricles become stiffer.
B. The heart slows due to decreased electrical conductivity in geriatric patients.
C. The volume of blood in the body decreases in geriatric patients.
D. Blood vessels become stiffer from atherosclerosis in geriatric patients.

32. Compared to younger patients, trauma in geriatric patients results in:


A. More fractures of the extremities.
B. Less-serious injuries due to the slower pace in older patients.
C. More bleeding.
D. A higher mortality rate.

33. Which of the following statements shows that an OEC Technician has an accurate understanding of vital signs
in relation to geriatric patients?
A. "The typical resting heart rate in geriatric patients is less than 60 beats per minute."
B. "Fever tends to be more common and more severe in elderly patients."
C. "There is no difference between the vital signs of a geriatric patient and those of a young adult."
D. "Respiratory capacity is greatly reduced in geriatric patients."

34. Which of the following statements concerning insulin pumps is false?


A. They have an external on/off button.
B. They are commonly found on a patient's belt.
C. They are implanted in the patient's chest.
D. They supply insulin via a catheter.

C H A P T E R 31
35. Which of the following conditions is associated with pulmonary embolism?
A. Pulmonary edema
B. Hypertension
C. Infection
D. Hypoxia

36. A new OEC Technician asks you why the elderly are at higher risk for developing pneumonia than younger
individuals. You inform him that the elderly are more susceptible to respiratory infections because of:
A. Impaired swallowing of saliva
B. Decreased alveolar gas exchange
C. Hardening of the major arteries
D. A reduced cough reflex

37. The class of medications that is most commonly prescribed to elderly patients is:
A. Respiratory medications.
B. Cardiovascular medications.
C. Gastrointestinal medications.
D. Neurological medications.

38. The vital signs of elderly patients with excessive internal or external bleeding may not provide an indication of
shock:
A. If these patients are taking beta-blockers or calcium-channel blockers.
B. If these patients are taking blood thinners.
C. Because the hearts of elderly patients pump less due to inactivity.
D. Because elderly patients have the ability to compensate for bleeding.

39. An advance directive is a:


A. directive provided by medical control.
B. legal document that provides medical direction for life-saving efforts.
C. court order directing prehospital providers to administer care.
D. protocol directed by a paramedic on scene.

Answer Key: Chapter 31 - Test


1=B, 2=D, 3=A, 4=C, 5=C, 6=B, 7=D, 8=B, 9=A, 10=B, 11=C, 12=D, 13=A, 14=D, 15=C, 16=B, 17=A,
18=A, 19=C, 20=A, 21=D, 22=D, 23=C, 24=C, 25=C, 26=A, 27=B, 28=A, 29=A, 30=B, 31=C, 32=D,
33=D, 34=C, 35=D, 36=D, 37=B, 38=A, 39=B

C H A P T E R 31
CHAPTER 32
“Outdoor Adaptive Athletes”

QUIZ

1. Autonomic dysreflexia can be caused by all but which of the following in a paraplegic skier?
A. Tight splints or straps
B. A full bladder or ostomy bag
C. The pain of a lower extremity fracture
D. Wet ski clothes

2. A patient with cognitive and verbal disabilities grows increasingly frustrated and angry during your assessment.
The most likely cause for this is that:
A. The patient is suffering from a head injury.
B. The patient does not like you.
C. The patient is in pain.
D. The patient is frustrated over her inability to communicate with you.

3. A patient suffering from ASD is most likely to die from which type of accident?
A. Thoracic trauma due to blunt impact collision
B. Drowning
C. A traumatic brain injury due to falling
D. A motor vehicle accident (MVA)

4. A paraplegic mono-skier has collided with a lift tower and suffered an obvious femur fracture. Which of the
following statements regarding this patient is true?
A. The patient will most likely be unaware of the injury.
B. The patient will be able to accurately gauge the level of pain on a 1-10 scale.
C. The OECT will be able to gauge CMS in the injured but not the uninjured leg.
D. The patient is at high risk of developing autonomic dysreflexia.

5. A patient with Down Syndrome falls hard while ice-skating. During your assessment of her she becomes angry
and combative. This is most likely due to:
A. The nature of Down syndrome patients.
B. The fact that she is uncomfortable with you and what you are doing.
C. A traumatic brain injury (TBI).
D. Pain.

6. Which of the following statements is NOT true?


A. Mental retardation is an acceptable term for individuals with intellectual disabilities.
B. The term disability is preferred over the term handicap.
C. A person with an impairment may not have a disability.

CHAPTER 32
D. An impairment is a loss or inability to perform a task in daily life.

Answer Key: Chapter 32 - Quiz


1=C, 2=D, 3=B, 4=D, 5=C, 6=A

TEST

1. You are caring for a hearing-impaired athlete and realize that in order to communicate, you:
A. can usually speak loudly enough to be understood.
B. should face the person because most hearing-impaired individuals can read lips.
C. will need paper and a pen because most hearing-impaired individuals cannot speak.
D. must know sign language or call for someone who does.

2. Its 2 pm on Saturday, and you're out for a bike ride with friends. About halfway down a small hill, you see an
adult and a child at the side of the road. It appears that the child may have fallen. As you approach, you ask if you
can help, to which you hear a mom's thankful "yes." The child, who is about 8 years old, is sitting quietly and does
not respond when you say "hi." The mother explains that her son has atypical autism. Based on this information,
which of the following techniques might you use to complete your assessment?
A. Give the child simple, one-step directions.
B. Speak only to the child.
C. Speak to the child loudly to get his attention.
D. Ask the child open-ended questions.

3. Spring is finally here, and your friend is helping you open up your summer cabin at the lake. At lunch that day,
your friend tells you that his son, who is down at the lakeshore playing with your children, was recently
diagnosed with autism. You are concerned about the child's autism because:
A. You do not want your children to be with children with autism.
B. Children with autism are known to be non-communicative, and you fear for your children.
C. You are concerned about how your children will react to the child's physical deformities.
D. Drowning is the leading cause of death for people with autism spectrum disorders.

4. Which of the following statements is not a consideration when assessing a patient with a cognitive disability?
A. Cognitive disabilities limit the person's ability to process information.
B. Patients with cognitive disabilities usually do not have problems with coordination.
C. Seizures can be associated with cognitive disabilities.
D. A cognitive disability can result from traumatic brain injuries.

5. When caring for a patient with dyslexia, it is important to remember that:


A. Many dyslexic people have spasticity.
B. You should not expect any impairment in communication.
C. The patient may have difficulty processing new information.
D. All individuals with dyslexia have below-normal intelligence.

CHAPTER 32
6. A 29-year-old woman with cerebral palsy has fallen and suffered a boot-top fracture of her right leg. When you
examine her she tells you that the spasticity in that leg is much worse than usual. Treatment in this situation may
best be accomplished by:
A. Stabilizing the leg with a splint in a position of comfort.
B. Repositioning the leg.
C. Applying traction to the leg.
D. Massaging the muscles in the leg.

7. A group of ski patrollers have invited you to join their team on a bike ride to raise money for multiple sclerosis
(MS) research. You recall learning about MS in your OEC class, but you ask your friend to remind you of some of
the key features of this condition. Which of the following statements made by your friend would be most
accurate?
A. "Speech and mood are generally not affected by MS."
B. "MS is a progressive disease that causes degeneration of both central and peripheral nerves."
C. "MS does not affect vision"
D. "MS does not affect mental capacity and memory."

8. One of the skiers in a group of adaptive athletes has spina bifida. If you needed to care for this patient, it is
important to remember that:
A. They may be allergic to natural latex, so contact with latex can result in anaphylactic shock.
B. They have also had a traumatic spinal cord injury.
C. Contrary to other spinal cord injuries, they will not have sensory deficits.
D. They may have a drain for lymph that is protected because it is inside the body.

9. A surgically created port used to drain the urinary system into the anterior abdomen or near the kidneys is
known as a(n):
A. urostomy.
B. ileostomy.
C. urethral catheter.
D. colostomy.

10. You are teaching a class on adaptive athletes and are asked to discuss Asperger's syndrome. Which of the
following statements is an appropriate response?
A. "Asperger's syndrome is a condition manifested by a standard group of symptoms, including physical
impairments.”
B. "Asperger's syndrome is an autism spectrum disorder in which the person may be fearful in unfamiliar
environments.”
C. "Individuals with Asperger's syndrome are less likely to come in contact with rescuers than are members of
the general population.”
D. "Asperger's syndrome implies that the person has no difficulty participating in normal day-to-day activities
but may have difficulty with long-range planning."

11. When caring for a patient with dyslexia, it is important to remember that:
A. the patient may have difficulty processing new information.
B. hyperactivity and impulsive behavior are common in individuals with dyslexia.
C. all individuals with dyslexia have below-normal intelligence.
D. you should not expect any impairment in communication.

CHAPTER 32
12. When caring for patients with life-long intellectual disabilities:
A. Provide care quickly because they will not know they are cold.
B. Use a stern, unemotional tone of voice because they may not pay attention to you.
C. Include their caretakers in the assessment because they can help with communication.
D. Talk to the patient as if they were at a sixth grade level.

13. You are assisting a 32-year-old woman whose right leg has been amputated below the knee. She is
complaining of severe pain in her right thigh. You note that the knee is flexed and that there is some deformity in
the middle of the femur. Your partner hands you a HARE traction splint. Which of the following statements
would be your best response to your partner?
A. "We need to force the knee to straighten and then we can use a splint."
B. "The HARE won't work. We will need to be creative and make our own traction device."
C. "Let's use a Quick Splint instead."
D. "We will need to adapt the boot hitch in order to secure it to the end of the leg."

14. The four major groups of adaptive alpine skiers are:


A. hearing-impaired skiers, tether skiers, sit-down skiers, and mono-skiers.
B. sit-down skiers, blind guided skiers, three-track skiers, and snow sliders.
C. sit-down skiers, three-track and four-track skiers, blind guided skiers, and two-track skiers.
D. hearing-impaired skiers, blind guided skiers, sledge hockey users, and outrigger skiers.

15. It is a cold, snowy Saturday afternoon, and unfortunately the chair lift has broken and a lift evacuation is
required. You know that there is an adaptive skier group somewhere on the lift. As you prioritize the lift
evacuation, you recognize that:
A. Skiers with intellectual disabilities should be left to the end because they will take a long time to
understand your directions.
B. Athletes with mobility problems are at increased risk of cold-exposure injuries.
C. A blind skier's guide should be evacuated before the blind skier so that the guide can direct the blind skier
down.
D. Someone will need to instruct paraplegic skiers on how to detach themselves from the sit-ski so they can
be lifted out for the evacuation.

16. You are caring for a hearing-impaired athlete and realize that in order to communicate, you:
A. Can usually speak loudly enough to be understood.
B. Will need paper and a pen because most hearing-impaired individuals cannot speak.
C. Must know sign language or call for someone who does.
D. Should face the person because most hearing-impaired individuals can read lips.

17. What is true about a patient with a cognitive disability?


A. Patients with cognitive disabilities usually have problems with coordination.
B. A cognitive disability can result from traumatic brain injuries.
C. Cognitive disabilities limit the person's ability to process information.
D. All of the above.

18. One of the skiers in a group of adaptive athletes has spina bifida. If you needed to care for this patient, it is
important to remember that:
A. contrary to other spinal cord injuries, they will not have sensory deficits.
B. they may be allergic to natural latex, so contact with latex can result in anaphylactic shock.

CHAPTER 32
C. they have had a traumatic spinal cord injury.
D. they may have a drain for lymph that is protected because it is inside the body.

19. When caring for patients with life-long intellectual disabilities:


A. recognize that disorientation is a sign of a head injury.
B. provide care quickly because they will not know they are cold.
C. include their caretakers in the assessment because they can help with communication.
D. use a stern, unemotional tone of voice because they may not pay attention to you.

20. You are assisting a 29-year-old spinal injury paraplegic skier who lost control of his sit ski and hit a tree. Even
though there are no obvious signs of injury, the patient suddenly starts to feel panicky, and you become concerned
about autonomic dysreflexia. Other signs/symptoms that would support this assessment include all of the
following except:
A. Hypotension
B. Blurred vision
C. Altered mental status
D. Headache

21. You are teaching a class on adaptive athletes and are asked to discuss Asperger's syndrome. Which of the
following statements is an appropriate response?
A. "Asperger's syndrome is an autism spectrum disorder in which the person may be fearful in unfamiliar
environments.”
B. "Asperger's syndrome implies that the person has no difficulty participating in normal day-to-day activities
but may have difficulty with long-range planning.”
C. "Asperger's syndrome is a condition manifested by a standard group of symptoms, including physical
impairments.”
D. "Individuals with Asperger's syndrome are less likely to come in contact with rescuers than are members of
the general population."

22. Cerebral palsy patients typically have some impairment of voluntary motor function and coordination. This
condition can present in one of three patterns. Which of the following statements does not describe one of those
patterns?
A. Slow, involuntary movements and contractions that can affect balance
B. Rigidity in one or more extremity
C. Involuntary muscle contraction in one or more extremities
D. Flaccid muscles in one or more extremity

Answer Key: Chapter 32 - Test


1=B, 2=A, 3=D, 4=B, 5=C, 6=A, 7=B, 8=A, 9=A, 10=B, 11=A, 12=C, 13=C, 14=C, 15=B, 16=D, 17=D,
18=B, 19=C, 20=A, 21=A, 22=D

CHAPTER 32
CHAPTER 33
“Behavioral Emergencies”

QUIZ

1. Which of the following statements about hallucinations is NOT true?


A. Auditory hallucinations are most commonly related to psychiatric disorders.
B. Hallucinations can lead an individual to engage in dangerous or homicidal behavior.
C. Hallucinations involve only two of the senses: hearing and sight.
D. Visual hallucinations are most commonly related to medical disorders.

2. Which of the following statements about agitation is true?


A. It can be stress or situationally induced.
B. Agitation includes a decreased level of sensitivity.
C. Agitation generally does not precede more aggressive behavior in patients with mental disorders but does
in patients undergoing situational stress.
D. Agitation is most often a symptom of a psychosis-based disorder such as schizophrenia.

3. A behavioral emergency is defined as:


A. Any incident of abnormal behavior.
B. Any form of altered mental status.
C. Behavior that constitutes a potential danger to the patient or others.
D. An incident of abnormal behavior not judged within the context of a given situation.

4. The two most common causes of altered mental state are:


A. Hypoxia and hypoglycemia.
B. TBI and alcohol overdose.
C. Fatigue and dehydration.
D. Stress and anger.

5. The mother of a missing 5-year-old becomes hysterical. She screams at you and berates you for not doing more
to find her child, then threatens you with everything from bodily harm to a lawsuit if you do not find her child.
This behavior is considered to be:
A. A sign of altered mental status with aggressive agitation.
B. A strong but understandable response, given the situation.
C. Cause for arrest due to the threat of bodily harm.
D. Paranoid delusional.

6. Normal versus abnormal behavior is generally defined by all of the following EXCEPT:
A. Regional or cultural norms.
B. Federal laws.
C. Local rules and regulations.

CHAPTER 33
D. A witness's observations.

Answer Key: Chapter 33 - Quiz


1=C, 2=A, 3=C, 4=A, 5=B, 6=B

TEST

1. Which of the following statements regarding the use of physical restraint is true?
A. Once you decide to restrain a patient, use the maximum force possible.
B. Law enforcement personnel can restrain anyone for any reason.
C. Physicians have unlimited latitude in ordering the physical restraint of a patient.
D. An OEC technician must follow local protocols for restraining patients.

2. Which of the following patients should an OEC Technician consider a potential behavioral emergency?
A. A 29-year-old man with diabetes who is combative and refusing care
B. A 36-year-old man with a known history who tells you he is hearing voices
C. A 56-year-old man with schizophrenia who has continued taking his medications
D. A 44-year-old woman with bipolar disorder who is complaining of a fever of 99.5 degrees °F

3. When mechanically restraining a patient, you should:


A. tell the patient you will release the restraints as soon as he calms down.
B. have six people available to safely apply the restraints.
C. designate two leaders: one to control the patient's upper extremities and one to control the patient's lower
extremities.
D. speak as harshly as necessary to get the patient to cooperate.

4. Two of the most common medical causes of abnormal behavior are:


A. Carbon monoxide poisoning and acute myocardial infarction
B. Hypotension and hyperthermia
C. Exercise and hypoglycemia
D. Hypoxia and hypoglycemia

5. A condition in which a person exhibits abnormal behavior but is able to function within the normal boundaries
of reality is known as:
A. A psychosis
B. Schizophrenia
C. A behavioral emergency
D. A neurosis

6. Your OEC class asks you to describe the term behavior. Which of the following statements would be the correct
response?

CHAPTER 33
A. Behavior pertains to how you perform a physical activity only.
B. Meditation is the absence of behavior.
C. Yelling is never an acceptable behavior.
D. What is acceptable behavior in one community may not be acceptable in another.

7. Which of the following patients is exhibiting signs of psychosis?


A. An 18-year-old who is so anxious about peer pressure that he is unable to go to school
B. A 37-year-old who is having hallucinations
C. A recently divorced 35-year-old who sleeps most of the day and is losing weight
D. A 72-year-old who was recently diagnosed with dementia

8. Which of the following pieces of information is most important to OEC Technicians in assessing a patient with
abnormal behavior?
A. What the patient's diagnosis is
B. Whether or not the patient is dangerous
C. Whether the patient is anxious or depressed
D. What medications the patient takes

9. You and a fellow OEC Technician are discussing how to talk with someone experiencing a behavioral
emergency. Which of the following statements indicates that you have a good understanding of the correct
approach?
A. "If the patient is hallucinating, you should go along with them so they won't get upset."
B. "Demonstrate active listening skills by repeating or paraphrasing what the patient said."
C. "If a patient yells, you should yell back so he will know you are not afraid."
D. "Don't encourage patients to talk about what is bothering them. They should only talk with a psychiatrist
about that."

10. Which of the following situations is an indication for the use of restraints?
A. The patient responds to the use of verbal calming skills
B. The patient is in imminent danger of harming himself.
C. The patient's friend is requesting restraints
D. The patient is alert, rational, and refuses medical care.

11. A condition in which a person exhibits abnormal behavior but is able to function within the normal
boundaries of reality is known as:
A. a psychosis.
B. a neurosis.
C. schizophrenia.
D. a behavioral emergency.

12. You are describing techniques for remaining safe when responding to a call for a patient with a behavioral
emergency. Which of the following statements is appropriate?
A. You should be sure that you have at least one exit route, and do not let the patient get between you and
that way out.”
B. "You should approach patients quietly and try to surprise them so they don't have time to develop a plan to
attack you.”
C. "If a patient raises his/her voice, you should raise yours to match it.”
D. "You need to encounter the patient first, and then come up with your safety plan."

CHAPTER 33
13. Which of the following actions is best to keep a behavioral patient calm and to avoid provoking a violent
outburst?
A. Inform the patient that he is responsible for all of his actions, and that if he hurts you, you will sue him.
B. Establish that you are in control of the situation and that the patient must cooperate or you will restrain
him.
C. Involve as many people as possible in the situation as a "show of force.”
D. Inform the patient of everything that is happening and use a calm voice throughout care.

14. When treating a patient exhibiting abnormal behavior, you should:


A. restrain the patient as a precaution
B. Assume until proven otherwise that the source of the abnormal behavior is an underlying medical
condition.
C. Assume that the abnormal behavior is a manifestation of an underlying psychological problem.
D. Assist the patient in taking glucose in case the patient is hyperglycemic.

15. Which of the following statements shows that an OEC Technician has a good understanding of assessing a
patient who is having a behavioral emergency?
A. "Try to assess patients from a safe distance until they start to answer your questions calmly."
B. "If a patient raises his voice to you, you must raise your voice in return. Research has shown that this helps
deter violence.”
C. "Touch conveys concern for a patient, so I try to touch the patient's shoulder or arm as soon as I arrive at
the scene.”
D. "I avoid making eye contact because patients may perceive eye contact as a threatening gesture and respond
with violence."

16. Your first priority in managing a patient with a behavioral emergency is:
A. Determining if the patient is a threat to himself or herself.
B. Assessing and managing the patient's airway.
C. Ensuring your own personal safety.
D. Providing the patient high-concentration oxygen.

17. Which of the following behaviors is a sign of potential aggression in a patient?


A. Increased sighing
B. The clenching of both fists
C. Avoiding eye contact
D. Refusal to answer some questions

18. You are called to the lodge to assist a 25-year-old man who reportedly is having hallucinations. When you
arrive, he is sitting quietly and appears calm. Information obtained in your SAMPLE history indicates that the
patient started having hallucinations yesterday. He was diagnosed with schizophrenia a few years ago and has
been prescribed two different medications. Which of the following questions is most pertinent to the situation
and should be asked next?
A. "Do you believe that the hallucinations are real?"
B. "Have you been eating and drinking adequately?"
C. "Has your doctor considered increasing the dose of your medication?"
D. "Have you been taking your medications as prescribed?"

CHAPTER 33
19. When mechanically restraining a patient, you should:
A. use a prone position when possible to ensure an open airway
B. Speak as harshly as necessary to get the patient to cooperate.
C. use a method consistent with your local protocols
D. Tell the patient you will release the restraints as soon as he calms down.

20. You have stopped a male skier and a female skier who were racing down a closed trail. Initially, the man was
calm, but suddenly he becomes angered and says that "you can't tell me what to do." When you try to talk with
him, he pulls out a knife and tells you to "get away or I'll kill you." The female skier is frightened and states that
her friend sometimes gets very aggressive when he drinks. You put in a call for security and additional staff, but
you know it will be a few minutes before they arrive. Which of the following statements describes your best
course of action in the interim?
A. Move quickly to pull the woman a safe distance away from the man.
B. Immediately attempt to physically restrain the man or take away his knife.
C. Remain a safe distance from the man and let him know you are there to help.
D. Approach the man slowly and calmly and ask him to hand you the knife.

21. Which of the following statements about behavioral emergencies is True?


A. The person's mental status is not altered.
B. The person acts in a way that is not tolerable to those around them.
C. The person is not a danger to themselves or others.
D. The context of the situation is not relevant.

22. You are out with a group of friends in a local tavern. One of the patrons gets argumentative and will not calm
down. By the time the police arrive, the patron has gotten combative and is breaking glass bottles. The police
subdue the patron and have him restrained. As you watch all of this happening, which of the following situations
would cause you to be concerned?
A. The police say they will not release the patron until he has been seen in an emergency department.
B. The patron is supine on the floor, with officers holding one of his hands over his head and the other at his
side.
C. The police have restrained the patron in a prone position on the floor.
D. The patron is threatening to sue the officers for physically restraining him.

Answer Key: Chapter 33 - Test


1=D, 2=A, 3=B, 4=D, 5=D, 6=D, 7=B, 8=B, 9=B, 10=B, 11=B, 12=A, 13=D, 14=B, 15=A, 16=C, 17=B,
18=D, 19=C, 20=C, 21=B, 22=C

CHAPTER 33
CHAPTER 34
“OB/GYN Emergencies”

QUIZ

1. Which statement best describes gynecologic emergencies?


A. They are uncommon and often life threatening
B. They are fairly common but usually not life threatening
C. They are almost always difficult for an OECT to deal with
D. They almost always cause extreme embarrassment to both rescuer and patient

2. A laboring woman tells you she needs to "go to the bathroom and poop." Your best response to her should be
to:
A. Reassure her that the first phase of labor often feels like an urge to defecate.
B. Show her where the restroom is and accompany her to it.
C. Check her for crowning.
D. Place her on a bedpan.

3. A woman in her third trimester of pregnancy has skidded down two steps and landed on a carpeted floor. She
denies injury. She tells you that prior to pregnancy her normal pulse rate was 70. You take it and find it to be 100.
This is most likely due to:
A. Possible fetal demise.
B. Impending shock.
C. Embarrassment over having fallen.
D. A normal third-trimester pulse increase.

4. A patient in her 5th month of pregnancy falls while skiing. In which position should you place her?
A. Trendelenburg
B. Recovery
C. Left lateral recumbant
D. Fowler's

5. A 19-year-old sexually active female presents with severe lower left abdominal pain. She tells you she is
sexually active and that she does not believe she is pregnant, but her period is 2 weeks late. This patient could be
suffering from what possible gynecological problem?
A. An ectopic pregnancy
B. A bladder infection
C. Appendicitis
D. Pelvic inflammatory disease

CHAPTER 34
Answer Key: Chapter 34 - Quiz
1=B, 2=C, 3=D, 4=C, 5=A

TEST

1. At what point during a delivery should you first suction the baby?
A. When the baby's chest has been delivered but before the legs are free of the vaginal canal
B. When the baby has been delivered and has been dried and warmed
C. Before the cord has been cut but after the baby has been assessed and determined to be stable and if there
is obvious obstruction to spontaneous breathing
D. After the baby's head has been delivered but before the torso is out of the vaginal opening and if there is
obvious obstruction to spontaneous breathing

2. After delivery, you note that blood continues to ooze from the mother's vaginal canal. The sanitary pads that
you placed between her legs continue to become soaked. Which of the following actions should you take next?
A. Insert sterile gauze into the vaginal canal.
B. Perform a uterine abdominal massage.
C. Place the mother on her left side.
D. Call for an ALS ambulance.

3. Which of the following findings should be of greatest concern when assessing a 33-year-old woman who is 8
months pregnant?
A. Daily contractions that are irregular and painful
B. Recent onset of a fever of 99.3 degrees °F
C. Vaginal bleeding that is not associated with pain
D. Shortness of breath when lying flat

4. In an ectopic pregnancy, the primary threat to the pregnant woman is:


A. Damage to pelvic organs
B. Hemorrhage
C. Severe pain
D. Infection

5. A 27-year-old woman is complaining of midline lower abdominal pain that worsens with urination and is
associated with blood in her urine. One possible cause of her symptoms would be:
A. dysmenorrhea.
B. pelvic inflammatory disease.
C. an ovarian cyst.
D. cystitis.

6. As soon as a baby is delivered from the vaginal canal, it is crucial that you:
A. vigorously dry and warm the newborn.
B. obtain a complete set of baseline vital signs.
C. clamp and cut the umbilical cord.
D. perform an APGAR assessment.

CHAPTER 34
7. When the placenta attaches over the cervix, the baby and the mother are at risk for which of the following
conditions?
A. Placenta previa
B. A uterine tear
C. An ectopic pregnancy
D. Placenta abruptio

8. The function of the uterus in pregnancy is to:


A. Provide an environment for the ovum to develop into a fetus.
B. Permit the excretion of wastes from the baby to the mother.
C. Protect the abdominal organs during fetal growth.
D. Manufacture specific blood cells needed for fetal survival.

9. A woman who is 36 weeks pregnant has fallen and is complaining of sharp abdominal pain. She is also
experiencing vaginal spotting. She has tenderness on palpation just right of the midline of her abdomen. Her
pulse is 122, respirations are 22, and blood pressure is 118/82 mmHg. Based on this presentation, which of the
following actions would you take?
A. Administer high-flow oxygen and immediately transport her.
B. Provide low-concentration oxygen and transport her.
C. Massage her abdomen, provide high-flow oxygen, and nonemergently transport her.
D. Place her in a supine position and immediately transport her.

10. The underlying pathophysiology associated with an ectopic pregnancy is that a fertilized egg is:
A. developing without a placenta.
B. lodged in a fallopian tube.
C. contaminated with abdominal bacteria.
D. attached to the ovary.

11. When assessing a pregnant woman it is important to remember that:


A. maternal heart rate is usually normal in the third trimester.
B. systolic and diastolic blood pressures are normally high in the third trimester.
C. the uterus will be expanded and may be as high as the diaphragm.
D. due to physiologic changes in pregnancy, women may lose only 15 percent of their blood volume before
they go into shock.

12. The APGAR scoring system is useful in determining:


A. the maturity of a newborn's heart and lungs.
B. a fetus's precise age of gestation.
C. the status of a newborn's overall condition.
D. the type of resuscitation a newborn needs.

13. Which of the following instructions describes the correct way to suction a newborn?
A. Suction the mouth first and then the nose.
B. Don't use suction until the baby has been delivered.
C. Insert the bulb syringe deep into the oropharynx to remove any thick secretions located there.
D. Use a portable suction device on its "high" setting.

14. Which of the following statements concerning the functions of the vagina is true?

CHAPTER 34
A. The vagina serves as a passageway for the fetus during delivery and an outlet for blood during
menstruation.
B. The vagina permits the outflow of menstrual blood and is the site at which a sperm fertilizes an egg.
C. The vagina allows the excretion of urine from the bladder and the outflow of menstrual blood.
D. The vagina allows the passage of a baby during delivery and the outflow of urine from the bladder.

15. In abruption placenta, the greatest threat to the baby is:


A. Maternal shock
B. Hypoxia
C. Infection
D. Pain

16. Which of the following structures is not located within the pelvic cavity?
A. The ovaries
B. The fallopian tubes
C. The appendix
D. The uterus

17. The most common cause of fetal death due to trauma to a pregnant woman's abdomen is:
A. a crushing injury to the fetus.
B. penetration of the uterus.
C. premature labor.
D. abruptio placenta

18. You are in the process of delivering a baby and have just delivered the baby's head. Which of the following
actions should you take next?
A. Check the baby's neck for the presence of the umbilical cord.
B. Suction the baby's mouth, and then the nose.
C. Gently pull the baby toward you.
D. Turn the baby clockwise and continue the delivery.

19. Hypotension can occur after 20 weeks of gestation when a pregnant woman is in a supine position. Which of
the following actions by a OEC Technician will not prevent hypotension?
A. Elevating her right hip approximately 1-2 inches
B. Elevating her feet 6-12 inches
C. Placing her in the left lateral recumbent position
D. Tipping the spine board so that she is slightly tilted onto her left side

20. A newborn's first APGAR score is 6; his second APGAR score is 9. This information:
A. indicates an improvement in the newborn's condition.
B. indicates a need for extensive resuscitation of the newborn.
C. indicates poor heart and lung function in the newborn.
D. provides a total score of 15.

21. When the placenta attaches over the cervix, the baby and the mother are at risk for which of the following
conditions?
A. Placenta previa
B. Infection

CHAPTER 34
C. A uterine tear
D. An ectopic pregnancy

22. Which of the following statements concerning the functions of the vagina is true?
A. The vagina allows the excretion of urine from the bladder and the outflow of menstrual blood.
B. The vagina permits the outflow of menstrual blood and is the site at which a sperm fertilizes an egg.
C. The vagina serves as a passageway for the fetus during delivery and an outlet for blood during
menstruation.
D. The vagina allows the passage of a baby during delivery and the outflow of urine from the bladder.

23. The second stage of labor ends with:
A. Rupture of the amniotic sac
B. Contractions that become regular and occur every 3-4 minutes
C. Delivery of the baby
D. Delivery of the placenta

24. A 27-year-old woman is complaining of midline lower abdominal pain that worsens with urination and is
associated with blood in her urine. The most common cause of her symptoms would be:
A. Pelvic inflammatory disease
B. An ovarian cyst
C. Bladder infection
D. Dysmenorrhea

25. In an ectopic pregnancy, the primary threat to the pregnant woman is:
A. severe pain.
B. damage to pelvic organs.
C. infection.
D. hemorrhage.

26. A 24-year-old woman complains of sharp pain in her left lower abdominal quadrant. She states that her last
period was 7 weeks ago. Which of the following conditions would you most suspect?
A. Ectopic pregnancy
B. Abruptio placenta
C. Appendicitis
D. Colitis

27. In abruptio placenta, the greatest threat to the baby is:


A. maternal shock.
B. pain.
C. hypoxia.
D. infection.

28. The components of the APGAR scoring system include:


A. respiration, breathing, and skin moisture.
B. airway, breathing, and weight at birth.
C. airway, breathing, and time of birth.
D. appearance, irritability, and activity.

CHAPTER 34
29. The underlying pathophysiology associated with an ectopic pregnancy is that a fertilized egg:
A. Contaminated with abdominal bacteria
B. Develops without a placenta
C. Implants in the top part of the uterus
D. Implants in a fallopian tube

30. When should an OEC Technician cut the umbilical cord?


A. Immediately after the delivery of the placenta
B. Just before the passage of the legs through the vaginal canal
C. After the delivery of the baby whenever transportation will be delayed more than 20 minutes
D. Immediately following the delivery of the baby

31. Which of the following findings should be of greatest concern when assessing a 33-year-old woman who is 8
months pregnant?
A. Vaginal bleeding that is not associated with pain
B. Recent onset of a fever of 99.3 degrees °F
C. Daily contractions that are irregular and painful
D. Shortness of breath when lying flat

32. During a delivery, you note that the umbilical cord is wrapped around the baby's neck. Which of the following
actions should you take immediately?
A. Slip the cord over the baby's head.
B. Try to push the baby's head back in so you can slip the cord over it.
C. Stop the delivery and call for an ALS ambulance.
D. Clamp and cut the cord.

33. Hypotension can occur after 20 weeks of gestation when a pregnant woman is in a supine position. Which of
the following actions by an OEC Technician will not prevent hypotension?
A. Elevating her feet 6-12 inches
B. Placing her in the left lateral recumbent position
C. Tipping the spine board so that she is slightly tilted onto her left side
D. Elevating her right hip approximately 1-2 inches

34. At what point during a delivery should you consider using suction to clear the airway of the baby?
A. When the baby's chest has been delivered but before the legs are free of the vaginal canal
B. Before the cord has been cut but after the baby has been assessed and determined to be stable and if there
is obvious obstruction to spontaneous breathing
C. When the baby has been delivered and has been dried and warmed
D. After the baby's head has been delivered but before the torso is out of the vaginal opening and if there is
obvious obstruction to spontaneous breathing

35. Which of the following statements describes the proper care for the cord after delivery?
A. The cord is appropriately clamped and then cut approximately 6 inches from the infant's abdomen.
B. The cord is clamped but is not cut until the arrival of the baby at a hospital.
C. The clamps are removed after the cord has been cut.
D. The cord is clamped and cut one inch from the baby's abdomen.

36. Which of the following statements about the umbilical cord is true?

CHAPTER 34
A. It transmits nourishment from the placenta to the fetus.
B. It connects the placenta and the mother's uterus.
C. It produces hormones for the baby's body.
D. It detoxifies blood reaching the baby before delivery.

37. A 24-year-old woman complains of sharp pain in her left lower abdominal quadrant. She states that her last
period was 7 weeks ago. Which of the following conditions would you most suspect?
A. Ectopic pregnancy
B. Appendicitis
C. Abruptio placenta
D. Colitis

38. Which of the following statements about the umbilical cord is true?
A. It detoxifies blood reaching the baby before delivery.
B. It transmits nourishment from the placenta to the fetus.
C. It always connects to the top side of the uterus.
D. It connects the placenta and the mother's uterus.

39. Which of the following statements about the role of the cervix in Imminent Childbirth is true?
A. The cervix constricts during the time of delivery to prevent a premature birth.
B. As delivery starts, the cervix dilates giving a "Bloody show" (mucous plug).
C. The cervix can be palpated to determine the frequency of the mother's contractions.
D. The placenta is normally positioned directly over the cervix.

40. The most common cause of fetal death due to trauma to a pregnant woman's abdomen is:
A. Abruptio placenta
B. A crushing injury to the fetus
C. Premature labor
D. Penetration of the uterus

41. Which of the following statements about the placenta is accurate?


A. The placenta is full of bacteria and should not be transported near the baby in the ambulance.
B. The placenta is delivered by applying gentle pressure to the abdomen after the baby has been born.
C. It is essential that the mother not be moved until the placenta has been delivered.
D. The placenta typically delivers itself within 30 minutes of the birth of the baby.

Answer Key: Chapter 34 - Test


1=D, 2=B, 3=C, 4=B, 5=D, 6=A, 7=A, 8=A, 9=A, 10=B, 11=C, 12=C, 13=A, 14=A, 15=B, 16=C, 17=D,
18=A, 19=B, 20=A, 21=A, 22=C, 23=C, 24=C, 25=D, 26=A, 27=C, 28=D, 29=D, 30=C, 31=A, 32=A,
33=A, 34=D, 35=A, 36=A, 37=A, 38=B, 39=B, 40=A, 41=D


CHAPTER 34
CHAPTER 35
“Special Operations”

QUIZ

1. The acronym CBRNE refers to:


A. The substance or substances involved in an act of terrorism.
B. A series of safety checks ambulance drivers should run through before leaving the station.
C. The type of chemical or substance carried by a transport vehicle.
D. An assessment tool to determine the type of care needed for an ALS patient.

2. Which statement about safe ambulance operations is true?


A. The majority of motor vehicle accidents (MVAs) involving ambulances occur at night or during inclement
weather.
B. The majority of motor vehicle accidents involving ambulances occur during daylight hours and in dry
conditions.
C. Ambulance operators may determine whether or not they can exceed posted speed limits.
D. Statistics show that ambulances and other emergency vehicles rarely, if ever, are involved in MVAs.

3. The "hot," "warm" and "cold" zones are best defined as:
A. The most dangerous or contaminated area at an incident, the transition zone, and the zone in which it is
safe to work without specialized protective equipment.
B. The area of a structural or wildland fire where the actual burn is; the area which has burned, been
extinguished, and is being examined for possible re-ignition points; and the area just outside of the
original burn.
C. The red, yellow, and green treatment and staging areas at a multi-casualty incident, respectively.
D. The inside of a burning structure, the peripheral walls of a burning structure, and the yard outside of the
burning structure.

4. Which level of HAZWOPER training and certification allows the provider to function in any of the hazard
zones?
A. Awareness
B. Operations
C. Commander
D. Technician

5. During a search operation, one of the search dogs comes over to sniff you. Your response should be to:
A. Point and say, "Over there, boy-go-search over there!" and indicate to the direction in which the victim is
believed to be.
B. Ask the handler the dog's name and whether or not you may pat the dog.
C. Ignore the dog and remain focused on your job.
D. Praise and pat the dog.

CHAPTER 35
6. Which comment about rescuer safety in an avalanche search zone is correct?
A. Rescue teams are generally safe in slide zones as long as rescue teams remain small, individual rescuers
stay at least 15-20 feet apart, and all rescuers are wearing transceivers and carrying shovels.
B. Because the area has already slid, rescuers are probably safe from a further slide.
C. The risk of an additional slide is a very real and dangerous possibility.
D. Rescuers should never enter an avalanche slide zone until all remaining snow has been blasted down by
the blast team.

7. What do the mnemonics DUMBELS and SLUDGE refer to?


A. Untrained personnel who interfere with the work of a hazardous operations crew
B. The symptoms of organophosphate poisoning
C. The contents of a DuoDote Kit
D. Contaminated waste removed from a hazardous site that still poses a threat to rescuers

8. When asked to assist an ambulance crew with patient care, an OECT should:
A. Check with a supervisor before proceeding.
B. Only provide care up to the point of loading the patient into an ambulance.
C. Politely decline because this would be out of the scope of care of an OECT.
D. Be prepared to provide the full range of his or her OEC skills and assist in advanced life support care as
authorized and trained.

Answer Key: Chapter 35 - Quiz


1=A, 2=B, 3=A, 4=D, 5=C, 6=C,7=B, 8=D

TEST

1. Which of the following statements regarding the NFPA safety diamond is true?
A. The NFPA safety diamond identifies the known safety hazards of a given material.
B. The number of NFPA safety diamonds on a vehicle indicates the toxicity of the hazardous material.
C. The NFPA safety diamond tells you whether the fuel in a fire is paper, liquid, gas, or wood.
D. The NFPA safety diamond must be displayed on any building or vehicle that contains hazardous materials.

2. Which of the following statements concerning locating and extricating a victim buried by an avalanche is
correct?
A. Spinal injuries are presumed until proven otherwise.
B. Once the victim's face is clear, the patient's back should be dug out so a backboard can be inserted.
C. There should be no danger of another avalanche because the snow has now settled.
D. Survival rates drop to 50 percent after the first 1.5 hours.

3. Which of the following statements regarding an ambulance parked at an incident is true?


A. Ambulance warning lights should be turned off so they will not blind oncoming drivers.

CHAPTER 35
B. If a law enforcement vehicle is on the scene, the ambulance should be parked right beside it.
C. When walking from the scene to the parked ambulance, you should walk on the side of the road closest to
traffic so you can be more easily seen.
D. The area around the ambulance and the accident scene is known as the ambulance operation zone.

4. In a HazMat incident, a boundary that is set up to prevent members of the public from entering the incident is
called a(n):
A. Decontamination line.
B. Incident perimeter.
C. Isolation perimeter.
D. Hazard boundary line.

5. Organized under the Office of the Surgeon General, community-based volunteer medical professionals who
have agreed to help in the event of a disaster are known as the:
A. Disaster Medical Assistance Team.
B. Emergency Support Annexes.
C. National Response Framework.
D. Medical Reserve Corps.

6. Toxic nerve agents:


A. Block stimulation of sensory nerve endings only.
B. Block stimulation of both sensory and motor nerve endings.
C. Decrease production of chemical transmitters.
D. Overstimulate the nerve endings and the central nervous system.

7. The contaminated area in a hazardous incident is known as the:


A. Cold zone.
B. Hot zone.
C. Decontamination zone.
D. Contaminated zone.

8. Search and rescue operations consist of which of the following five basic tasks?
A. Incident Commander assigned, team leaders and teams assigned, grids determined, search & rescue, and
reporting
B. Notification of rescue personnel, perimeter marking, grid assignments, search & rescue, and reporting
C. Information gathering, grid assignments, search, medical treatment, and evacuation
D. Mobilization, intelligence gathering, containment, search, and rescue

9. Which of the following lists correctly places the tasks of fire ground operations in order of priority?
A. Accountability, rescue/life safety, confinement, control, size-up, property conservation, ventilation
B. Size-up, confinement, control, ventilation, rescue/life safety, accountability, property conservation
C. Size-up, accountability, rescue/life safety, confinement, control, ventilation, property conservation
D. Confinement, size-up, control, property conservation, rescue/life safety, accountability, ventilation

10. Which of the following tasks is not a task an OEC Technician should be ready to perform in assisting an EMT
in the back of an ambulance?
A. Medication administration
B. Putting a cervical collar on a patient

CHAPTER 35
C. Hemorrhage control
D. Splinting

11. You are instructing an OEC class on the handling of a HazMat incident. You are asked where OEC Technicians
would be assigned if they were assisting at a HazMat scene. Which of the following statements is the appropriate
response?
A. "Because of your training in this class, you could be assigned to any of the hazard control zones."
B. "Because of your OEC training, you would probably be assigned to the cold zone."
C. "OEC Technicians do not have the skills required to assist in a HazMat incident."
D. "OEC Technicians could assist inside of the isolation perimeter."

12. Which of the following activities is not considered a special operation?


A. A low-angle rescue
B. Toboggan transportation
C. A vehicle extrication
D. A lift evacuation

13. In a HazMat incident, the transition area in which decontamination occurs is known as the:
A. Warm zone
B. Transport zone
C. Cold zone
D. Treatment area

14. Special operations is a term used to denote:


A. A specialized procedure that is used for victims a of chemical terrorism event.
B. Protocols OEC Technicians use when working with an ambulance service in an urban setting.
C. Infrequently performed activities that require specialized training, skills, and equipment in a remote
setting.
D. A federal team that is organized to handle mass casualty situations involving mountainous terrain.

15. You overhear a group of EMTs discussing the concept of driving an ambulance with due regard for the safety
of others. Which of the following statements indicates that the EMT understands the concept?
A. "It is acceptable to take a nonemergency patient to the hospital using lights and siren if dispatch informs
me that there is another call waiting.
B. "When taking a patient to the hospital, I drive as quickly as possible so that I can get back into service for
another call.”
C. "When responding to an emergency, I drive as fast as possible so that emergency care can be delivered as
quickly as possible.”
D. "When approaching an intersection, I wait for all traffic to stop before proceeding through."

16. A low-angle rescue is any rescue work that:


A. mainly involves crevasse rescues in mountaineering and climbing.
B. takes place on a slope ranging from horizontal to 45 degrees.
C. takes place on a slope that is approximately 45 degrees or more in either an ascending or a descending
direction.
D. is utilized only when a patient must be lowered out of an incident zone.

17. Which of the following lists correctly places the tasks of fire ground operations in order of priority?

CHAPTER 35
A. Accountability, rescue/life safety, confinement, control, size-up, property conservation, ventilation
B. Size-up, confinement, control, ventilation, rescue/life safety, accountability, property conservation
C. Confinement, size-up, control, property conservation, rescue/life safety, accountability, ventilation
D. Size-up, accountability, rescue/life safety, confinement, control, ventilation, property conservation

18. You and your friend arrive at an accident scene where a compact car has rolled down an embankment. Based
on your OEC training, you realize that before anyone enters the vehicle, it should be stabilized. Vehicle
stabilization includes all of the following actions except:
A. Disconnecting the battery cables.
B. Having several men hold the vehicle to prevent it from moving.
C. Shutting off the engine.
D. Engaging the parking brake.

19. Which of the following organizations is not a disaster response agency that OEC Technicians are encouraged
to join?
A. The Medical Reserve Corps
B. The Civilian Emergency Response Team
C. The Incident Command Resource System
D. The Disaster Medical Assistance Team

20. Logistics, one of the four basic functions of an organized avalanche rescue, is primarily concerned with:
A. Getting the rescuers to the scene
B. Keeping a log of the search's activities
C. Arranging for extrication of the victims
D. Caring for the rescuers in the field

21. Which of the following statements concerning HAZWOPER training is false?


A. All EMS personnel are trained in all aspects of a HazMat-related incident.
B. Training includes the types of hazardous materials present and their risks to the rescuers and the public.
C. Annual refresher training is generally required.
D. Training generally includes classroom instruction and practical training.

22. In the event that OEC Technicians are exposed to an organophosphate or a chemical nerve agent, they may
need to self-administer the nerve agent antidote. Which of the following statements regarding the self-
administration of a nerve agent antidote is false?
A. "If severe symptoms are present, three atropine auto-injectors and three 2-PAM Cl injectors are
administered in rapid succession.”
B. "If the nerve agent has been ingested, exposure may continue for some time, and a relapse is a possibility.”
C. "The medication is administered in a way that is similar to that for an Epi-Pen.”
D. "Depending on the symptoms observed, the initial treatment will either be an atropine auto-injector or a 2-
PAM Cl injector."

23. HAZWOPER training required by the federal government for anyone who works in an environment in which
uncontrolled hazardous materials may be encountered. Each letter stands for a word. Which is the correct word
for a letter in HAZWOPER?
A. E: energy
B. W: water
C. R: rescue

CHAPTER 35
D. O: operations

Answer Key: Chapter 35 - Test


1=A, 2=A, 3=D, 4=C, 5=D, 6=D, 7=B, 8=D, 9=C, 10=A, 11=B, 12=B, 13=A, 14=C, 15=D, 16=B, 17=D,
18=B, 19=C, 20=D, 21=A, 22=D, 23=D


CHAPTER 35
CHAPTER 36
“ALS Interface”

QUIZ

1. A paramedic makes several attempts at inserting an endotracheal tube into a critically injured patient but fails
to secure the airway. The next course of action should be for the:
A. OECT to attempt to insert the ET tube.
B. EMTP to declare the patient as "code."
C. OECT to continue with BLS airway maintenance and ventilation skills.
D. OECT to reposition the patient's airway so the EMTP can make another attempt.

2. "White and green over right, smoke over fire" refers to what?
A. Which running lights on the ambulance to use in heavy traffic
B. The LZ for a fire scene
C. The placement of EKG electrodes
D. Triage

3. As an OECT, the most important thing to adhere to when assisting a medic is:
A. First, do no harm.
B. Always defer to a more senior member of the team.
C. Follow the medic's instructions exactly.
D. Refuse to do anything you are not trained to do.

4. ALS care should be summoned under which of the following circumstances?


A. When a patient requires difficult or technical extrication
B. When a patient has obvious mortal injuries
C. When BLS providers do not know what to do
D. When a patient is in severe pain

5. You have set up an IV system for a critically injured patient. The medic is busy attempting to insert an ET tube
and tells you, "Go ahead and insert the catheter into the patient's vein; you have my authorization to do so." At
this point, you should:
A. Ask the medic into which vein the catheter should be inserted.
B. Politely decline.
C. Go ahead and insert the catheter as directed by the medic.
D. Ask another OECT to do so because you do not know how.

6. Who should assume patient care at a scene?


A. The First Responder on the scene
B. The highest legal authority on the scene
C. The senior most responder at the scene

CHAPTER 36
D. The highest-trained prehospital provider on scene

7. A medic has inserted an ET tube and you hear phonation. This indicates that:
A. The tube has ruptured or is leaking.
B. The patient is attempting to ventilate on his or her own.
C. The tube has been properly placed.
D. The OECT failed to squeeze the small pilot bulb.

8. One of the most important tools an OECT can provide to an incoming medic is the ability to:
A. Assist with a cricothyrotomy and other ALS procedures.
B. Assist with setting up an IV.
C. Deliver a solid "short" report.
D. Rapidly assemble equipment for an endotracheal intubation.

9. An OECT may provide ALS assistance under which of the following conditions?
A. If your area or resort permits you to do so, if you have a physician who has accepted responsibility for you
to assist and provides medical direction to do so, and if the state in which you are an OECT allows you to
legally do so
B. An OECT may only determine that ALS care is needed but may not assist in the implementation of said
care
C. If your area or resort permits you to do so
D. A patient is dying and there is no medic, physician or other ALS provider available

Answer Key: Chapter 6 - Quiz


1=C, 2=C, 3=A, 4=D, 5=B, 6=D, 7=A, 8=C, 9=A

TEST

1. You are assisting an ALS provider to initiate IV therapy for a 62 year old with a probable ruptured spleen. She
asks you to help prepare 0.9% normal saline with macrodrip tubing. While preparing the equipment you realize
that she chose this combination because:
A. normal saline is an ideal solution for patients that require the administration of medications.
B. normal saline acts as a catalyst that helps move drugs out of the bloodstream and into the tissues.
C. macrodrip tubing has a small needle within the chamber, which is ideal for the slow infusion of fluids.
D. normal saline is an ideal volume expander that is useful in treating shock.

2. You are caring for a 37-year-old patient and are trying to determine if you might need an ALS provider to
initiate intravenous therapy. From your OEC training, you recall that IV therapy is typically used for patients
with:
A. minor lacerations and facial contusions.
B. diabetes and severe dehydration.
C. cold symptoms and mild diarrhea.

CHAPTER 36
D. ankle sprains.

3. Epinephrine and atropine are medications used to:


A. raise the patient's blood pressure.
B. increase the patient's heart rate.
C. decrease the patient's heart rate.
D. make the heart's rhythm more regular.

4. "Spiking the IV bag" refers to:


A. filling the IV drip chamber until it is approximately half full.
B. pushing the spike of the IV tubing into the port of the solution bag.
C. hanging the IV bag higher than the patient's heart.
D. adding medications to the IV solution before administering it.

5. A candidate in your OEC class is attempting to describe advanced life support (ALS). Which of the following
statements concerning ALS is incorrect?
A. It should be assumed that prompt initiation of ALS will make the difference between life and death.
B. ALS includes advanced airway management and drug therapy.
C. ALS is emergency medical care that exceeds basic life-support measures.
D. All EMTs, paramedics, RNs and PAs are part of the ALS system.

6. A good way to remember how to correctly apply cardiac monitor leads is the phrase:
A. "Red and blue to the left, and fire under smoke."
B. "Green and white to the right, and fire over smoke."
C. "White and green to the right, and smoke over fire."
D. "Blue and white to the right, and smoke over fire."

7. Which of the following statements describes how you initially fill the drip chamber after you have attached the
administration set to the IV fluid bag?
A. Open the sliding clamp lock and keep it positioned higher than the IV fluid bag.
B. Open the sliding clamp lock and let the drip chamber fill passively.
C. Close the sliding clamp lock, and then squeeze and release the drip chamber until it is half full.
D. Close the sliding clamp lock, and then squeeze the bag to force fluid into the drip chamber.

8. The heart's electrical activity is most commonly examined through the use of a(n):
A. ECG.
B. AED.
C. defibrillator.
D. cardioverter.

9. You are placing a patient on a cardiac monitor. In which of the following locations would you place the green
"RL" lead?
A. On the patient's lower left chest wall, on the anterior axillary line
B. On the patient's right thorax, under the clavicle
C. On the patient's left thorax, under the clavicle
D. On the patient's lower right chest wall, on the anterior axillary line.

CHAPTER 36
10. If assisting an ALS provider with endotrachial intubation is permitted by local protocol, OEC Technicians can
check the light source on the laryngoscope blade by:
A. attaching the blade to the handle and raising the blade into position.
B. twisting the light bulb on the laryngoscope blade clockwise.
C. turning the "on" switch at the bottom of the handle.
D. twisting the laryngoscope handle clockwise.

11. Which of the following values is the generally accepted initial setting when setting up a ventilator?
A. FiO2: 100 percent
B. Tidal volume: 500-1000 mL
C. Respiratory rate: 12-20 breaths per minute
D. PEEP: average of 15 mm Hg

12. A medication used to reduce blood pressure or to alleviate ischemic cardiac pain is:
A. lidocaine.
B. nitroglycerin.
C. albuterol.
D. dopamine.

13. You have gathered all the equipment an ALS provider needs to initiate IV therapy. Before assembling the
equipment, which of the following actions should you take first?
A. Check the expiration date on the IV bag.
B. Open the IV catheter and hand it to the ALS provider.
C. Place a tourniquet on the patient's arm.
D. Remove the protective cap from the IV tubing spike.

14. Which of the following sites for affixing electrodes to the chest would not obscure the transmission of an
electrical signal?
A. Bony prominences such as the clavicle
B. The fleshy part of the lower abdomen
C. Areas with significant chest hair
D. Any area of skin that has blood or sweat on it

15. You are instructing an OEC class on how to assist an ALS provider with defibrillation. Which of the following
statements correctly identifies pad placement?
A. "The negative pad is placed on the upper left chest at the clavicle along the anterior axillary line, whereas
the positive pad is placed about 4 inches over the left nipple along the mid-clavicular line.”
B. "The positive pad is placed about 2-3 inches above the right nipple, and the negative pad is placed between
the shoulder blades.”
C. "The negative pad is placed on the upper right chest beneath the clavicle on the mid-clavicular line,
whereas the positive pad is placed about 2-3 inches beneath the left nipple and 3-4 inches along the
anterior axillary line.”
D. "The positive pad is placed over the left precordium, and the negative pad is placed on the back between
the shoulder blades."

16. The first medication used to treat asthma or COPD is:


A. epinephrine.
B. atropine.

CHAPTER 36
C. diphenhydramine.
D. albuterol.

17. You are assisting an ALS provider in caring for a patient with hypovolemic shock. While gathering equipment
the provider needs to start administering the intravenous fluid, you recall that the most common size of needle
used in emergency medicine is:
A. 20 gauge.
B. 22 gauge.
C. 18 gauge.
D. 14 gauge.

18. Which of the following is one of the "five rights" of medication administration?
A. The patient has the right of refusal.
B. The patient's right arm or leg is the best site of administration.
C. Administering the right dose must be ensured.
D. Place all medications into the ALS provider's right hand.

19. Your friend tells you that her daughter was recently in the emergency department for an asthma attack and
was given something called an MDI. Your friend tries to describe the MDI and its use, and she got most of the
information correct. Which of the following statements is not correct?
A. "Oxygen is used with it to make the medicine an aerosol."
B. "The medication is placed in a bowl and then you screw the cover on tightly."
C. "It is better than a pill because the medication reaches the lower airways quickly."
D. "If you see a mist coming out of the corrugated tubing, then you are not breathing in deeply enough."

20. When an ALS provider has completed intubating your 39-year-old patient, she then checks for proper tube
placement by:
A. noting that the colorimetric CO2 detector is yellow.
B. listening for sounds upon auscultation of the stomach.
C. seeing an equal, bilateral chest rise with each ventilation.
D. listening to either lung.

21. When an ALS provider asks you to perform the "BURP" maneuver while you are assisting with an intubation,
you would:
A. use the palm of one hand to gently put pressure on the patient's abdomen in a backward and slightly
rightward direction.
B. place the second and third fingers of one hand under the chin and lift the head backward and slightly to the
right.
C. use the thumb and index finger of one hand to deliver light pressure on the cricoid membrane in a
backward, upward, and rightward direction.
D. place a hand on either side of the patient's face and hold the head backward and slightly to the right.

22. Which of the following pieces of equipment is not necessary when initiating an IV in a patient?
A. IV tubing
B. A tourniquet
C. A syringe
D. A bag of fluid

CHAPTER 36
23. You are working with a OEC Technician candidate in the first-aid room at the local ski patrol. The candidate
asks why you would decide to call for ALS transport rather than BLS. Which of the following statements would be
an appropriate response?
A. "ALS can give IV dextrose, so I call for it for every diabetic patient regardless of the nature of the problem."
B. "I always prefer to call ALS just in case something goes wrong."
C. "If I have any concerns about the patient's ABCDs, I will call for ALS."
D. "ALS can give pain medication, so it should be called for every patient who is complaining of pain."

24. You are caring for a 56-year-old woman who was struck by another skier and had a hard tumbling fall. Which
of the following signs or symptoms would be most suggestive that an advanced airway may be needed?
A. Respiratory rate of 12
B. Confusion
C. Bleeding from the mouth
D. Cyanotic lips

25. You are preparing equipment an ALS provider needs to perform an intubation. In order to assess that the
endotrachial tube is working correctly and to have it prepared for the provider, which of the following series of
tasks would you perform?
A. Inflate the cuff with 5 cc of water. Check for leaking. Remove the water.
B. Inflate the cuff with 10 cc of air. Place the end of the tube in water and watch for bubbles; if none appear,
leave the air in the cuff.
C. Inflate the cuff with 5 cc of air. As long as it does not leak, leave the air in place with the syringe attached.
D. Inflate the cuff with 10 cc of air. Check for leaking; if none occurs, deflate the cuff.

26. What is the function of the drip chamber in IV tubing?


A. It increases or decreases the flow rate of IV fluid.
B. It increases or decreases the size of the drop of IV fluid entering the chamber.
C. It attaches the IV tubing to the IV catheter.
D. It connects the IV bag to the patient.

Answer Key: Chapter 6 - Test


1=D, 2=B, 3=B, 4=B, 5=D, 6=C, 7=C, 8=A, 9=D, 10=A, 11=C, 12=B, 13=A, 14=B, 15=C, 16=D, 17=C,
18=C, 19=D, 20=C, 21=C, 22=C, 23=C, 24=D, 25=D, 26=A

CHAPTER 36

You might also like